C1, C2 & C3 - Combined Module - (v13) - CL

You might also like

Download as pdf or txt
Download as pdf or txt
You are on page 1of 84

Fifth Edition

Chemistry
Module
Content

C1. Basic Concepts of Chemistry ..................................................................................... 1


C2. Atomic Structure ........................................................................................................... 29
C3. Periodic Properties of Elements ............................................................................. 68
C1 – Basic Concepts of Chemistry 1

Fifth Edition

C1. Basic Concepts of Chemistry


TABLE OF CONTENTS

C1. Basic Concepts of Chemistry 1


C1.1 Mole Concept ……………….……………….……………….……………….……………….……………….…………………………….2
C1.2 Relating Mass and Moles ……………….……………….……………….……………….……………….…………………………….5
C1.3 Stoichiometric Calculations ……………….……………….……………….……………….……………….……………………..….7
C1.4 Limiting and Excess Reagents in a Reaction ……………….……………….……………….……………….……………….10
C1.5 Concentration ……………….……………….……………….……………….……………….……………….……………….……..….13
Test Practice Problems ……………….……………….……………….……………….……………….……………….…………………….15
Answer Key ………………………….……………….……………….……………….……………….……………….…………………..……...23
C1 – Basic Concepts of Chemistry 2

C1.1 Mole Concept


CONCEPTS
1. Atomic mass unit
2. Isotopes
3. Definition of mole

PRE-TEST
Q1. 288 oranges are distributed equally in 24 boxes. How many oranges are present in 7 boxes?
Q2. The weight of 21 boxes of paper is 45 𝑘𝑔. How many boxes when taken together weigh 15 𝑘𝑔?
Q3. 12 dozens make a ream. How many articles are present in 20 reams? (1 dozen = 12 articles)
Q4. A typical tetra-pack of nuts contains 2 almonds, 4 cashew nuts and 3 pistachio nuts. If you have 35 tetra-packs,
what is the total number of almonds and cashew nuts that you have?
Q5. Every classroom in a school has 13 girls and 12 boys. If the total number of boys in the school is 300, how many
girls are there in the school?

PRE-READING

Category Book Name (Edition) Chapter Section

REQUIRED NCERT Class XI 1 1.6, 1.7, 1.8

NOTE: We have used various combinations of 6.022 × 1023 and 6.023 × 1023 to represent Avogadro’s number (𝑁𝐴 ) in
the following questions.

PRE-READING EXERCISE
Q1. In a _________ (homogeneous/heterogeneous) mixture, the components completely mix with each other and its
composition is uniform throughout.
Q2. When two or more atoms of different elements combine chemically, a _________ is obtained.
Q3. _________ (physical/chemical) properties can be measured without changing the identity or composition of the
substance.
Q4. One atomic mass unit is defined as ________ the mass of a 𝐶 − 12 atom.

IN-CLASS EXERCISE
LEVEL 1
Q1. Compute the molecular mass of the following given molecules using atomic mass of constituent atoms/elements:
I. 𝐶𝐻4 II. 𝐻2 𝑂 III. 𝐶𝐻3 𝐶𝐻2 𝐶𝐻3 IV. 𝐻𝐶𝑙
Q2. A sample of an element 𝑋 contains 60 atoms each weighing 12 𝑎𝑚𝑢 and 40 atoms each weighing 14 𝑎𝑚𝑢 .
Calculate the average atomic mass of the element 𝑋.

C1.1
C1 – Basic Concepts of Chemistry 3

Q3. Calculate:
I. the number of carbon atoms in 1.5 𝑚𝑜𝑙𝑒𝑠 of 𝐶2 𝐻6
II. the total number of atoms in 3 𝑚𝑜𝑙𝑒𝑠 of 𝐶3 𝐻8

LEVEL 2
Complete the table given below:

Number of moles of Number of moles of


Number of molecules Number of 𝐻 atoms in
Q. No. Compound molecules of the 𝐻 atoms in the
of the compound the compound
compound compound

Q4. 𝐻2 𝑂 5 𝑚𝑜𝑙𝑒𝑠

Q5. 𝐻2 𝑆 2 𝑚𝑜𝑙𝑒𝑠

Q6. 𝐻𝐶𝑙 18.069 × 1023

Q7. 𝐻2 𝑆𝑂4 36.138 × 1023

Q8. Which of the following will have the least number of hydrogen atoms?
I. 1 𝑚𝑜𝑙𝑒 of 𝐻2 II. 1 𝑚𝑜𝑙𝑒 of 𝐻2 𝑆𝑂4
III. 1 𝑚𝑜𝑙𝑒 of 𝐻2 𝑂 IV. 1 𝑚𝑜𝑙𝑒 of 𝐻𝐶𝑙
Q9. If the average atomic mass of 𝐹𝑒 is 56 𝑎𝑚𝑢, determine the percent abundance for 𝐹𝑒-54 and 𝐹𝑒-58, given the
following data:

Isotope Atomic weight Percent abundance

𝐹𝑒-54 54 ?

𝐹𝑒-57 57 50

𝐹𝑒-58 58 ?

HOMEWORK
LEVEL 1
Q1. Answer the following:
I. How many oxygen atoms are present in 4 molecules of 𝐻3 𝑃𝑂4 ?
II. How many oxygen molecules are present in 3 𝑚𝑜𝑙 of 𝑂2 ?
III. How many moles of water molecules are present in 12.046 × 1023 molecules of water?
IV. How many atoms of sulphur are present in 12 𝑚𝑜𝑙𝑒𝑠 of 𝐻2 𝑆𝑂4 ?
V. There are 0.5 𝑚𝑜𝑙 of helium atoms in a container. What is the number of helium atoms in the container?
VI. There are 18.069 × 1023 molecules of 𝐻2 𝑆𝑂4 present in a test tube. Find the number of moles of 𝐻2 𝑆𝑂4
present in the test tube.
Q2. A sample has molecular formula 𝐻𝑌3 𝐶𝑙 and molecular mass equal to 84.5 𝑎𝑚𝑢. Predict the element 𝑌.
C1 – Basic Concepts of Chemistry 4

Q3. In a sample of 400 lithium atoms, it is found that 30 atoms are lithium-6 (6 𝑎𝑚𝑢) and 370 atoms are lithium-
7 (7 𝑎𝑚𝑢). Calculate the average atomic mass of lithium.
Q4. Copper has two stable isotopes 63 65
29𝐶𝑢 and 29𝐶𝑢, whose atomic masses are 62.93 𝑎𝑚𝑢 and 64.92 𝑎𝑚𝑢 respectively.
Their relative abundances in nature are 70 % and 30 % respectively. Calculate the average atomic mass of
copper from the given data.
Q5. In five moles of butane 𝐶4 𝐻10 , calculate the following:
I. Number of moles of carbon atoms II. Number of moles of hydrogen atoms
III. Number of molecules of butane IV. Number of atoms of carbon
V. Number of atoms of hydrogen
LEVEL 2
Q6. There are 3.0115 × 1023 atoms of 𝐻 present in a container containing 𝐶3 𝐻6 . Calculate the number of molecules
of 𝐶3 𝐻6 present in the container.

Complete the following table:

Number of Number of moles Number of 𝐻


Number of Number of 𝐶 atoms in the
Q. No. Compound molecules of the of 𝐻 atoms in the atoms in the
moles compound
compound compound compound

Q7. 𝐶3 𝐻8 𝑂 2 𝑚𝑜𝑙

Q8. 𝐶𝐻4 3 𝑚𝑜𝑙

Q9. 𝐶6 𝐻6 36.138 × 1023

Q10. 𝐶2 𝐻6 𝑂 36.138 × 1023

Q11. 𝐶2 𝐻8 30.115 × 1023

Q12. Magnesium exists as three isotopes, two of which are 𝑀𝑔-24 and 𝑀𝑔-25. The third isotope, 𝑋 is unknown.
However, the percentage of each of them is 78%, 10% and 12% respectively. Relative atomic mass of 𝑀𝑔 is
found to be 24 𝑎𝑚𝑢. Find out the atomic mass of the unknown isotope, 𝑋.

C1.1
C1 – Basic Concepts of Chemistry 5

C1.2 Relating Mass and Moles


CONCEPTS
1. Relation between number of moles and mass of a substance
2. Empirical formula and molecular formula of a substance
3. Mass percentage of any species in a compound

PRE-READING

Category Book Name (Edition) Chapter Section

REQUIRED NCERT Class XI 1 1.8, 1.9

PRE-READING EXERCISE
Q1. The mass of 1 𝑚𝑜𝑙 of a substance is called its _________.
Q2. What is the mass of 1 𝑚𝑜𝑙 of 𝐻2 𝑆?
Q3. How many moles of 𝐶𝐻4 are present in 64 𝑔 of 𝐶𝐻4 ?
Q4. One 𝑚𝑜𝑙𝑒 is the amount of a substance that contains as many particles or entities as there are atoms in exactly
12 𝑔 of the _________ isotope.
Q5. 1 𝑚𝑜𝑙𝑒 of hydrogen atoms = _________ atoms of hydrogen.
Q6. A/An _________ (empirical/molecular) formula represents the simplest whole number ratio of various atoms
present in a compound.

IN-CLASS EXERCISE
LEVEL 1
Q1. How many moles of
I. Atoms are present in 13 𝑔 of potassium
II. Molecules are present in 96 𝑔 of oxygen gas
Q2. Which of the following has the highest number of moles? Which has the highest number of molecules?
A) 6 𝑔 of 𝐻2 B) 128 𝑔 of 𝑂2 C) 36 𝑔 of water
Q3. There are 12.046 × 1023 molecules of water present in a jar. Calculate the mass of water present in the jar.
Q4. Calculate the percent composition by mass of each element in calcium carbonate 𝐶𝑎𝐶𝑂3 .

LEVEL 2
Q5. A compound is found to contain 50% sulphur and 50% oxygen by weight. What is the empirical formula of this
compound? If the molecular weight of this compound is 64 𝑎𝑚𝑢, what is its molecular formula?
Q6. What are the empirical and molecular formulae for a compound with 84% carbon and 16% hydrogen if its
molecular weight is 400 𝑔?
C1 – Basic Concepts of Chemistry 6

Q7. Consider the following four samples. Which of the four samples is the heaviest?
A) 2 𝑚𝑜𝑙 of 𝐶𝐻4 B) 1.5 𝑚𝑜𝑙 of 𝐶2 𝐻6
C) 0.5 𝑚𝑜𝑙 of 𝐶3 𝐻8 D) 0.1 𝑚𝑜𝑙 of 𝐶4 𝐻10

Q8. From 392 𝑚𝑔 of 𝐻2 𝑆𝑂4 , 1.204 × 1021 molecules are removed. How many moles of 𝐻2 𝑆𝑂4 are left?
Q9. 213 𝑔 of the oxide of a metal 𝑀, which has the formula 𝑀𝑂2 is found to contain 3 𝑚𝑜𝑙 of molecules. Determine
the atomic mass of the metal and hence, try to deduce what metal this is likely to be. (You can use the periodic
table to deduce the element by matching masses)
Q10. The atomic weights of two elements 𝐴 and 𝐵 are 40 𝑎𝑚𝑢 and 80 𝑎𝑚𝑢 respectively. If 𝑋 𝑔 of 𝐴 contains 𝑌 atoms,
how many atoms are present in 2𝑋 𝑔 of 𝐵?
Q11. Which of the following contains more carbon atoms: 9 𝑔 of glucose (𝐶6 𝐻12 𝑂6 ) or 12 𝑔 of ethanol (𝐶2 𝐻6 𝑂)?

HOMEWORK
LEVEL 1
Q1. How many moles of
I. Atoms are present in 219 𝑔𝑟𝑎𝑚𝑠 of strontium (𝑆𝑟 = 87.6)
II. Molecules are present in 17 𝑔𝑟𝑎𝑚𝑠 𝐻2 𝑂2
Q2. What is the mass of the following species?
I. 10 𝑚𝑜𝑙𝑒𝑠 of 𝐶𝑎 II. 0.25 𝑚𝑜𝑙𝑒𝑠 of water
III. 4 𝑚𝑜𝑙𝑒𝑠 of 𝐶𝑂2 IV. 1 𝑚𝑜𝑙𝑒 of 𝐾𝐶𝑙𝑂4
Q3. How many atoms are present in a 3 𝑔𝑟𝑎𝑚 sample of sodium (𝑁𝑎)?
Q4. How many moles of 𝑁𝑎𝑂𝐻 are present in 90 𝑔𝑟𝑎𝑚 of 𝑁𝑎𝑂𝐻?
Q5. Calculate the percent by mass of sodium (𝑁𝑎) and chlorine (𝐶𝑙) in sodium chloride (𝑁𝑎𝐶𝑙).
Q6. It has been estimated that 93% of all atoms in the entire universe are hydrogen and that the vast majority of
those remaining are helium. Based on only these two elements, estimate the mass percentage composition of the
universe.
LEVEL 2
Q7. Calculate the total number of atoms of each element present in 122.5 𝑔 of 𝐾𝐶𝑙𝑂3 .
Q8. Calculate the atomic mass of element X, if 2 moles of 𝑋𝐶𝑙2 has a mass of 190 𝑔.
Q9. One atom of an element X weights 6.67 × 10−23 𝑔. Calculate the number of moles in 40 𝑘𝑔 of the same element.
(𝑁𝐴 ≈ 6 × 1023 )
Q10. From 220 𝑚𝑔 of 𝐶𝑂2 , 1021 molecules are removed. How many moles of 𝐶𝑂2 are left? (𝑁𝐴 ≈ 6 × 1023 )
Q11. A metal 𝑀 of atomic weight 55 𝑎𝑚𝑢 has a density of 8 𝑔/𝑐𝑐. Calculate the apparent volume occupied by one atom
of the metal.
Q12. If a mole were to contain 1 × 1024 particles, what would be the mass of
I. A single oxygen molecule? II. One mole of oxygen

C1.2
C1 – Basic Concepts of Chemistry 7

C1.3 Stoichiometric Calculations


CONCEPTS
1. Principle of atom conservation (POAC)
2. Balancing chemical reactions

PRE-READING

Category Book Name (Edition) Chapter Section

REQUIRED NCERT Class XI Chemistry 1 1.10 (Except 1.10.1, 1.10.2)

ADDITIONAL PRE-READING
Based on the type of reactants involved or nature of products formed, reactions are classified into different
categories. One such type is explained below
Combustion Reactions - Combustion usually occurs when a hydrocarbon reacts with oxygen to produce carbon dioxide
and water. Combustion is an exothermic reaction, so it releases heat, for e.g.,
Combustion of methane: 𝐶𝐻4 (𝑔) + 2𝑂2 (𝑔) → 𝐶𝑂2 (𝑔) + 2𝐻2 𝑂(𝑔)
In general, for any compound that has only 𝐶, 𝐻 atoms:
𝑦 𝑦
𝐶𝑥 𝐻𝑦 + (𝑥 + ) 𝑂2 → 𝑥𝐶𝑂2 + 𝐻2 𝑂
4 2

PRE-READING EXERCISE
Q1. How many moles of water are formed when 2 𝑚𝑜𝑙𝑒𝑠 of hydrogen react with 1 𝑚𝑜𝑙𝑒 of oxygen?
Q2. In the reaction, 2𝐶 + 𝑂2 → 2𝐶𝑂, for every mole of oxygen used in the reaction, _________ moles of carbon are
used up and _________ moles of carbon monoxide are formed.
Q3. Balance the following chemical equations:
I. 𝐹𝑒 + 𝐻2 𝑆04 → 𝐹𝑒2 (𝑆𝑂4 )3 + 𝐻2 II. 𝐶2 𝐻6 + 𝑂2 → 𝐻2 𝑂 + 𝐶𝑂2
III. 𝐾𝑂𝐻 + 𝐻3 𝑃𝑂4 → 𝐾3 𝑃𝑂4 + 𝐻2 𝑂 IV. 𝑆𝑛𝑂2 + 𝐻2 → 𝑆𝑛 + 𝐻2 𝑂

IN CLASS EXERCISE
LEVEL 1
Q1. Balance the following equations using the principle of atom conservation:
I. 𝐹𝑒𝑆2 + 𝑂2 → 𝐹𝑒2 𝑂3 + 𝑆𝑂2 II. 𝐶2 𝐻5 + 𝑂2 → 𝐶𝑂2 + 𝐻2 𝑂
Q2. If 16 moles of 𝑋 are present in each of the following reactions, determine the amount of 𝑌 needed for a complete
reaction. Assume that these reactions are balanced.
I. 𝑋 + 2𝑌 → 2𝑍 + 3𝑊 II. 2𝑋 + 𝑌 → 3𝑍 + 𝑊
Q3. Diborane (𝐵2 𝐻6 ) is formed according to the reaction
𝐵4 + 6𝐻2 → 2𝐵2 𝐻6 . Answer the following questions:
I. How many moles of boron are required to completely react with 10 𝑚𝑜𝑙𝑒𝑠 of hydrogen?
II. How many moles of diborane can be prepared from 2 𝑚𝑜𝑙𝑒𝑠 of boron?
III. How many moles of hydrogen are needed to form 7 𝑚𝑜𝑙𝑒𝑠 of diborane?
C1 – Basic Concepts of Chemistry 8

LEVEL 2
Q4. How many grams of chlorine can be liberated from the decomposition of 60.5 𝑔𝑟𝑎𝑚 of 𝐴𝑢𝐶𝑙3 by the reaction:
2𝐴𝑢𝐶𝑙3 → 2𝐴𝑢 + 3𝐶𝑙2 (atomic mass of 𝐴𝑢 = 196 𝑎𝑚𝑢)
Q5. Sodium carbonate dissolves in water to form sodium bicarbonate according to the reaction:
𝑁𝑎2 𝐶𝑂3 + 𝐻2 𝑂 + 𝐶𝑂2 → 2𝑁𝑎𝐻𝐶𝑂3 .
Determine the mass of water and sodium carbonate needed to form 84 𝑔𝑟𝑎𝑚𝑠 of sodium bicarbonate.
Q6. If 490 𝑔𝑟𝑎𝑚𝑠 of 𝐾𝐶𝑙𝑂3 are taken and heated strongly, what would be the loss in weight of the sample?
2𝐾𝐶𝑙𝑂3 → 2𝐾𝐶𝑙 + 3𝑂2 .
Given mass of 𝐾 = 39.
Q7. 25.4 𝑔 of iodine and 14.2 𝑔 chlorine are made to react completely to yield a mixture of 𝐼𝐶𝑙 and 𝐼𝐶𝑙3 . Determine
the ratio of moles of 𝐼𝐶𝑙 and 𝐼𝐶𝑙3 so formed (𝐶𝑙 − 35.5 𝑎𝑚𝑢, 𝐼 − 127 𝑎𝑚𝑢).

LEVEL 3
Q8. 2.4 𝑔 of a mixture of 𝐵𝑎𝑂 and 𝐶𝑎𝑂 when reacted with enough 𝐻2 𝑆𝑂4 produced 6 𝑔 of the mixed sulphates. Find
the percentage of 𝐵𝑎𝑂 present in the mixture. Assume the following molecular weights to solve this.
(𝐵𝑎𝑆𝑂4 = 250 𝑎𝑚𝑢, 𝐶𝑎𝑆𝑂4 = 150 𝑎𝑚𝑢, 𝐶𝑎𝑂 = 50 𝑎𝑚𝑢, 𝐵𝑎𝑂 = 150 𝑎𝑚𝑢).

HOMEWORK
LEVEL 1
Q1. Ethylene 𝐶2 𝐻4 burns in oxygen to give carbon dioxide and water. Write the complete balanced reaction.
Q2. Butane burns according to the reaction 2𝐶4 𝐻10 + 13𝑂2 → 8𝐶𝑂2 + 10𝐻2 𝑂.
I. What amount of oxygen is needed to produce 10 𝑚𝑜𝑙𝑒𝑠 of carbon dioxide?
II. If 3 𝑚𝑜𝑙𝑒𝑠 water is produced, how many moles of 𝐶𝑂2 are produced?
LEVEL 2
Q3. Nitric acid 𝐻𝑁𝑂3 is manufactured by the Ostwald process according to the following reaction: 3𝑁𝑂2 + 𝐻2 𝑂 →
2𝐻𝑁𝑂3 + 𝑁𝑂. How many 𝑔 of nitrogen dioxide (𝑁𝑂2 ) are required in this reaction to produce 6.3 𝑔 of nitric
acid?
Q4. 138 𝑔 of silver carbonate on being strongly heated releases a gas and decreases in weight. What will be the weight
of the residue? The reaction is (𝐴𝑔 = 108 𝑢)
𝐴𝑔2 𝐶𝑂3 (𝑠) → 𝐴𝑔2 𝑂(𝑠) + 𝐶𝑂2 (𝑔)
Q5. What weight of gaseous product will be produced when 10 𝑔 of oxalic acid – 𝐻2 𝐶2 𝑂4 is reacted with 𝐻2 𝑆𝑂4 ? The
reaction you need to use is
𝐻2 𝐶2 𝑂4 → 𝐶𝑂 + 𝐶𝑂2 + 𝐻2 𝑂
Q6. Copper metal reacts with nitric acid according to the reaction (𝐶𝑢 = 63.5 𝑢)
3𝐶𝑢 + 8𝐻𝑁𝑂3 → 3𝐶𝑢(𝑁𝑂3 )2 + 2𝑁𝑂 + 4𝐻2 𝑂
If 9.375 𝑔 copper nitrate is eventually formed, how many 𝑔 of 𝑁𝑂 are formed?
Q7. Acrylonitrile, 𝐶3 𝐻3 𝑁 is produced from propylene 𝐶3 𝐻6 by reacting with nitric oxide. How many grams of
acrylonitrile can be produced from 630 kg propylene? The balanced reaction is
3 3 1
𝐶3 𝐻6 + 𝑁𝑂 → 𝐶3 𝐻3 𝑁 + 𝐻2 𝑂 + 𝑁2
2 2 4

C1.3
C1 – Basic Concepts of Chemistry 9

Q8. A 1 𝑔 mixture of 𝐶𝑎𝐶𝑂3 and 𝐶𝑎𝑂 is reduced to yield 0.6 𝑔 of metallic calcium. Determine the percent composition
of the original mixture. (𝐶𝑎 = 40 𝑢)
Q9. Given the following reaction:
𝐻2 𝑆(𝑔) + 𝑂2 (𝑔) → 𝑆𝑂2 (𝑔) + 𝐻2 𝑂(𝑠)
How many atoms of oxygen are needed in order to get 18 𝑔𝑟𝑎𝑚 of ice?

LEVEL 3
Q10. 410 grams of limestone is treated with oxalic acid to yield 512 𝑔𝑟𝑎𝑚𝑠 of calcium oxalate. What is the mass
percentage of calcium carbonate in this limestone? The reaction is: 𝐶𝑎𝐶𝑂3 + 𝐻2 𝐶2 𝑂4 → 𝐶𝑎𝐶2 𝑂4 + 𝐻2 𝑂 + 𝐶𝑂2
Q11. What weight of 𝐶𝑂 is required to form 𝑅𝑒2 (𝐶𝑂)10 from 3 𝑔 of 𝑅𝑒2 𝑂7 according to the unbalanced reaction:
𝑅𝑒2 𝑂7 + 𝐶𝑂 → 𝑅𝑒2 (𝐶𝑂)10 + 𝐶𝑂2 (Assume 𝑅𝑒2 𝑂7 = 476 𝑢, 𝐶 = 12 𝑢 and 𝑂 = 16 𝑢)
C1 – Basic Concepts of Chemistry 10

C1.4 Limiting and Excess Reagents in a Reaction


CONCEPTS
1. Limiting reagent and excess reagents
2. Percentage purity
3. Yield of a reaction

PRE-READING

Category Book Name (Edition) Chapter Section

REQUIRED NCERT Class XI Chemistry 1 1.10.1

ADDITIONAL PRE READING


In many cases, the reactions go to completion so that the yield of the reaction is 100%. But in some cases, side
reactions take place and the yield is not 100% but less than that. Percent yield of the reaction is given by
actual yield of product
Percent yield = × 100
theoretical yield of product
A good example where such a calculation finds use is that of extraction of metals from their ores. Generally, when
mining is done, we get a lot of impurities along with the species of interest. In such a case, you need to know what
quantity of chemicals to add to carry out the necessary reaction to get pure metal. To do this calculation we can use
basic concepts of stoichiometry along with the idea of percentage yield as mentioned above.

PRE-READING EXERCISE
Q1. Consider the reaction 𝐴 + 2𝐵 → 𝐶 . If we allow 3 𝑚𝑜𝑙 of 𝐴 to react with 4 𝑚𝑜𝑙 of 𝐵 , the limiting reagent
is_____________
Q2. 2 𝑚𝑜𝑙 of reactant 𝑋 is made to react with 3 𝑚𝑜𝑙 of reactant 𝑌 according to the reaction 𝑋 + 2𝑌 → 𝑍. 𝑋 is the
limiting reagent. ( True / False)
Q3. Given the reaction 2𝐴 + 3𝐵 → 𝐶 + 2𝐷, answer the following:
I. 2 𝑚𝑜𝑙𝑒𝑠 of 𝐴 require _________ 𝑚𝑜𝑙 of 𝐵 for reaction to complete
II. 1 𝑚𝑜𝑙𝑒 of 𝐵 will yield a maximum of _________ 𝑚𝑜𝑙 of 𝐶
III. 2 𝑚𝑜𝑙𝑒𝑠 of 𝐶 would be formed from _________ 𝑚𝑜𝑙 of 𝐴 and _________ 𝑚𝑜𝑙 of 𝐵

IN CLASS EXERCISE
LEVEL 1
Q1. Classify the following reactions as stoichiometric or non-stoichiometric based on the amounts of reactants given:
I. 1 𝑚𝑜𝑙 𝐶𝑆2 and 2 𝑚𝑜𝑙 oxygen reacting as 𝐶𝑆2 + 3𝑂2 → 𝐶𝑂2 + 2𝑆𝑂2
II. 4 𝑚𝑜𝑙 𝑁𝑎𝑂𝐻 and 4 𝑚𝑜𝑙 𝐻𝐶𝑙 reacting as 𝑁𝑎𝑂𝐻 + 𝐻𝐶𝑙 → 𝑁𝑎𝐶𝑙 + 𝐻2 𝑂
III. 4 𝑚𝑜𝑙 𝑁𝐻3 , 2 𝑚𝑜𝑙𝑒 oxygen and 4 𝑚𝑜𝑙 𝐶𝐻4 reacting as 2𝑁𝐻3 + 3𝑂2 + 2𝐶𝐻4 → 2𝐻𝐶𝑁 + 6𝐻2 𝑂

C1.4
C1 – Basic Concepts of Chemistry 11

Q2. 𝐴 and 𝐵2 react according to the balanced reaction: 3𝐴 + 2𝐵2 → 𝐴3 𝐵4 . Shown below are three possible reaction
mixtures of 𝐴 and 𝐵2 . In each case, identify the limiting reagent.
I. 6 𝑚𝑜𝑙 of 𝐴 and 3 𝑚𝑜𝑙 of 𝐵2 II. 10 𝑚𝑜𝑙 of 𝐴 and 20 𝑚𝑜𝑙 of 𝐵2
III. 9 𝑚𝑜𝑙 of 𝐴 and 3 𝑚𝑜𝑙 of 𝐵2
Q3. Which of the following reaction mixtures will not be completely used up after reacting?
I. 30 𝑔 𝑁𝑎𝑂𝐻 and 73 𝑔 𝐻𝐶𝑙 as per 𝑁𝑎𝑂𝐻 + 𝐻𝐶𝑙 → 𝑁𝑎𝐶𝑙 + 𝐻2 𝑂
II. 63.5 𝑔 𝐶𝑢 and 126 𝑔 of 𝐻𝑁𝑂3 as per 𝐶𝑢 + 2𝐻𝑁𝑂3 → 𝐶𝑢(𝑁𝑂3 )2 + 𝐻2
LEVEL 2
Q4. The reaction between aluminum and iron (III) oxide is used to weld metals. The reaction is 2𝐴𝑙 + 𝐹𝑒2 𝑂3 →
𝐴𝑙2 𝑂3 + 2𝐹𝑒. If 108 𝑔 of aluminum is made to react with 160 𝑔 of the iron oxide, how many moles of iron metal
are generated in the process? Also, compute the weight of the excess reagent left in the reaction mixture.
Q5. A reaction mixture containing 2 𝑚𝑜𝑙𝑒𝑠 of 𝑁2 and 7 𝑚𝑜𝑙𝑒𝑠 of hydrogen is used to produce ammonia, which is
liquefied and separated. What mass of oxygen will be required to completely react with the excess reagent
remaining in the reaction mixture? If 𝑁2 is present in excess then the compound formed is 𝑁𝑂. If 𝐻2 is present in
excess then 𝐻2 𝑂 is formed.
Q6. Diethyl ether (𝐶2 𝐻5 𝑂𝐶2 𝐻5 ) is prepared from ethyl alcohol (𝐶2 𝐻5 𝑂𝐻) in the presence of an acid
140℃, 𝑎𝑐𝑖𝑑
2𝐶2 𝐻5 𝑂𝐻(𝑙) → 𝐶2 𝐻5 𝑂𝐶2 𝐻5 (𝑙) + 𝐻2 𝑂(𝑙)
Calculate % yield if 46 𝑔 of ethyl alcohol gives 30 𝑔 ether.
Q7. Methyl salicylate is prepared by heating salicylic acid with methanol according to the reaction :
𝐶7 𝐻6 𝑂3 + 𝐶𝐻3 𝑂𝐻 → 𝐶8 𝐻8 𝑂3 + 𝐻2 𝑂.
In the reaction, 27.6 𝑔 of salicylic acid (𝐶7 𝐻6 𝑂3 ) reacts with 10.8 𝑔 of methanol (𝐶𝐻3 𝑂𝐻). How much methyl
salicylate (𝐶8 𝐻8 𝑂3 ) will be formed?

LEVEL 3
Q8. What mass of a 80% pure sample (by mass) of 𝐾𝐶𝑙𝑂3 is required to completely react with 48 𝑔 of methane,
according to the following reactions:
2𝐾𝐶𝑙𝑂3 → 2𝐾𝐶𝑙 + 3𝑂2
𝐶𝐻4 + 2𝑂2 → 𝐶𝑂2 + 2𝐻2 𝑂

HOMEWORK
LEVEL 1
Complete the following table for each of the reactions mentioned. Assume that the reactions are in balanced form:

Reaction Moles of 𝐴 Moles of 𝐵2 Limiting reagent Moles of Product formed

Q1. 4𝐴 + 2𝐵2 → 3𝐶 2 3

Q2. 𝐴 + 𝐵2 → 𝐶 4 2

Q3. 𝐴 + 4𝐵2 → 𝐶 5 4

Q4. 2𝐴 + 𝐵2 → 2𝐴𝐵 10 20
C1 – Basic Concepts of Chemistry 12

Q5. Distinguish whether the following reactions are stoichiometric or non- stoichiometric:
I. 84 𝑔𝑟𝑎𝑚𝑠 𝐾𝑂𝐻 and 38 𝑔𝑟𝑎𝑚𝑠 𝐹2 reacting as 2𝐾𝑂𝐻 + 2𝐹2 → 2𝐾𝐹 + 𝐹2 𝑂 + 𝐻2 𝑂
II. 138 𝑔𝑟𝑎𝑚𝑠 𝑁𝑂2 and 9 𝑔𝑟𝑎𝑚𝑠 𝐻2 𝑂 reacting as 3𝑁𝑂2 + 𝐻2 𝑂 → 2𝐻𝑁𝑂3 + 𝑁𝑂

LEVEL 2
Q6. Consider the reaction 𝐴 + 𝐵2 → 𝐴𝐵2 . The atomic mass of 𝐴 = 10 𝑎𝑚𝑢 and the atomic mass of 𝐵 is equal to
6 𝑎𝑚𝑢. For each of the following reaction mixtures, identify the limiting reagent and calculate the number of
moles of the product that will be formed.
I. 20𝑔 𝐴 + 12𝑔 𝐵2 II. 50𝑔 𝐴 + 100𝑔 𝐵2 III. 100𝑔 𝐴 + 120𝑔 𝐵2

Q7. Assume that you have 2 𝑚𝑜𝑙 of 𝑁2 and 5 𝑚𝑜𝑙 of 𝐻2 .


I. How many 𝑔 of ammonia (𝑁𝐻3 ) can you make?
II. Which reactant will be left over? Compute the mass left over.
III. How many 𝑔 of the limiting reactant will be required to react with the reactant present in larger amount?

Q8. Hydrogen cyanide is prepared from ammonia according to the reaction


2𝑁𝐻3 + 3𝑂2 + 2𝐶𝐻4 → 2𝐻𝐶𝑁 + 6𝐻2 𝑂
A reaction vessel contains 1.7 𝑔 𝑁𝐻3 , 16 𝑔 𝑂2 and 16 𝑔 𝐶𝐻4 . What is the maximum mass of hydrogen cyanide that
could be made?
Q9. How many moles of sodium chloride will be formed if 219 𝑔𝑟𝑎𝑚𝑠 of hydrochloric acid reacts with 150 𝑔𝑟𝑎𝑚𝑠 of
sodium hydroxide?
Q10. Urea (𝑁𝐻2 )2 𝐶𝑂 is prepared by reacting ammonia with carbon dioxide.
2𝑁𝐻3 (𝑔) + 𝐶𝑂2 (𝑔) → (𝑁𝐻2 )2 𝐶𝑂 (𝑎𝑞) + 𝐻2 𝑂(𝑙)
In one process, 680 𝑔 of 𝑁𝐻3 are allowed to react with 1100 𝑔 of 𝐶𝑂2 .
I. Which of the two reactants is the limiting reagent?
II. Calculate the mass of urea formed.
III. How much of the excess reagent (in 𝑔) is left at the end of the reaction?
Q11. A 15.6 𝑔 sample of 𝐶6 𝐻6 is mixed with excess 𝐻𝑁𝑂3 in presence of 𝐻2 𝑆𝑂4 . The reaction taking place is:
𝐻2 𝑆𝑂4
𝐶6 𝐻6 + 𝐻𝑁𝑂3 → 𝐶6 𝐻5 𝑁𝑂2 + 𝐻2 𝑂
We isolate 18 𝑔 of 𝐶6 𝐻5 𝑁𝑂2 . What is the percent yield of 𝐶6 𝐻5 𝑁𝑂2 in the reaction?
LEVEL 3
Q12. When a mixture of 𝑁𝑎𝐵𝑟 and 𝑁𝑎𝐶𝑙 is repeatedly reacted with sulphuric acid, all the 𝐵𝑟 and 𝐶𝑙 is expelled out and
𝑁𝑎2 𝑆𝑂4 is formed. If with a particular mixture, weight of 𝑁𝑎2 𝑆𝑂4 is same as weight of 𝑁𝑎𝐵𝑟 and 𝑁𝑎𝐶𝑙 mixture,
calculate the ratio of masses of 𝑁𝑎𝐵𝑟 and 𝑁𝑎𝐶𝑙.

C1.4
C1 – Basic Concepts of Chemistry 13

C1.5 Concentration
CONCEPTS
1. Concentration terms for a solution
2. Mole fraction
3. Concentration change on mixing solutions

PRE-READING

Category Book Name (Edition) Chapter Section

REQUIRED NCERT Class XI Chemistry 1 1.10.2

PRE-READING EXERCISE
Q1. A _________ (solute/solvent) is the dissolved substance in the solution.
Q2. A 1 liter solution of 2 𝑀 𝑁𝑎𝑂𝐻 solution will contain __________ moles of 𝑁𝑎𝑂𝐻.
Q3. 1 mole of 𝑁𝑎𝐶𝑙 is dissolved in 2 𝑘𝑔 of water. Calculate the molality of the solution.
Q4. If a solution of 𝑁𝑎𝐶𝑙 is diluted by adding more water, then the molarity of the solution will decrease. [True/False]
Q5. 2 𝑚𝑜𝑙 of 𝑁𝑎𝑂𝐻 are present in a 1 𝑙𝑖𝑡𝑟𝑒 solution. Calculate the molarity of the solution.

IN-CLASS EXERCISE
LEVEL 1
Q1. Using the information given below, express the strength of the resultant solution in the units mentioned.
I. Molarity of a 5 𝐿 aqueous solution containing 196 𝑔 of 𝐻2 𝑆𝑂4 dissolved in it
II. Molality of a solution formed by mixing 4 𝑚𝑜𝑙 of 𝐻𝐶𝑙 in 6 𝑘𝑔 of water
III. Strength in 𝑤/𝑣 of a 4 𝐿 solution formed by dissolving 2 𝑚𝑜𝑙 of 𝑁𝑎𝑂𝐻 in water.

Q2. 7 𝑚𝑜𝑙 of 𝐶𝑎(𝑂𝐻)2 are mixed with 13 𝑚𝑜𝑙 water to prepare a solution. Calculate the mole fraction of water in the
solution.
Q3. What mass of table sugar (𝐶12 𝐻22 𝑂11 ) do you need to prepare 100 𝑚𝐿 of a 1 𝑀 aqueous solution of sugar?

LEVEL 2
Q4. Compute the following:
I. Molarity of a 6.25 𝑚 solution of 𝑁𝑎𝑂𝐻 in water. Assume density of solution is 1 𝑔/𝑐𝑐.
II. Molality of a 2 𝐿 solution formed by dissolving 30 𝑔 of urea (𝑁𝐻2 𝐶𝑂𝑁𝐻2 ) in water. Assume density of
solution is 1.265 𝑔/𝑐𝑐.
Q5. Calculate the molarity of the resulting solution in 50 𝑚𝐿 of 1𝑀 𝑁𝑎𝑂𝐻 solution mixed with 100 𝑚𝐿 of 3𝑀 𝑁𝑎𝑂𝐻
solution.
Q6. Calculate the molarity of 𝐻𝐶𝑙 left unreacted if a mixture is formed by mixing 50 𝑚𝐿 of 0.2 𝑀 𝐻𝐶𝑙 solution with
25 𝑚𝐿 of 0.1 𝑀 𝑁𝑎𝑂𝐻 solution.
C1 – Basic Concepts of Chemistry 14

Q7. If a 6 𝑀 solution of 𝐻𝐶𝑁 in water has a molality 5.3 𝑚𝑜𝑙𝑎𝑙, find the density in 𝑔/𝑐𝑐 of the solution. (Density of
water = 1𝑔/𝑐𝑐 = 1𝑔/𝑚𝐿)

LEVEL 3
Q8. Calculate the concentration of hydrochloric acid in moles per litre in a sample which has a density, 1.095 𝑔 𝑚𝐿−1 ,
the mass per cent of the acid in it being 30%.

HOMEWORK
LEVEL 1
Q1. 2 𝑚𝑜𝑙 of 𝐻𝐶𝑙 are added to 3 𝑘𝑔 of water to prepare a solution. Calculate the molality of 𝐻𝐶𝑙 in the solution.
Q2. 8 𝑚𝑜𝑙 of 𝐾𝑂𝐻 are present in a 4 𝑙𝑖𝑡𝑟𝑒 solution. Calculate the molarity of 𝐾𝑂𝐻 in the solution.
Q3. 50 𝑚𝐿 of a 2 𝑀 𝑁𝑎𝑂𝐻 solution is taken in a container. Calculate the number of moles of 𝑁𝑎𝑂𝐻 present in the
container.
Q4. 294 𝑔 of 𝐻2 𝑆𝑂4 are added with 3 𝑘𝑔 of water to prepare a solution. Calculate the molality of the solution.
Q5. How many litres of 0.5 𝑀 𝐵𝑎𝐶𝑙2 aqueous solution contain 20.8 𝑔 of 𝐵𝑎𝐶𝑙2 ?

LEVEL 2
Q6. Hydrochloric acid is sold commercially as a 15.0 𝑀 aqueous solution. How many moles and how many grams of
𝐻𝐶𝑙 are in 500.0 𝑚𝐿 of 15.0 𝑀 solution?
Q7. Calculate the molality of the solution. (Density of water = 1𝑔/𝑐𝑐 which is the same as 1𝑔/𝑚𝑙)
I. 73 𝑔 of 𝐻𝐶𝑙 is added to 200 𝑚𝑙 of water to prepare a solution
II. 80 𝑔 of 𝑁𝑎𝑂𝐻 is added to 500 𝑚𝑙 of water to prepare a solution.
Q8. You have a solution that is 2 𝑀 𝐻2 𝑆𝑂4 (sulfuric acid). How many milliliters of this acid do you need to prepare
200 𝑚𝐿 of 0.5 𝑀 𝐻2 𝑆𝑂4 ?
Q9. 250 𝑚𝑙 of 5𝑀 𝐻𝐶𝑙 solution is taken in a beaker. Water is then added to the beaker to make the volume of the
solution as 4 𝑙𝑖𝑡𝑟𝑒. Calculate the molarity of the resulting solution.
LEVEL 3
Q10. Consider the following reaction:
𝐵𝑎𝐶𝑙2 (𝑎𝑞) + 𝐻2 𝑆𝑂4 (𝑎𝑞) → 𝐵𝑎𝑆𝑂4 (𝑠) + 2𝐻𝐶𝑙(𝑎𝑞)
50 𝑚𝐿 of 20 𝑀 𝐵𝑎𝐶𝑙2 (𝑎𝑞) and 100 𝑚𝐿 of 15𝑀 𝐻2 𝑆𝑂4 (𝑎𝑞) solutions are mixed. Which reactant is left over after
the reaction? What is the mass of this reactant left over?
(𝐵𝑎 = 137, 𝐶𝑙 = 35.5, 𝑆 = 32, 𝐻 = 1, 𝑂 = 16)
Q11. In the reaction between barium nitrate and sodium sulphate, how many grams of sodium nitrate can be prepared
from 1 𝑚𝐿 of 1 𝑀 barium nitrate?
The reaction is: 𝐵𝑎(𝑁𝑂3 )2 + 𝑁𝑎2 𝑆𝑂4 → 𝐵𝑎𝑆𝑂4 + 2𝑁𝑎𝑁𝑂3

C1.5
C1 – Basic Concepts of Chemistry 15

Test Practice Problems


Purpose: To practice a mixed bag of questions in a speed based format similar to what you will face in entrance
examinations. In most entrance examinations, you will get not more than 3 minutes to attempt a question. Hence,
you need to be able to attempt a question in less than 3 minutes, and at the end of 3 minutes skip the question
and move to the next one.
Approach:
 Attempt the Test Practice Problems only when you have the stipulated time available at a stretch.
 Start a timer and attempt the section as a test.
 DO NOT look at the anwer key / solutions after each question.
 DO NOT guess a question if you do not know it. Competitive examinations have negative marking.
 Solve as much as possible within the stipulated time, and then fill the OMR provided at the end of the TPP.
 Fill the table at the end of the TPP and evaluate the number of attempts, and accuracy of attempts, which will
help you evaluate your preparedness level for the chapter.

TEST PRACTICE PROBLEMS – 1


No. of questions: 30 Total time: 90 Minutes Time per question: 3 Minutes

Q1. In a gaseous reaction of the type, 𝑎𝐴 + 𝑏𝐵 → 𝑐𝐶 + 𝑑𝐷, which statement is wrong?


A) 𝑎 moles of 𝐴 combine with 𝑏 moles of 𝐵 to give 𝐶 and 𝐷
B) 𝑎 grams of 𝐴 combine with 𝑏 grams of 𝐵 to give 𝐶 and 𝐷
C) 𝑎 molecules 𝐴 combine with 𝑏 molecules of 𝐵 to give 𝐶 and 𝐷
D) None of the above
3
Q2. The equation 2𝐴𝑙(𝑠) + 𝑂2 (𝑔) → 𝐴𝑙2 𝑂3 (𝑠) shows that
2
7
A) 2 𝑚𝑜𝑙𝑒𝑠 of 𝐴𝑙 react with 3/2 𝑚𝑜𝑙𝑒𝑠 of 𝑂2 to produce 𝑚𝑜𝑙𝑒𝑠 of 𝐴𝑙2 𝑂3
2
3
B) 2𝑔 of 𝐴𝑙 react with 𝑔 of 𝑂2 to produce 𝑜𝑛𝑒 𝑚𝑜𝑙𝑒 of 𝐴𝑙2 𝑂3
2
3
C) 2𝑔 of 𝐴𝑙 react with 𝑙𝑖𝑡𝑟𝑒𝑠 of 𝑂2 to produce 1 𝑚𝑜𝑙𝑒 of 𝐴𝑙2 𝑂3
2
3
D) 2 𝑚𝑜𝑙𝑒𝑠 of 𝐴𝑙 react with 𝑚𝑜𝑙𝑒𝑠 of 𝑂2 to produce 1 𝑚𝑜𝑙𝑒 of 𝐴𝑙2 𝑂3
2

Q3. 2.5 𝑔 of silver carbonate (275 𝑔/𝑚𝑜𝑙) on being strongly heated yields a residue of 𝐴𝑔 weighing
(atomic mass of 𝐴𝑔 = 108 𝑎𝑚𝑢)
A) 1.96 𝑔Har B) 2.48 𝑔 C) 2.32 𝑔 D) 4.64 𝑔
Q4. If 0.5 𝑚𝑜𝑙 of 𝐵𝑎𝐶𝑙2 is mixed with 0.2 𝑚𝑜𝑙 of 𝑁𝑎3 𝑃𝑂4 , the maximum number of moles of 𝐵𝑎3 (𝑃𝑂4 )2 that can be
formed is
A) 0.7 B) 0.5 C) 0.30 D) 0.1

Q5. Which has maximum % by mass of 𝐶𝑙?


A) 𝐶6 𝐻6 𝐶𝑙6 B) 𝐶𝐻𝐶𝑙3 C) 𝐶6 𝐻5 𝐶𝑙 D) 𝐶𝐻3 𝐶𝑙
Q6. A certain metal sulphide, 𝑀𝑆2 , is used extensively as a high temperature lubricant. If 𝑀𝑆2 is 40% by mass
sulphur, metal 𝑀 has atomic mass:
A) 160 𝑎𝑚𝑢 B) 64 𝑎𝑚𝑢 C) 40 𝑎𝑚𝑢 D) 96 𝑎𝑚𝑢
C1 – Basic Concepts of Chemistry 16

Q7. When 0.300 𝑔 of 𝑀𝑔 is heated strongly in a nitrogen (𝑁2 ) atmosphere, 0.416 𝑔 of the compound is formed.
Hence, compound formed is:
A) 𝑀𝑔3 𝑁2 B) 𝑀𝑔3 𝑁 C) 𝑀𝑔2 𝑁3 D) 𝑀𝑔𝑁
Q8. A sample of ammonium phosphate (𝑁𝐻4 )3 𝑃𝑂4 contains 3.18 𝑚𝑜𝑙 of 𝐻 atoms. The number of moles of 𝑂 atoms
in the sample is:
A) 0.265 B) 0.795 C) 1.06 D) 3.18
Q9. Average atomic mass of magnesium is 24.31 𝑎𝑚𝑢. This magnesium is composed of 79 𝑚𝑜𝑙𝑒 % of 24
𝑀𝑔 and
remaining 21 𝑚𝑜𝑙𝑒 % of 25𝑀𝑔 and 26𝑀𝑔. Calculate mole % of 26𝑀𝑔.
A) 10 B) 1 C) 15 D) 16
Q10. For the reaction, 2𝐹𝑒(𝑁𝑂3 )3 + 3𝑁𝑎2 𝐶𝑂3 → 𝐹𝑒2 (𝐶𝑂3 )3 + 6𝑁𝑎𝑁𝑂3
Initially 2.5 𝑚𝑜𝑙 of 𝐹𝑒(𝑁𝑂3 )2 and 3.6 moles of 𝑁𝑎2 𝐶𝑂3 are taken. If 6.3 𝑚𝑜𝑙 of 𝑁𝑎𝑁𝑂3 is obtained then % yield
of given reaction is:
A) 50 B) 84 C) 87.5 D) 100
Q11. How many moles of 𝑃4 can be produced by reaction of 4 𝑚𝑜𝑙𝑒𝑠 𝐶𝑎5 (𝑃𝑂4 )3 𝐹 , 10 𝑚𝑜𝑙𝑒𝑠 𝑆𝑖𝑂2 and 20 𝑚𝑜𝑙𝑒𝑠 𝐶
according to the following reaction?
4𝐶𝑎5 (𝑃𝑂4 )3 𝐹 + 18𝑆𝑖𝑂2 + 30𝐶 → 3𝑃4 + 2𝐶𝑎𝐹2 + 18𝐶𝑎𝑆𝑖𝑂3 + 30𝐶𝑂
A) 1.66 B) 2 C) 1.9 D) 0.5
Q12. 2.0 𝑔 sample of mixture of 𝑆𝑖𝑂2 and 𝐹𝑒2 𝑂3 , on very strong heating leaves a residue weighing 1.96 𝑔. The reaction
responsible for loss of weight is
𝐹𝑒2 𝑂3 (𝑠) → 𝐹𝑒3 𝑂4 (𝑠) + 𝑂2 (𝑔)
What is the percentage by mass of 𝑆𝑖𝑂2 in original sample? (Atomic weight of 𝐹𝑒 = 56)
A) 10% B) 20% C) 40% D) 60%
Q13. Balance the following equation and choose the quantity which is the sum of the coefficients of products, when all
the coefficients are the smallest possible whole numbers:
𝐶𝑆2 + 𝐶𝑙2 → 𝐶𝐶𝑙4 + 𝑆2 𝐶𝑙2
A) 5 B) 3 C) 6 D) 2
Q14. 100 𝑚𝐿 of 𝐻2 𝑆𝑂4 solution having molarity 1 𝑀 and density 1.5𝑔/𝑚𝐿 is mixed with 400 𝑚𝐿 of water. Calculate
final molarity of 𝐻2 𝑆𝑂4 solution:
A) 4.4 𝑀 B) 0.14 𝑀 C) 0.52 𝑀 D) 0.20 𝑀
Q15. How many millilitres of 0.1 𝑀 𝐻2 𝑆𝑂4 must be added to 50 𝑚𝐿 of 0.1 𝑀 𝑁𝑎𝑂𝐻 to give a solution that has a
concentration of 0.05 𝑀 in 𝐻2 𝑆𝑂4 ?
A) 400 𝑚𝐿 B) 200 𝑚𝐿
C) 100 𝑚𝐿 D) None of these
Q16. Using atomic masses of constituent atoms/elements, find out the lightest molecule?
A) 𝐶6 𝐻5 𝑁𝑂2 B) 𝑁𝑎𝐶𝑙 C) 𝐻3 𝑃𝑂4 D) 𝑁2 𝑂5
Q17. Which of the following species will have the greatest number of molecules?
A) 1 𝑚𝑜𝑙𝑒 of 𝐻2 𝑆
B) 1 𝑚𝑜𝑙𝑒 of 𝐻2 𝑆𝑂4
C) 1 𝑚𝑜𝑙𝑒 of 𝐻2 𝑂
D) All have the same number of molecules

T.P.P.
C1 – Basic Concepts of Chemistry 17

Q18. Naturally occurring Thallium has two stable isotopes 𝑇𝑙-205 and 𝑇𝑙-203, and an average atomic mass of 204.4.
What is the percentage occurrence of 𝑇𝑙- 205?
A) 50% B) 14% C) 70% D) 30%
Q19. Number of atoms present in 1.8 𝑔 𝐻2 𝑂, 1.7 g 𝑁𝐻3 and 1.6 𝑔 𝐶𝐻4 , in the increasing order is :
A) 𝐻2 𝑂 < 𝑁𝐻3 < 𝐶𝐻4 B) 𝐶𝐻4 < 𝑁𝐻3 < 𝐻2 𝑂
C) 𝐶𝐻4 = 𝑁𝐻3 = 𝐻2 𝑂 D) 𝑁𝐻3 < 𝐶𝐻3 < 𝐻2 𝑂
Q20. 𝑁𝑎2 𝑆𝑂3 . 𝑥𝐻2 𝑂 has 50% 𝐻2 𝑂 by mass. Hence, 𝑥 is :
A) 4 B) 6 C) 5 D) 7
Q21. From a glass-tube containing 18 𝑔 of glucose (𝐶6 𝐻12 𝑂6 ), 0.08 𝑚𝑜𝑙 of glucose is taken out. Glucose left in the glass-
tube is:
A) 0.10 𝑔 B) 17.92 𝑔 C) 3.60 𝑔 D) 0.36 𝑔
Q22. A powerful pesticide 𝐷𝐷𝑇 has 14 carbon atoms per molecule. The compound is 48% carbon by mass. The
approximate molar mass of 𝐷𝐷𝑇 is
A) 120 𝑔 B) 80 𝑔 C) 180 𝑔 D) 350 𝑔
Q23. A compound 𝑀. 𝑥𝐻2 𝑂 contains 40 mass per cent of water. If the molar mass of 𝑀 is 189 𝑔 𝑚𝑜𝑙 −1 , the value of 𝑥
will be
A) 3 B) 5 C) 7 D) 9
Q24. What weight of oxygen is needed to completely react with 1 𝑔 of calcium? (𝐶𝑎 = 40 𝑢)
The reaction that takes place is: 𝐶𝑎 + 𝑂2 → 𝐶𝑎𝑂
A) 32 𝑔 B) 1.6 𝑔 C) 20 𝑔 D) 0.4 𝑔
Q25. Iron burns in oxygen according to the reaction 𝐹𝑒 + 𝑂2 → 𝐹𝑒2 𝑂3 . How many moles of 𝑂2 are needed to produce
5 𝑚𝑜𝑙𝑒𝑠 of 𝐹𝑒2 𝑂3 ?
A) 5 B) 7.5 C) 2.5 D) 10
Q26. 6.0 𝑔 of 𝐶 reacts completely with 12 𝑔 of 𝑂2 producing a mixture of 𝐶𝑂(𝑔) and 𝐶𝑂2 (𝑔). The mass percent of
𝐶𝑂(𝑔) in the gaseous mixture will be about
A) 48% B) 39% C) 56% D) 76%
Q27. 49 𝑔 of sulphuric acid reacts with an element 𝑋 to liberate hydrogen gas. How much hydrogen is liberated?
𝑋 + 𝐻2 𝑆𝑂4 → 𝑋2 (𝑆𝑂4 )3 + 𝐻2
A) 0.5 𝑔 B) 1 𝑔 C) 2 𝑔 D) 4 𝑔
Q28. If 𝑎, 𝑏, 𝑐, 𝑑, 𝑒 are stoichiometric coefficients in the balanced reaction below, give their values in order
𝑎𝑋𝑌 + 𝑏𝑍𝑋2 → 𝑐𝑋3 𝑌𝑍4 + 𝑑𝑋 + 𝑒𝑌
A) 2, 4, 1, 7, 1 B) 3, 20, 5, 2, 5 C) 1, 3, 1, 5, 2 D) 7, 1, 1, 4, 2
Q29. For the reaction 𝐴 + 2𝐵 → 𝐶, 5 𝑚𝑜𝑙𝑒𝑠 of 𝐴 and 8 𝑚𝑜𝑙𝑒𝑠 of 𝐵 will produce
A) 5 𝑚𝑜𝑙𝑒𝑠 of 𝐶 B) 4 𝑚𝑜𝑙𝑒𝑠 of 𝐶
C) 8 𝑚𝑜𝑙𝑒𝑠 of 𝐶 D) 13 𝑚𝑜𝑙𝑒𝑠 of 𝐶
Q30. What mass of 𝐶𝑂2 could be formed by the reaction of 16 𝑔 𝐶𝐻4 with 32 𝑔 of 𝑂2 ?
𝐶𝐻4 + 2𝑂2 → 𝐶𝑂2 + 2𝐻2 𝑂
A) 44 𝑔 B) 33 𝑔 C) 16 𝑔 D) 22 𝑔
C1 – Basic Concepts of Chemistry 18

TEST PRACTICE PROBLEMS – 2


No. of questions: 30 Total time: 90 Minutes Time per question: 3 Minutes

Q31. For the formation of 5.00 𝑚𝑜𝑙 of water, which reaction uses the most nitric acid?
A) 3𝐶𝑢 + 8𝐻𝑁𝑂3 → 3𝐶𝑢(𝑁𝑂3 )2 + 2𝑁𝑂 + 4𝐻2 𝑂
B) 𝐴𝐼2 𝑂3 + 6𝐻𝑁𝑂3 → 2𝐴𝐼(𝑁𝑂3 )3 + 3𝐻2 𝑂
C) 4𝑍𝑛 + 10𝐻𝑁𝑂3 → 4𝑍𝑛(𝑁𝑂3 )2 + 𝑁𝐻4 𝑁𝑂3 + 3𝐻2 𝑂
D) 𝐶𝑢 + 4𝐻𝑁𝑂3 → 𝐶𝑢(𝑁𝑂3 )2 + 2𝑁𝑂2 + 2𝐻2 𝑂
Q32. In the following reaction,
2𝐻2 𝑆 + 𝑆𝑂2 → 2𝐻2 𝑂 + 3𝑆
1 𝑚𝑜𝑙 each of 𝑆𝑂2 and 𝐻2 𝑆 will give sulphur:
A) 1 𝑚𝑜𝑙 B) 3 𝑚𝑜𝑙 C) 1.5 𝑚𝑜𝑙 D) 2 𝑚𝑜𝑙
Q33. You have a solution that is 1.5 𝑀 𝐻2 𝑆𝑂4 (sulfuric acid). How many milliliters of this acid do you need, to prepare
a 100 𝑚𝐿 of 0.15 𝑀 𝐻2 𝑆𝑂4 solution?
A) 15 𝑚𝐿 B) 5 𝑚𝐿
C) 10 𝑚𝐿 D) Insufficient data
Q34. Which of the following means of expressing concentration remains independent of temperature?
A) Molarity B) All are dependent on temperature
C) Weight/Volume percentage D) Mole fraction
Q35. 480 𝑚𝐿 of solution ′1′ (2𝑀) is mixed with 520 𝑚𝐿 of solution ′2′ (1𝑀) resulting in solution 3 . What is the
molarity of solution ′3′? None of the solutions are electrolytes.
A) 1.48 𝑀 B) 1.344 𝑀 C) 1.58 𝑀 D) Can’t say
Q36. What is the mole fraction of methyl alcohol in a 10 𝑚𝑜𝑙𝑎𝑙 aqueous solution of methyl alcohol (𝐶𝐻3 𝑂𝐻) ?
A) 0.76 B) 0.27 C) 0.15 D) 0.09
Q37. What is the molarity of a solution of acetic acid (𝐶𝐻3 𝐶𝑂𝑂𝐻) whose molality is 2.3 𝑚𝑜𝑙/𝑘𝑔.
A) 2.2 B) 2.1
C) 2.4 D) Insufficient data
Q38. 10 𝑔 of 𝐶𝑎𝐶𝑂3 contains
A) 10 𝑚𝑜𝑙𝑒𝑠 of 𝐶𝑎𝐶𝑂3 B) 0.1 𝑚𝑜𝑙 of 𝐶𝑎
C) 6.022 × 1023 atoms of 𝐶𝑎 D) 0.1 gram of 𝐶𝑎
Q39. Two glucose solutions are mixed. One has a volume of 480 𝑚𝐿 and a concentration of 1.50 𝑀 and the second has
a volume of 520 𝑚𝐿 and concentration 1.20 𝑀. The molarity of final solution is
A) 1.20 𝑀 B) 1.50 𝑀 C) 1.344 𝑀 D) 2.70 𝑀
Q40. How many moles of ferric alum (𝑁𝐻4 )2 𝑆𝑂4 𝐹𝑒2 (𝑆𝑂4 )3 ∙ 24𝐻2 𝑂 can be made from the sample of 𝐹𝑒 containing
0.0056 𝑔 of it?
A) 10−4 𝑚𝑜𝑙 B) 0.5 × 10−4 𝑚𝑜𝑙
C) 0.33 × 10−4 𝑚𝑜𝑙 D) 2 × 10−4 𝑚𝑜𝑙
Q41. Suppose elements 𝑋 and 𝑌 combine to form two compounds 𝑋𝑌2 and 𝑋3 𝑌2 when 0.1 𝑚𝑜𝑙 of former weighs 10 𝑔
while 0.05 𝑚𝑜𝑙𝑒 of the latter weigh 9 𝑔. What are the atomic weights of 𝑋 and 𝑌?
A) 40,30 B) 60,40 C) 20,30 D) 30,20

T.P.P.
C1 – Basic Concepts of Chemistry 19

Q42. In an experiment 6.67 𝑔 of 𝐴𝑙𝐶𝑙3 was produced and 0.54 𝑔 𝐴𝑙 remained unreacted. How many moles of 𝐴𝑙 and
chlorine atoms were taken originally (𝐴𝑙 = 27 𝑢, 𝐶𝑙 = 35.5 𝑢)?
A) 0.07, 0.15 B) 0.07, 0.05 C) 0.02, 0.05 D) 0.02, 0.15
Q43. A gaseous mixture contains 𝑂2 and 𝑁2 in the ratio of 1: 4 by weight. The ratio of their molecules is
A) 1: 4 B) 1: 8 C) 7: 32 D) 3: 16
Q44. Upon mixing 50.0 𝑚𝐿 of 0.1 𝑀 lead nitrate solution with 50 𝑚𝐿 of 0.05 𝑀 chromic sulphate solution,
precipitation of lead sulphate solution takes place. How many moles of lead sulphate are formed and what is the
molar concentration (in 𝑀) of chromic sulphate left in the solution?
A) 0.005, 0.0084 B) 0.0084, 0.005 C) 0.005, 0.00084 D) 0.05, 0.00084
Q45. 13.4 𝑔 of a sample of unstable hydrated salt 𝑁𝑎2 𝑆𝑂4 ∙ 𝑥𝐻2 𝑂 was found to contain 6.3 𝑔 of 𝐻2 𝑂.The number of
molecules of water of crystallisation is
A) 5 B) 7 C) 2 D) 10
Q46. One litre of 0.15 𝑀 𝐻𝐶𝑙 and one litre of 0.3 𝑀 𝐻𝐶𝑙 is given. What is the maximum volume of 0.2 𝑀 𝐻𝐶𝑙 which one
can make from these two solutions? No water is added.
A) 1.2 𝐿 B) 1.5 𝐿 C) 1.3 𝐿 D) 1.4 𝐿
Q47. A certain compound has the molecular formula 𝑋4 𝑂6 . If 10 𝑔 of 𝑋4 𝑂6 has 5.72 𝑔 𝑋, the atomic mass of 𝑋 is
A) 32 𝑎𝑚𝑢 B) 37 𝑎𝑚𝑢 C) 42 𝑎𝑚𝑢 D) 98 𝑎𝑚𝑢
Q48. The molarity of 𝐻2 𝑆𝑂4 is 18 𝑀. Its density is 1.8 𝑔 𝑚𝐿−1 . Hence molality is:
A) 36 B) 200 C) 500 D) 18
Q49. How many grams of phosphoric acid would be needed to neutralise 100 𝑔 of magnesium hydroxide? (The
molecular weights are: 𝐻3 𝑃𝑂4 = 98 and 𝑀𝑔(𝑂𝐻)2 = 58.3)
A) 66.7 𝑔 B) 252 𝑔 C) 112 𝑔 D) 168 𝑔
Q50. The simplest formula of a compound containing 50% of an element 𝑋 (atomic weight 10) and 50% of element 𝑌
(atomic weight 20) is:
A) 𝑋𝑌 B) 𝑋2 𝑌 C) 𝑋𝑌2 D) 𝑋2 𝑌3
Q51. When 10 𝑚𝐿 of ethyl alcohol (density = 0.7893 𝑔 𝑚𝐿−1 ) is mixed with 20 𝑚𝐿 of water (density 0.9971 𝑔 𝑚𝐿−1 )
at 25°𝐶, the final solution has a density of 0.9571 𝑔 𝑚𝐿−1 . The percentage change in total volume on mixing is
A) 3.1% B) 2.4%
C) 1% D) None of these
Q52. The molality of 1 𝐿 solution with 𝑥% 𝑤/𝑣 𝐻2 𝑆𝑂4 is equal to 9. The weight of the solvent present in the solution is
910 𝑔. The value of 𝑥 is:
A) 90 B) 80.3 C) 40.13 D) 9
Q53. The density of 1 𝑀 solution of 𝑁𝑎𝐶𝑙 is 1.0585 𝑔 𝑚𝐿−1 . The molality of the solution is
A) 1.0585 B) 1.00 C) 0.10 D) 0.0585
Q54. 100 𝑚𝐿 of mixture 𝑁𝑎𝑂𝐻 and 𝑁𝑎2 𝑆𝑂4 is neutralised by 10 𝑚𝐿 of 0.5 𝑀 𝐻2 𝑆𝑂4 . Hence, 𝑁𝑎𝑂𝐻 in 100 𝑚𝐿 solution
is
A) 0.2 𝑔 B) 0.4 𝑔 C) 0.6 𝑔 D) None
C1 – Basic Concepts of Chemistry 20

Q55. A hydrate of 𝑁𝑎2 𝑆𝑂3 has 50% water by mass. It is


A) 𝑁𝑎2 𝑆𝑂3 ∙ 5𝐻2 𝑂 B) 𝑁𝑎2 𝑆𝑂3 ∙ 6𝐻2 𝑂
C) 𝑁𝑎2 𝑆𝑂3 ∙ 7𝐻2 𝑂 D) 𝑁𝑎2 𝑆𝑂3 ∙ 2𝐻2 𝑂
Q56. Mole fraction of ethanol in ethanol water mixture is 0.25. Hence, the percentage concentration of ethanol by
weight of mixture is
A) 25% B) 75% C) 46% D) 54%
Q57. How many moles of electrons weigh one kilogram?
1
A) 6.023 × 1023 B) × 1031
9.108
6.023 1
C) × 1054 D) × 108
9.108 9.108×6.023

Q58. The weight of 1 × 1022 molecules of 𝐶𝑢𝑆𝑂4 ∙ 5𝐻2 𝑂 is (Atomic weight of 𝐶𝑢 = 63.5)
A) 4.14 𝑔 B) 5.14 𝑔 C) 6.14 𝑔 D) 7.14 𝑔
Q59. How many moles of 𝑂2 will be liberated by one mole of 𝐶𝑟𝑂3 in the following reaction:
𝐶𝑟𝑂3 + 𝐻2 𝑆𝑂4 → 𝐶𝑟2 (𝑆𝑂4 )3 + 𝐻2 𝑂 + 𝑂2
A) 4.5 B) 2.5 C) 1.25 D) None
Q60. The expression relating mole fraction of solute (𝑥2 ) and molarity (𝑀) of the solution is: (where 𝑑 is the density
of the solution in 𝑔 𝐿−1 and 𝑀𝑤1 and 𝑀𝑤2 are the molar masses of solvent and solute, respectively
𝑀×𝑀𝑤1 𝑀×𝑀𝑤1
A) 𝑥2 = B) 𝑥2 =
𝑀(𝑀𝑤1 −𝑀𝑤2 )+1000𝑑 𝑀(𝑀𝑤1 −𝑀𝑤2 )+𝑑
𝑀×𝑀𝑤1 𝑀×𝑀𝑤1
C) 𝑥2 = D) 𝑥2 =
𝑀(𝑀𝑤1 −𝑀𝑤2 )−1000𝑑 𝑀(𝑀𝑤1 −𝑀𝑤2 )−𝑑

TEST PRACTICE PROBLEMS – 3


No. of questions: 25 Total time: 75 Minutes Time per question: 3 Minutes

Q61. Consider the ionisation of 𝐻2 𝑆𝑂4 as follows:


𝐻2 𝑆𝑂4 + 2𝐻2 𝑂 → 2𝐻3 𝑂⊕ + 𝑆𝑂42−
The total number of ions furnished by 100 𝑚𝐿 of 0.1 𝑀 𝐻2 𝑆𝑂4 will be
A) 1.2 × 1023 B) 0.12 × 1023 C) 0.18 × 1023 D) 1.8 × 1023
Q62. Calculate the number of oxygen atoms required to combine with 7.0 𝑔 of 𝑁2 to form 𝑁2 𝑂3 if 80% by weight of 𝑁2
is converted into products.
3
𝑁2 + 𝑂2 → 𝑁2 𝑂3
2
A) 3.24 × 1023 B) 3.6 × 1023 C) 18 × 1023 D) 6.02 × 1023
Q63. 36.5% (𝑤⁄𝑤)𝐻𝐶𝑙 has density equal to 1.20 𝑔 𝑚𝐿−1 . The molarity (𝑀) and molality (𝑚), respectively, are
A) 15.7, 15.7 B) 12, 12 C) 15.7, 12 D) 12, 15.7
Q64. 10 𝑚𝐿 of 1 𝑀 𝐵𝑎𝐶𝑙2 solution and 5 𝑚𝐿 0.5 𝑀 𝐾2 𝑆𝑂4 are mixed together to precipitate out 𝐵𝑎𝑆𝑂4 . The amount of
𝐵𝑎𝑆𝑂4 precipitated will be
A) 0.005 𝑚𝑜𝑙 B) 0.00025 𝑚𝑜𝑙 C) 0.025 𝑚𝑜𝑙 D) 0.0025 𝑚𝑜𝑙
Q65. Mole fraction of a solute in an aqueous solution is 0.2. The molality of the solution will be
A) 13.88 B) 1.388 C) 0.138 D) 0.0138

T.P.P.
C1 – Basic Concepts of Chemistry 21

Q66. Two samples of 𝐻𝐶𝑙 of 1.0 𝑀 and 0.25 𝑀 are mixed. Find the volumes of these samples taken in order to prepare
0.75 𝑀 𝐻𝐶𝑙 solution. Assume no water is added
I. 20 𝑚𝐿, 10 𝑚𝐿 II. 100 𝑚𝐿, 50 𝑚𝐿
III. 40 𝑚𝐿, 20 𝑚𝐿 IV. 50 𝑚𝐿, 25 𝑚𝐿
A) I, II, IV B) I, II C) II. III, IV D) I, II, III, IV
Q67. If 100 𝑚𝐿 of 𝐻2 𝑆𝑂4 and 100 𝑚𝐿 of 𝐻2 𝑂 are mixed, the mass percent of 𝐻2 𝑆𝑂4 in the resulting solution is
(𝑑𝐻2𝑆𝑂4 = 0.9 𝑔 𝑚𝐿−1 , 𝑑𝐻2𝑂 = 1.0 𝑔 𝑚𝐿−1 )
A) 90 B) 47.36 C) 50 D) 60
Q68. The largest number of molecules is in
A) 36 𝑔 of water B) 28 𝑔 of carbon monoxide
C) 46 𝑔 of ethyl alcohol D) 54 𝑔 of nitrogen pentoxide
Q69. A molal solution is one that contains 1 𝑚𝑜𝑙 of a solute in
A) 1000 𝑔 of solvent B) 1𝐿 of solvent
C) 1 𝐿 of solution D) 22.4 𝐿 of solution.
Q70. In which mode of expression, the concentration of a solution remains independent of temperature?
A) Molarity B) Normality C) Formality D) Molality
Q71. Which of the following has the maximum number of atoms?
A) 24 𝑔 of 𝐶(12) B) 56 𝑔 of 𝐹𝑒(56)
C) 27 of 𝐴𝑙(27) D) 108 𝑔 of 𝐴𝑔(108)
Q72. Given that the abundance of isotopes 54𝐹 𝑒, 56𝐹 𝑒, and 57𝐹 𝑒 is 5%, 90%, and 5%, respectively. The atomic mass of
𝐹𝑒 is
A) 55.85 B) 55.95 C) 55.75 D) 55.05
Q73. Dissolving 120 𝑔 of urea (𝑀𝑤 = 60) in 1000 𝑔 of water gave a solution of density 1.15 𝑔 𝑚𝐿−1 . The molarity of
solution is:
A) 1.78 𝑀 B) 2.00 𝑀 C) 2.05 𝑀 D) 2.22 𝑀
Q74. The molarity of a solution obtained by mixing 750 𝑚𝐿 of 0.5 𝑀 𝐻𝐶𝑙 with 250 𝑚𝐿 of 2𝑀 𝐻𝐶𝑙 will be:
A) 1.00 𝑀 B) 1.75 𝑀 C) 0.975 𝑀 D) 0.875 𝑀
Q75. A sample of 𝐶𝑎𝐶𝑂3 has 𝐶𝑎 = 40%, 𝐶 = 12% and 𝑂 = 48%. If the law of constant proportion is true then the
weight of calcium in 5 𝑔 of a sample of 𝐶𝑎𝐶𝑂3 from another source will be
A) 0.20 𝑔 B) 2.0 𝑔 C) 2.5 𝑔 D) 4.0 𝑔
Q76. The ratio of mass of 1 𝑚𝑜𝑙𝑒 of sodium and 1023 atoms of sodium is:
23
A) 6.022 B) 23 C) D) 23 × 6.022
6.022

Q77. The number of moles in 0.64 𝑔 of 𝑆𝑂2 is:


A) 100 B) 10 C) 0.1 D) 0.01
Q78. The number of 𝑂3 molecules in 16 𝑔 of ozone is approximately
A) 2 × 1023 B) 3 × 1023 C) 4 × 1023 D) 6 × 1023
Q79. The total number of atoms present in 0.1 𝑚𝑜𝑙𝑒 of sucrose (𝐶12 𝐻22 𝑂11 ) is:
A) 6.02 × 1022 B) 2.7 × 1024 C) 6.02 × 1024 D) 2.7 × 1025
C1 – Basic Concepts of Chemistry 22

Q80. Which of the following has maximum number of molecules?


A) 1 𝑔 of 𝐶𝑂2 B) 1 𝑔 of 𝑁2 C) 1 𝑔 of 𝐻2 D) 1 𝑔 of 𝐶𝐻4
Q81. The number of atoms present in 1 𝑔 of hydrogen gas is the same as are present in
A) 0.4 𝑔 of 𝐻𝑒 B) 22 𝑔 of 𝐶𝑂2 C) 6 𝑔 of 𝐻2 𝑂 D) 24 𝑔 of 𝐶
Q82. One gram is more than
A) 0.1 𝑚𝑜𝑙 of 𝐶𝑂2 B) mass of 6.02 × 1022 molecules or water
C) mass of 2.27 𝐿 of hydrogen gas at 𝑆. 𝑇. 𝑃. D) 0.1 𝑚𝑜𝑙 of potassium
Q83. On analysis, a compound was found to contain iodine and oxygen in the ratio of 254 𝑔 iodine and 80 𝑔 of oxygen.
Which of the following is the formula of the compound (atomic masses: iodine = 127, oxygen = 16)?
A) 𝐼𝑂 B) 𝐼2 𝑂 C) 𝐼5 𝑂2 D) 𝐼2 𝑂5
Q84. A phosphorus oxide has 43.6% phosphorus (atomic mass = 31). The empirical formula of the compound is;
A) 𝑃2 𝑂5 B) 𝑃2 𝑂3 C) 𝑃4 𝑂6 D) 𝑃𝑂2
Q85. The simple formula of a compound containing 32.5% 𝐾, 0.839% 𝐻, 26.7% 𝑆 and 39.9% 𝑂 is (atomic mass of 𝐾 =
39)
A) 𝐾𝐻𝑆𝑂4 B) 𝐾𝐻𝑆𝑂3 C) 𝐾2 𝑆𝑂4 ∙ 2𝐻2 𝑂 D) 𝐾𝐻𝑆𝑂2

DATA Guide
A # of questions Total problems in TPP
B # Attempts Total attempts in OMR
C # Correct Total questions correct
D # Incorrect Out of the ones marked in OMR
E # Unattempted 𝐴 − 𝐵
ANALYSIS
F Percentage attempts 𝐵
× 100
𝐴
G Percentage Accuracy 𝐶
× 100
𝐵

Question type # Correct (C) # Incorrect (I) # Unattempted (U)


Easy
Medium
Hard

Tip: To begin with, your accuracy must be high, typically > 60%. Percentage attempts should be > 50%
As time progresses, your percentage attempts should increase without a reduction in accuracy.
Additionally, you should be able to get > 80% Level 1 questions correct, as they involve basic recall of the concepts and
formulae of the chapter.

T.P.P.
C1 – Basic Concepts of Chemistry 23

Answer Key
C1.1 MOLE CONCEPT
PRE-TEST Q4. One-twelfth (1/12𝑡ℎ)
Q1. 84 oranges IN-CLASS EXERCISE
Q2. 7 boxes
LEVEL 1
Q3. 2880 articles
Q1. I. 16 𝑎𝑚𝑢
Q4. 210 almonds and cashew nuts
II. 18 𝑎𝑚𝑢
Q5. 325 girls III. 44 𝑎𝑚𝑢
IV. 36.5 𝑎𝑚𝑢
PRE READING EXERCISE
Q2. 12.8 𝑎𝑚𝑢
Q1. Homogeneous Q3. I. 3𝑁𝐴 atoms of 𝐶
Q2. Compound II. 33 𝑁𝐴 atoms
Q3. Physical

LEVEL 2

Number of molecules Number of moles of 𝐻 Number of 𝐻 atoms in


Q No. Compound Number of Moles
of the compound atoms in the compound the compound

Q4. 𝐻2 𝑂 5 𝑚𝑜𝑙𝑒𝑠 5 𝑁𝐴 2 × 5 = 10 10 𝑁𝐴

Q5. 𝐻2 𝑆 2 𝑚𝑜𝑙𝑒𝑠 2 𝑁𝐴 2×2=4 4 𝑁𝐴

Q6. 𝐻𝐶𝑙 3 𝑚𝑜𝑙𝑒𝑠 3 𝑁𝐴 1× 3=3 3 𝑁𝐴

Q7. 𝐻2 𝑆𝑂4 3 𝑚𝑜𝑙𝑒𝑠 3 𝑁𝐴 2×3= 6 6𝑁𝐴

Q8. I. 2 𝑚𝑜𝑙 of 𝐻 atoms IV. 72.26 × 1023


II. 2 𝑚𝑜𝑙 of 𝐻 atoms V. 3.011 × 1023
III. 2 𝑚𝑜𝑙 of 𝐻 atoms VI. 3 𝑚𝑜𝑙𝑒𝑠
IV. 1 𝑚𝑜𝑙 of 𝐻 atoms Q2. The element 𝑌 can be oxygen.
Therefore 1 𝑚𝑜𝑙𝑒 of 𝐻𝐶𝑙 has the least number of
Q3. 6.925 𝑎𝑚𝑢
hydrogen atoms
Q4. 63.52 𝑢
Q9. 𝐹𝑒 − 54 is 37.5% and 𝐹𝑒 − 58 is 12.5 %
Q5. I. 20 𝑚𝑜𝑙 of 𝐶 atoms.
abundant in nature.
II. 50 𝑚𝑜𝑙 of 𝐻 atoms.
III. 5 × 𝑁𝐴 molecules of 𝐶4 𝐻10 .
HOMEWORK
IV. 20 × 𝑁𝐴 atoms of carbon.
LEVEL 1 V. 50 × 𝑁𝐴 atoms of hydrogen.
Q1. I. 16 atoms of oxygen
LEVEL 2
II. 18.06 × 1023
III. 2 𝑚𝑜𝑙 Q6. 0.08 𝑁𝐴 atoms of 𝐶3 𝐻6 are present in container
C1 – Basic Concepts of Chemistry 24

Number of Number of moles of Number of 𝐻 Number of 𝐶


Number of
Q. No. Compound molecules of the 𝐻 atoms in the atoms in the atoms in the
moles
compound compound compound compound

Q7. 𝐶3 𝐻8 𝑂 2 𝑚𝑜𝑙𝑒𝑠 2 𝑁𝐴 2×8 2 × 8 × 𝑁𝐴 2 × 3 × 𝑁𝐴

Q8. 𝐶𝐻4 3 𝑚𝑜𝑙𝑒𝑠 3 𝑁𝐴 3×4 3 × 4 × 𝑁𝐴 3 × 1 × 𝑁𝐴

Q9. 𝐶6 𝐻6 6 𝑚𝑜𝑙𝑒𝑠 6 𝑁𝐴 6×6 6 × 6 × 𝑁𝐴 6 × 6 × 𝑁𝐴

Q10. 𝐶2 𝐻6 𝑂 1 𝑚𝑜𝑙𝑒 𝑁𝐴 1×6 1 × 6 𝑁𝐴 1 × 2 × 𝑁𝐴

Q11. 𝐶2 𝐻8 2.5 𝑚𝑜𝑙𝑒𝑠 2.5 𝑁𝐴 8 × 2.5 8 × 2.5 × 𝑁𝐴 5 × 𝑁𝐴

Q12. The third isotope is 𝑀𝑔-23.

C1.2 RELATING MASS AND MOLES


PRE-READING EXERCISE Q10. 𝑌 atoms
Q1. Molar mass Q11. 12 𝑔 𝐶2 𝐻6 𝑂 contains more number of 𝐶 atoms.
Q2. 34 𝑔 HOMEWORK
Q3. 4
12
LEVEL 1
Q4. C
Q5. 6.022 × 1023 Q1. I. 2.5 𝑚𝑜𝑙
II. 0.5 𝑚𝑜𝑙
Q6. Empirical
Q2. I. 400 𝑔
IN-CLASS EXERCISE II. 4.5 𝑔
LEVEL 1 III. 176 𝑔
IV. 138.5 𝑔
Q1. I. 0.33 𝑚𝑜𝑙
II. 3 𝑚𝑜𝑙 Q3. 7.8 × 1022 atoms of 𝑁𝑎

Q2. Oxygen has both highest number of molecules Q4. 2.25 𝑚𝑜𝑙𝑒𝑠
and moles. Q5. % by 𝑚𝑎𝑠𝑠 of 𝑁𝑎 = 39.3%
Q3. 36 𝑔 % 𝑚𝑎𝑠𝑠 of 𝐶𝑙 = 60.7%

Q4. Mass percent composition of 𝐶𝑎 = 40% Q6. Mass percentage of 𝐻 = 77%


Mass percent composition of 𝐶 = 12% Mass percentage of 𝐻𝑒 = 23%
Mass percent composition of 𝑂 = 48%
LEVEL 2
LEVEL 2
No. of atoms of 𝐾 = 6.022 × 1023
Q5. Molecular formula =empirical formula= 𝑆𝑂2 Q7. No. of atoms of 𝐶𝑙 = 6.022 × 1023 }
No. of atoms of 𝑂 = 18.066 × 1023
Q6. The molecular formula is 𝐶28 𝐻64 . The empirical
formula is 𝐶7 𝐻16 . Q8. Mass of 1 𝑚𝑜𝑙 of 𝑋 = 24𝑔

Q7. Sample 𝐵 is the heaviest. Q9. 1000 𝑚𝑜𝑙

Q8. 2 × 10−3 𝑚𝑜𝑙𝑒𝑠 of 𝐻2 𝑆𝑂4 are left. Q10. No. of moles of 𝐶𝑂2 left = 0.0033

Q9. The metal is potassium. Q11. Volume occupied by 1 𝑎𝑡𝑜𝑚 = 1.141 × 10−23 𝑐𝑐

Ans.
C1 – Basic Concepts of Chemistry 25

Q12. I. Mass of a molecule is 32 𝑎𝑚𝑢 . (Changing the molecular mass of oxygen, it only changes
definition of a mole does not change the the molar mass)
II. 52.31 × 10−3 𝑘𝑔

C1.3 STOICHIOMETRIC CALCULATIONS


PRE READING EXERCISE Q6. Loss in weight of sample is 192 𝑔𝑟𝑎𝑚𝑠
Q1. 2 Q7. 0.1 𝑚𝑜𝑙𝑒 of 𝐼𝐶𝑙 and 0.1 𝑚𝑜𝑙𝑒 of 𝐼𝐶𝑙3 formed.
So ratio of amount of products formed = 1: 1.
Q2. 2, 2
LEVEL 3
Q3. I. 2𝐹𝑒 + 3𝐻2 𝑆04 → 𝐹𝑒2 (𝑆𝑂4 )3 + 3𝐻2
II. 2𝐶2 𝐻6 + 7𝑂2 → 6𝐻2 𝑂 + 4𝐶𝑂2 Q8. Percentage of 𝐵𝑎𝑂 in mixture = 37.5%
III. 3𝐾𝑂𝐻 + 𝐻3 𝑃𝑂4 → 𝐾3 𝑃𝑂4 + 3𝐻2 𝑂
HOMEWORK
IV. 𝑆𝑛𝑂2 + 2𝐻2 → 𝑆𝑛 + 2𝐻2 𝑂
LEVEL 1
IN-CLASS EXERCISE
Q1. 𝐶2 𝐻4 + 3𝑂2 → 2𝐶𝑂2 + 2𝐻2 𝑂
LEVEL 1
Q2. I. 16.25 𝑚𝑜𝑙𝑒𝑠 of 𝑂2
11
Q1. I. 2𝐹𝑒𝑆2 + 𝑂2 → 𝐹𝑒2 𝑂3 + 4𝑆𝑂2 II. 2.4 𝑚𝑜𝑙𝑒𝑠 of 𝐶𝑂2 will be formed.
2
13
II. 2𝐶2 𝐻5 + 𝑂2 → 4𝐶𝑂2 + 5𝐻2 𝑂 LEVEL 2
2

Q2. I. 16 𝑚𝑜𝑙 of 𝑋 will react with 32 𝑚𝑜𝑙 of 𝑌 for Q3. 6.9 𝑔 of 𝑁𝑂2
completion of a reaction.
Q4. 116 𝑔 will be the weight of the residue.
II. 16 𝑚𝑜𝑙 of 𝑋 will react completely with
8 𝑚𝑜𝑙 of 𝑌. Q5. 8 𝑔𝑟𝑎𝑚𝑠

Q3. I. 10 𝑚𝑜𝑙 of hydrogen will react completely Q6. 1 𝑔 𝑁𝑂 is formed


with 1.66 𝑚𝑜𝑙 of boron Q7. 795 𝑘𝑔
II. 2 𝑚𝑜𝑙 of boron will give 4 𝑚𝑜𝑙 of diborane. Q8. % composition of 𝐶𝑎𝐶𝑂3 in mixture = 37%
III. 7 𝑚𝑜𝑙 of diborane will require 21 𝑚𝑜𝑙 of % Composition of 𝐶𝑎𝑂 in mixture = 63%
hydrogen (𝐻2 ) Q9. 9.03 × 1023 atoms of 𝑂2 are required to form
LEVEL 2 18 𝑔𝑟𝑎𝑚 of ice.

Q4. mass of 𝐶𝑙2 liberated from 64 𝑔𝑟𝑎𝑚𝑠 of 𝐴𝑢𝐶𝑙3 LEVEL 3


⇒ 21.3 𝑔𝑟𝑎𝑚𝑠
Q10. Percentage by mass = 97%
Q5. 53 𝑔 of 𝑁𝑎2 𝐶𝑂3 and 9 𝑔 of 𝐻2 𝑂 is required.
Q11. Wt. of 𝐶𝑂 = 3 𝑔.

C1.4 REACTIONS INVOLVING LIMITING REAGENT


PRE READING EXERCISE IN CLASS EXERCISE
LEVEL 1
Q1. B
Q2. False Q1. I. Non – stoichiometric
II. Stoichiometric
Q3. I. 3
1
III. Non – stoichiometric
II.
3
Q2. I. 𝐴 is the excess reagent and 𝐵 is limiting
III. 4, 6
reagent.
C1 – Basic Concepts of Chemistry 26

II. 𝐵 is the excess reagent and 𝐴 is limiting LEVEL 2


reagent.
Q4. Amount of 𝐴𝑙 left in the mixture = 54 𝑔
III. 𝐴 is the excess reagent and 𝐵 is limiting
Number of moles of 𝐹𝑒 formed = 2 𝑚𝑜𝑙𝑒𝑠
reagent.
Q5. 16 𝑔𝑟𝑎𝑚𝑠 of 𝑂2 .
Q3. I. 𝐻𝐶𝑙 would be left in the mixture after
reacting. Q6. % Yield = 81%
II. None of the reactants would be left in the Q7. 30.4 𝑔 of methyl salicylate
mixture after reacting
LEVEL 3
Q8. mass of sample = 612.5 𝑔𝑟𝑎𝑚𝑠

HOMEWORK
LEVEL 1

Reaction Moles of 𝐴 Moles of 𝐵 Limiting reagent Moles of Product formed

Q1. 4𝐴 + 2𝐵2 → 3𝐶 2 3 𝐴 1.5

Q2. 𝐴 + 𝐵2 → 𝐶 4 2 𝐵2 2

Q3. 𝐴 + 4𝐵2 → 𝐶 5 4 𝐵2 1

Q4. 2𝐴 + 𝐵2 → 2𝐴𝐵 10 20 𝐴 10

Q5. I. Non-stoichiometric III. Additional 𝐻2 required = 2 𝑔


II. Non-stoichiometric
Q8. We will obtain 2. 7𝑔 of 𝐻𝐶𝑁
LEVEL 2 Q9. 3.75 𝑚𝑜𝑙𝑒𝑠 of 𝑁𝑎𝐶𝑙 will be formed.
Q6. I. 𝐵2 is the limiting reagent, 1 𝑚𝑜𝑙 products Q10. I. Thus, 𝑁𝐻3 is present in limiting quantity and
are formed 𝐶𝑂2 is present in excess.
II. 𝐴 is the limiting reagent, 5 𝑚𝑜𝑙 products are II. 1200 𝑔
formed III. 𝐶𝑂2 left = 220 𝑔
III. They react completely, 10 𝑚𝑜𝑙 products are Q11. % yield= 73%
formed
LEVEL 3
Q7. I. 56.67 𝑔 𝑁𝐻3
II. Amount of 𝑁2 left over = 9.24 𝑔 Q12. Ratio of masses (𝑁𝑎𝐶𝑙: 𝑁𝑎𝐵𝑟) = 2: 3

C1.5 CONCENTRATION
PRE READING EXERCISE IN-CLASS EXERCISE
Q1. Solute LEVEL 1
Q2. 2 Q1. I. 0.4 M
Q3. 0.5 𝑚 II. 0.66 𝑚
III. 20 𝑔/𝐿
Q4. True
Q2. 0.65
Q5. 2𝑀
Q3. 34.2 𝑔

Ans.
C1 – Basic Concepts of Chemistry 27

LEVEL 2 Q3. 0.1 𝑚𝑜𝑙


Q4. I. Molarity of solution = 5 𝑀 Q4. 1𝑚
II. Molality = 0.2 𝑚 Q5. 0.2 𝐿
Q5. Molarity of the new solution is 2.33 𝑀.
LEVEL 2
Q6. 0.1𝑀
Q6. 273.75 𝑔 𝐻𝐶𝑙
Q7. 1.29 𝑔/𝑐𝑐
Q7. I. 10 𝑚
LEVEL 3 II. 4 𝑚
Q8. 9𝑀 Q8. We need 50 𝑚𝐿 of 2𝑀 𝐻2 𝑆𝑂4 solution.
Q9. Molarity of resulting solution = 0.31𝑀
HOMEWORK
LEVEL 1 LEVEL 3

Q1. 0.66 𝑚 Q10. Amount of 𝐻2 𝑆𝑂4 left unreacted = 49 𝑔

Q2. 2𝑀 Q11. Amount of 𝑁𝑎𝑁𝑂3 that can be formed = 170 𝑚𝑔

ANSWERS TO TEST PRACTICE PROBLEMS

Mark (C) / (I) / (U) Mark (C) / (I) / (U)


Q. No. Ans. Level Q. No. Ans. Level
as appropriate as appropriate
Q1. B Easy Q18. C Medium
Q2. D Easy Q19. A Medium
Q3. A Medium Q20. D Medium
Q4. D Medium Q21. C Easy
Q5. B Easy Q22. D Medium
Q6. D Medium Q23. C Hard

Q7. A Medium Q24. D Medium


Q8. C Medium Q25. B Easy
Q9. A Medium Q26. B Hard
Q10. C Medium Q27. B Easy
Q11. A Hard Q28. A Medium
Q12. C Hard Q29. B Easy
Q13. D Easy Q30. A Medium
Q14. D Medium Q31. C Medium
Q15. C Hard Q32. C Easy
Q16. B Easy Q33. C Medium
Q17. D Easy Q34. D Easy
C1 – Basic Concepts of Chemistry 28

Q35. A Medium Q61. C Medium


Q36. C Medium Q62. B Medium
Q37. D Medium Q63. D Medium
Q38. B Easy Q64. D Medium
Q39. C Medium Q65. A Medium
Q40. B Medium Q66. D Hard
Q41. A Medium Q67. B Medium
Q42. A Medium Q68. A Medium
Q43. C Easy Q69. A Easy
Q44. A Medium Q70. D Easy
Q45. B Medium Q71. A Easy
Q46. B Hard Q72. B Medium
Q47. A Medium Q73. C Medium
Q48. C Medium Q74. D Medium
Q49. C Medium Q75. B Easy
Q50. B Easy Q76. A Easy
Q51. A Medium Q77. D Easy
Q52. B Hard Q78. A Medium
Q53. B Easy Q79. B Medium
Q54. B Medium Q80. C Easy
Q55. C Easy Q81. C Medium
Q56. C Medium Q82. C Medium
Q57. D Easy Q83. D Medium
Q58. A Easy Q84. A Medium
Q59. D Medium Q85. B Medium
Q60. B Hard

Ans.
C2 – Atomic Structure 29

Fifth Edition

C2. Atomic Structure


TABLE OF CONTENTS

C2. Atomic Structure 29


C2.1 Discovery of Sub – Atomic Particles and Early Atomic Models ……………………….………………………………30
C2.2 Dual Nature of EM Radiation and Photoelectric Effect …………………………………..………………………….……34
C2.3 Atomic Spectra and Bohr’s atomic Model ………………………………………………………………………………………38
C2.4 Quantum Mechanical Model of Atom ……………………………………………………………………………………….……41
C2.5 Writing Electronic Configurations …………………………………………………………………………………………..……44
Additional Reading ………………………………………………………………………………………………………………………………47
Test Practice Problems …………………………………………………………………………………………………………………..……51
Answer Key …………………………………………………………………………………………………………………………………………61
C2 – Atomic Structure 30

C2.1 Discovery of Sub – Atomic Particles and Early Atomic Models


CONCEPTS
1. Sub-atomic particles: Electrons, Protons and Neutrons.
2. Millikan’s oil drop experiment.
3. Atomic Models: Thomson’s Plum Pudding Model and Rutherford’s Planetary Model of Atom.
4. Mass Number and Atomic Number.
5. Isotopes, isobars and isotones.

PRE-READING

Category Book Name Chapter Name Chapter Section

REQUIRED NCERT XI Structure of Atom 2 2.1, 2.2

PRE-READING EXERCISE
Q1. Complete the following table
Sub – atomic particle Charge (positive/negative/neutral)
Electron
Proton
Neutron

Q2. The positive electrode used in the cathode ray experiment is called the __________ (anode/cathode).
Q3. Like charges __________ (attract/repel) and unlike charges __________ (attract/repel) each other.
Q4. The cathode ray experiment was carried out at high pressure and high voltage. True/False?
Q5. Consider the experimental setup for Millikan’s Oil Drop Experiment.
I. A/An __________ was used to produce a fine mist of oil drops.
II. The oil drops were exposed to a/an __________ (electric/magnetic) field.
Q6. When a cathode ray hits a __________ (zinc sulphide/lead sulphide) coating, glow (or scintillation) is observed.

C2.1
C2 – Atomic Structure 31

IN-CLASS EXERCISE

LEVEL 1

Q1. Based on the above experiment to determine the charge to mass ratio of an electron, answer the following:
I. Deflection of a particle under electric or magnetic field confirms the behavior of a/an __________ particle.
II. When both electric field and magnetic field were balanced, electrons strike at
A) Only A B) Only C C) Both A and C D) Only B
III. Fill in the blanks :
a) The cathode rays deviate towards the __________(negative/positive) plate.
b) The cathode rays are made up of __________
c) The charge to mass ratio of an electron was determined to be __________.
IV. What happens if magnetic field is reversed?
Q2. What was the purpose of the 𝑋-ray beam in Millikan’s Oil drop experiment?
Q3. Based on the models given below, answer the following:

I. The above models represent ___________ and __________ model of an atom respectively.
II. Which of the following did Thomson’s atomic model explain correctly?
A) Charge on a proton B) Existence of a dense nucleus
C) Positive charge of a nucleus D) Electrical neutrality of an atom
III. What causes scintillation in circular 𝑍𝑛𝑆 plate in Rutherford’s experiment?
A) Electrons B) Protons C) 𝛼-particles D) Neutrons
IV. Complete the following table on the basis of understanding of Rutherford’s experiment.
Observation in Experiment Conclusion
1. Most of the 𝛼-particles passed undeflected
2. Some of the 𝛼-particles were deflected by small
angles. A few particles were reflected by 180o
C2 – Atomic Structure 32

Q4. According to Thomson’s model/Plum Pudding model of an atom:


I. The atom is __________ (spherical/oval) in shape.
II. The mass of the atom is __________ (uniformly/non-uniformly) distributed.
III. The positive charge of an atom is __________ (uniformly/non-uniformly) distributed.
Q5. Based on the information given answer the following:

Q6. Cross word Puzzle


Across:
1. These rays were discovered using modified cathode ray
experiment.
2. __________ proposed the nuclear model of atom.
3. The sub-atomic particle that moves from cathode to
anode.
4. This is positively charged electrode.
5. This property of an electron was given the value 9.1 ×
10−31 .
6. This isotope contains only 1 proton and 1 neutron.

Down:
1. Millikan’s oil drop experiment helped to find out this property of an electron.
4. These particles are formed when helium nucleus loses two electrons.
7. The sub-atomic particle that was discovered in the year 1932.
8. This screen is used to block 𝛼 −particles in Rutherford’s experiment.
9. These were used in Millikan’s oil drop experiment to ionize the air inside experimental chamber.
LEVEL 2
Q7. Find I) the total number and II) the total mass of neutrons present in 7 𝑚𝑔 of 14 𝐶. (Given that the mass of a
neutron = 1.675 × 10–27 𝑘𝑔).
Q8. An ion with mass number 37 possesses one unit of negative charge. If the ion contains 11.1% more neutrons
than the electrons, find the symbol of the ion.

C2.1
C2 – Atomic Structure 33

Q9. Complete the following table

Element No. of electrons No. of neutrons No. of protons


35
17𝐶𝑙 17
40
20𝐶𝑎 20
235
92𝑈 92

Q10. Identify the isobars and the isotopes from the list given below
3 39 40 4 12 13 40
2𝐻𝑒, 19𝐾 , 20𝐶𝑎 , 2𝐻𝑒, 6𝐶 , 6𝐶 , 19𝐾

HOMEWORK
LEVEL 1
Q1. The magnitude of electrical charge on an oil droplet is 𝑞 = 𝑛𝑒, where 𝑒 is the fundamental unit of electric charge
(1.6 × 10−19 C). 𝑛 can take
A) Only integral values B) Only fractional values C) All real values
Q2. If the static electric charge on an oil drop in Millikan’s experiment is – 1.282 × 10–18 𝐶, calculate the number of
electrons present on it.
Q3. Calculate the number of electrons which will together weigh one gram.
Q4. Which of the following statements about the electron is incorrect?
A) It is a negatively charged particle.
B) The mass of an electron is equal to the mass of a neutron.
C) It is a basic constituent of all uncharged atoms.
D) It is a constituent of cathode rays.
Q5. Which of the following will NOT show any deflection from their path when passed through an electric field?
Proton, cathode rays, electron, neutron, canal rays.
Q6. Two atoms are said to be isobars if:
A) They have same atomic number but different mass number.
B) They have same number of electrons but different number of neutrons.
C) They have same number of neutrons but different number of electrons.
D) Sum of the number of protons and neutrons is same but the number of protons is different.
Q7. Which of the following conclusions was NOT derived from Rutherford’s 𝛼-particle scattering experiment?
A) Most of the space in the atom is empty.
B) The radius of the atom is about 10−10 𝑚 while that of nucleus is 10−15 𝑚.
C) Electrons move in a circular path of fixed energy called orbits.
D) Electrons and the nucleus are held together by electrostatic forces of attraction.
Q8. An atom having atomic mass number 13 has 7 neutrons. What is the atomic number of the atom?

LEVEL 2
Q9. Calculate the mass and charge of one mole of electrons.
Q10. An ion with mass number 56 contains 3 units of positive charge and 30% more neutrons than electrons. Assign
the symbol to this ion.
C2 – Atomic Structure 34

C2.2 Dual Nature of EM Radiation and Photoelectric Effect


CONCEPTS
1. Properties of electromagnetic radiations: Wavelength, Frequency, and Amplitude.
2. Photoelectric Effect.

PRE-READING

Category Book Name Chapter Name Chapter Section

REQUIRED NCERT XI Structure of Atom 1 2.3.1 to 2.3.2

ADDITIONAL PRE-READING
1. Important quantities to be remembered:
𝑚𝑒 = 9.1 × 10−31 𝑘𝑔
Charge on an electron = 𝑞𝑒 = 1.6 × 10−19 𝐶
2. Stopping Potential: The opposing potential at which the photoelectric current becomes zero is called stopping
potential or cut-off potential. Stopping potential is that value of retarding potential difference between two plates
which is just sufficient to halt the most energetic photo electrons emitted. It is denoted by 𝑉𝑜 or 𝑉𝑠𝑡𝑜𝑝 .
If the Kinetic Energy of the emitted electrons is 𝐾𝐸 and 𝑞𝑒 is the charge on an electron, then the stopping potential
required to halt the photoelectrons is given by
𝐾𝐸
𝑉𝑜 =
𝑞𝑒
Or 𝑞𝑒 𝑉𝑜 = 𝐾𝐸
3. Conversion of joules to 𝒆𝑽 and vice versa:
One 𝑒𝑉 is the amount of energy gained (or lost) by an electron while moving through a potential difference of 1𝑉.
1 𝑒𝑉 = 1.60 × 10−19 𝐽
And hence 1 𝐽 = 6.24 × 1018 𝑒𝑉
Example 1: convert 3.7 𝑒𝑉 into 𝐽.
Solution: we know that 1 𝑒𝑉 = 1.60 × 10−19 𝐽
So that 3.7 𝑒𝑉= 3.7 × 1.60 × 10−19 𝐽 = 5.92 × 10−19 𝐽
Example 2: Convert 2.5 𝐽 into 𝑒𝑉
Solution: we know that 1 𝐽 = 6.24 × 1018 𝑒𝑉
So that 2.5 𝐽 = 2.5 × 6.24 × 1018 𝑒𝑉 = 1.56 × 1019 𝑒𝑉
𝒉𝒄
4. Computing Energy using 𝑬 =
𝝀

i) When 𝐸 needs to be calculated in joules: Use ℎ = 6.626 × 10−34 𝐽𝑠


ii) When 𝐸 needs to be calculated in eV:
Use ℎ = 4.136 × 10−15 𝑒𝑉𝑠
1242
Or directly substitute the value of wavelength (in nanometers) in the expression 𝐸 =
𝜆
1242
For example, if 𝜆 = 100 𝑛𝑚, then 𝐸 = = 12.42 𝑒𝑉
100

C2.2
C2 – Atomic Structure 35

PRE-READING EXERCISE

Q1. Origin and Nature of Electromagnetic Radiation


Electromagnetic radiation is emitted when a ___________ (charged/neutral) particle is ___________
(accelerated/moved with constant velocity).
Q2. Wave Nature of Electromagnetic Radiation
I. When radiation interacts with matter it shows ___________ − like (particle/wave) behavior. When it
propagates, it shows ___________ –like (wave/particle) behavior of electromagnetic radiation.
II. The wavelength of the visible spectrum of the electromagnetic radiation lies between _____ 𝑛𝑚 and _______𝑛𝑚.
III. X-rays have _________ (short/long) wavelengths and _________ (high/low) frequencies.
IV. Interference and diffraction can be explained by the _________ (wave/particle) nature of electromagnetic
radiation.
V. Yellow light emitted from a sodium lamp has a wavelength (𝜆) of 580 nm. Calculate the wavenumber (𝜈̅ ) of
the yellow light.
Q3. Quantum Nature of Electromagnetic Radiation
Planck proposed that atoms and molecules emit and absorb energy only in discrete quantities. The value of the
smallest quantum of energy that can be emitted or absorbed by electromagnetic radiation of frequency 𝜈 is given
by _________.
Q4. Photoelectric Effect: Einstein used this understanding of electromagnetic radiation to explain the photoelectric
effect. According to him,
I. A beam of light is basically a beam of light particles called _______.
II. The energy possessed by a photon of frequency 𝜈 is given by ________.
III. For every metal, the photoelectric effect is observed only above the threshold frequency. This behavior could
be explained by the ………. (wave/particle) behavior of electromagnetic radiation.
Q5. Convert
I. 20 𝐽 = _______ 𝑒𝑉
II. 1.6 𝑒𝑉 = ________ 𝐽

IN-CLASS EXERCISE
LEVEL 1
Q1. Calculate the wavelength, frequency and wavenumber of a light wave whose time period is 2.0 × 10–10 𝑠.
Q2. Determine the energy of a photon emitted from a light source of
I. Frequency = 1000 𝐻𝑧
II. Wavelength = 300 𝑛𝑚
Q3. A metal just begins to emit photoelectrons when a light of wavelength 1200 𝑛𝑚 is incident on it. Determine the
work function and threshold frequency of the metal. (Use ℎ = 6.6 × 10−34 𝐽𝑠).
Q4. The kinetic energy of an electron ejected due to photoelectric effect is found out to be 1.6 × 10−18 J. Determine
the positive voltage that needs to be applied to bring the electron to rest.
C2 – Atomic Structure 36

LEVEL 2
Q5. Electromagnetic radiation of wavelength 330 𝑛𝑚 is just sufficient to ionize a sodium atom. Calculate the
ionization energy of sodium in 𝑘𝐽 𝑚𝑜𝑙 –1 . (Use ℎ = 6.6 × 10−34 𝐽𝑠 and 𝑁𝐴 = 6 × 1023 )
Q6. A photon of wavelength 300 𝑛m strikes a metal surface whose work function is 2.136 𝑒𝑉. Calculate
I. the energy of the photon (𝑒𝑉) II. the kinetic energy of the emission
III. the velocity of the photoelectron (1 𝑒𝑉 = 1.6 × 10 𝐽, 𝑚𝑒 = 9.1 × 10−31 𝑘𝑔).
–19

Q7. The ejection of a photoelectron from silver metal in the photoelectric effect experiment can be stopped by
applying the voltage of 0.30 𝑉 when radiation of wavelength 310 𝑛𝑚 is used. Calculate the work function of silver
metal.
Q8. The threshold frequency 𝜈𝑜 for a metal is 7.0 × 1014 𝑠 −1 . Calculate the kinetic energy of an electron emitted when
radiation of frequency 𝜈 = 1.0 × 1015 𝑠 −1 hits the metal.
Q9. A 25 𝑤𝑎𝑡𝑡 bulb emits monochromatic yellow light of wavelength of 0.5 𝜇𝑚. Calculate the rate of emission of
photons per second. ( 1 𝑊𝑎𝑡𝑡 = 1 𝐽 / 𝑠)

LEVEL 3
Q10. When a photon of frequency 1.0 × 1015 𝑠 –1 was allowed to hit a metal surface, an electron having 0.626 ×
10–19 𝐽 of kinetic energy was emitted. Determine the kinetic energy of the electron emitted when a photon with
a wavelength equal to 600 𝑛𝑚 hits the metal surface.

HOMEWORK
LEVEL 1
Q1. Find the energy of photons which
I. Propagate with a wavelength of 0.50 Å.
II. Corresponds to light of frequency 6 × 1014 𝐻𝑧.
Q2. In a given metal, radiation of 600 𝑛𝑚 wavelength and another radiation of 150 𝑛𝑚 are incident. The threshold
frequency of the metal is 3× 1015 𝑠 −1 . Will there be any photoelectric effect observed in both cases?
Q3. A bulb emits light of wavelength 450 𝑛𝑚. The bulb is rated as 150 𝑤𝑎𝑡𝑡 and 8 percent of the energy is emitted as
light. How many photons are emitted by the bulb per second? (Watt is the unit of power).
Q4. Neon gas is generally used in sign boards. If a sign board emits light at 600 𝑛𝑚, calculate
I. The frequency of emission.
II. Distance traveled by this radiation in 30 𝑠.
III. Energy of one emitted photon.
IV. Number of photons emitted by the signboard every second if it produces 2 𝐽 of energy per second.
Q5. Chlorophyll present in green leaves of plants absorbs light at frequency 6.05 × 1014 𝐻𝑧 . Calculate the
wavelength of radiation in nanometers. Which part of the electromagnetic spectrum does it belong to?

LEVEL 2
Q6. How many photons of wavelength 4000 𝑝𝑚 can provide 1 𝐽 of energy?
Q7. Wavelengths of different radiations are given below:
𝜆𝐴 = 300 𝑛𝑚 𝜆𝐵 = 300 𝜇𝑚 𝜆𝐶 = 3 𝑛𝑚 𝜆𝐷 = 30 𝐴0
Arrange these radiations in the decreasing order of their energy.

C2.2
C2 – Atomic Structure 37

Q8. Light of frequency 𝜈 falls on a metal whose threshold frequency is 𝜈𝑜 . 𝑛 electrons are ejected per second. What is
the kinetic energy of one ejected electron?
Q9. Which of the following depends on the intensity of light falling on the metal?
I. Magnitude of photoelectric current
II. Kinetic energy of the electrons ejected
Q10. The work function for cesium atom is 1.2 𝑒𝑉. Calculate (a) the threshold wavelength and (b) the threshold
frequency of the radiation. If the Caesium element is irradiated with a wavelength 500 𝑛𝑚, calculate the kinetic
energy and the velocity of the ejected photoelectron. (To simplify calculation, use ℎ = 6.6 × 10−34 𝐽𝑠).

LEVEL 3
Q11. If the photon of the wavelength 150 𝑝𝑚 strikes an atom and one of its inner bound electrons is ejected out with
a velocity of 1.5 × 107 𝑚 𝑠 –1 , calculate the energy with which it is bound to the nucleus.
C2 – Atomic Structure 38

C2.3 Atomic Spectra and Bohr’s atomic Model


CONCEPTS
1. Emission and absorption spectra of an atom.
2. The formula used to express lines in a hydrogen spectrum.
3. Assumptions and postulates of Bohr’s atomic theory.
4. Expressions for the radius, velocity, frequency and energy of an electron in the 𝑛𝑡ℎ orbit of a Bohr atom.

PRE-READING

Category Book Name Chapter Name Chapter Section

REQUIRED NCERT XI Structure of Atom 1 2.3.3

PRE-READING EXERCISE
Q1. In a prism white light undergoes ___________ to split into its constituent wavelengths.
Q2. When white light is passed through a prism, ___________ light is deviated the most and ___________ light is deviated
the least.
Q3. Lyman, Balmer, Paschen, Brackett and Pfund are the names of the series of lines of the ___________
(emission/absorption) spectrum of the hydrogen atom.

Answer the next 3 questions on the basis of your understanding of Bohr’s model of the atom.
Q4. The circular paths of the electron around the nucleus of the hydrogen atom are called ___________, or ___________
Q5. An electron has to ___________ (absorb/release) energy to move from a lower stationary state to a higher stationary
state. An electron has to ___________ (absorb/release) energy to move from a higher energy state to a lower energy
state.
Q6. According to Bohr’s atomic model, angular momentum of an electron in the 3𝑟𝑑 orbit is __________ ℎ/2𝜋

IN-CLASS EXERCISE
LEVEL 1
Q1. What is the wavelength of light emitted when an electron in a hydrogen atom undergoes transition from an
energy level with 𝑛 = 4 to an energy level with 𝑛 = 2?
Q2. How much energy is required to ionise a 𝐻 atom if the electron occupies 5𝑡ℎ orbit?
Q3. Determine the ratio of the circumferences of the 3rd Bohr orbit of 𝐿𝑖 2+ and the 4th Bohr orbit of 𝐵𝑒 3+ .
Q4. Calculate the velocity of electron in the second excited state of Bohr orbit of hydrogen.

LEVEL 2
Q5. Determine the frequency of revolution of an electron in 2𝑛𝑑 Bohr’s orbit of hydrogen atom.

C2.3
C2 – Atomic Structure 39

Q6. What is the total possible number of emission lines when the excited electron of a 𝐻 atom in 𝑛 = 6 drops to the
2nd excited state?
Q7. Calculate the wavenumber of the longest wavelength transition in the Balmer series of atomic hydrogen.
Q8. What transition in the hydrogen spectrum would have the same wavelength as the Balmer transition from 𝑛 =
4 to 𝑛 = 2 of 𝐻𝑒 + spectrum?
Q9. If the radius of 3𝑟𝑑 Bohr orbit of 𝐻𝑒 + is 𝑟𝑜 , determine the radius of the 4𝑡ℎ Bohr orbit of 𝐵4+ in terms of 𝑟𝑎 .

HOMEWORK
LEVEL 1
Q1. Calculate the wave number of line associated with the transition in Balmer series of hydrogen when the electron
moves from 𝑛 = 4 orbit. (𝑅𝐻 = 109677 𝑐𝑚–1 )
Q2. What is the wavelength of the second line (𝛽-line or line with the second longest wavelength) in the Lyman series
of the hydrogen atom?
Q3. Calculate the frequency of the spectral line when an electron from the fifth orbit jumps to the second orbit in a
hydrogen atom. (𝑅𝐻 = 109677 𝑐𝑚−1 )
Q4. Calculate the
I. Radius of the stationary state 𝑛 = 2 in 𝐻𝑒 + ion.
II. Energy of the electron in the fifth orbit of the hydrogen atom.
III. Velocity of the electron in the first orbit of 𝐿𝑖 2+ ion.
Q5. Calculate the level to which an electron in an 𝐻 atom is excited if the emitted radiation belongs to the Paschen
series and has a wavelength of 1285 𝑛𝑚.

LEVEL 2
Q6. The wavelength corresponding to maximum energy transition for hydrogen is 91.2 𝑛𝑚. Find the corresponding
wavelength for 𝐻𝑒 + ion.
Q7. Calculate the energy required (in joules) to remove an electron of an 𝐻-atom completely from the 𝑛 = 2 orbit.
What is the wavelength of light (in 𝑐𝑚) that can be used to cause this transition?
Q8. In the hydrogen atom, the transition energy will be maximum for (𝑛𝑖 denotes the 𝑖𝑡ℎ orbit)
A) 𝑛5 → 𝑛4 B) 𝑛4 → 𝑛3
C) 𝑛3 → 𝑛2 D) Will be the same in all transitions
Q9. An electron in a hydrogen like species makes a transition from 𝑛𝑡ℎ Bohr orbit to next outer Bohr orbit (𝑛 + 1).
Find an approximate relation between the dependence of the frequency of the photon absorbed as a function of
𝑛. Assume ‘𝑛’ to be a large value. [For very small values of 𝑥 and large values of 𝑛, take (1 + 𝑥)𝑛 = (1 + 𝑛𝑥)].
Q10. What is the energy in joules, required to shift the electron of a hydrogen atom from the first Bohr orbit to the
second Bohr orbit and what is the wavelength of light emitted when the electron returns to the ground state?
Q11. Find the energy released when 2.0 moles of hydrogen atoms undergo transition giving a spectral line of lowest
energy in the visible region of its atomic spectra.
C2 – Atomic Structure 40

LEVEL 3
Q12. In a hydrogen – like species, electron is in 2𝑛𝑑 excited state. If the binding energy of 4𝑡ℎ state of this species is
13.6 𝑒𝑉, then, which of the following statements is true?
A) 22 𝑒𝑉 photon can set free the electron from the second excited state of this sample.
B) Photons of 3 different energies will be observed if an electron makes transition up to ground state from the
second excited state.
C) If a 23 𝑒𝑉 photon is incident on an electron in the 2𝑛𝑑 excited state, then 𝐾. 𝐸. of the ejected electron is 2 𝑒𝑉.
D) 2𝑛𝑑 line of Balmer series of this sample has same energy value as 1𝑠𝑡 excitation energy of 𝐻-atoms.

C2.3
C2 – Atomic Structure 41

C2.4 Quantum Mechanical Model of Atom


CONCEPTS
1. de Broglie’s relation between wavelength and momentum of a given particle.
2. Heisenberg’s uncertainty principle.
3. Quantum mechanical model of atom.
4. Significance of 𝑛, 𝑙, 𝑚𝑙 and 𝑚𝑠 with regard to a given atomic orbital.

PRE-READING

Category Book Name Chapter Name Chapter Section

COMPULSORY NCERT Class XI Part 1 Atomic Structure 2 2.6.1 and 2.6.2

ADDITIONAL PRE-READING
Relationship between kinetic energy and momentum of a particle
𝑝2
𝐾𝐸 = , where 𝑝 is the momentum of the particle.
2𝑚
We know that kinetic energy, 𝐾𝐸 of a particle with velocity = 𝑣 and mass = 𝑚 is given as
1
𝐾𝐸 = 𝑚𝑣 2
2
multiplying both numerator and denominator by 𝑚, we get
1 𝑚 1 (𝑚2 𝑣 2 )
𝐾𝐸 = 𝑚𝑣 2 . =
2 𝑚 2 𝑚
Since we know that momentum of a particle is given as 𝑝 = 𝑚𝑣, hence replacing 𝑚𝑣 in the above equation by 𝑝, we get
𝑝2
𝐾𝐸 =
2𝑚
This is an important result and should be remembered.

PRE-READING EXERCISE
Q1. ̂ 𝜓 = 𝐸𝜓.
Consider the fundamental equation of the quantum mechanical model: 𝐻
I. What is the above equation referred to as?
II. What is the operator 𝐻 called?
III. The function 𝜓 is called the ___________ of the orbital.
IV. Which of the following energies does E represent? (Tick all that apply)
A) Kinetic energy of electrons
B) Kinetic energy of nuclei
C) Attractive forces between nuclei and electrons
D) Repulsive forces between nuclei
Q2. The wave functions associated with the energy levels of a one-electron species are called ___________.
Q3. The probability density of finding an electron described by the wave function 𝜓, is proportional to ___________.
Q4. Quantum mechanics is useful for studying the motion of ___________ (microscopic /macroscopic) objects.
C2 – Atomic Structure 42

Q5. Which of the following shortcomings of Bohr’s atomic model does the quantum mechanical model overcome?
A) No consideration of wave particle duality of macroscopic objects
B) No regard to wave particle duality of microscopic objects
C) Adherence to Heisenberg’s uncertainty principle
D) All of the above
Q6. The Bohr’s model could not explain the Stark effect and the Zeeman effect? What are these effects?

IN-CLASS EXERCISE
LEVEL 1
Q1. What will be the wavelength of a ball of mass 100 𝑔 moving with a velocity of 10 𝑘𝑚/𝑠?
Q2. A table-tennis ball has a mass 10 𝑔 and a speed of (100 ± 0.1) 𝑚/𝑠. What will be the minimum uncertainty in its
position?
Q3. For an orbital with principal quantum number 𝑛 = 3
I. Write down all the possible values of azimuthal quantum number?
II. Write down the possible values of magnetic orbital quantum number for each of the above azimuthal
quantum numbers.
III. Specify the number of atomic orbitals.
IV. Specify the maximum number of electrons that can be accommodated.
Q4. Pick the set of degenerate orbitals in the following list.
2𝑝𝑥 , 3𝑝𝑥 , 3𝑝𝑦 , 2𝑠, 3𝑑𝑧 2

LEVEL 2
Q5. An electron is accelerated through a potential difference of 10 𝑉. Find the de Broglie wavelength associated with
the electron. Assume that the mass of an electron is 9.1 × 10−31 𝑘𝑔.
Q6. The mass of an electron is 9.1 × 10−31 𝑘𝑔. If its 𝐾. 𝐸. is 3 × 10−25 𝐽, calculate its wavelength.
Q7. A particle of mass 1 𝑔 moves with velocity 10 𝑚/𝑠. The error in its position measurement is 0.52 𝑝𝑚. Determine
the minimum percentage error in the measurement of its velocity.
Q8. Which of the following sets of quantum number(s) is(are) not possible? (More than 1 option may be correct)

𝑛 𝑙 𝑚𝑙 𝑚𝑠
A) 4 2 −2 +1/2
B) 3 0 0 −1/2
C) 3 2 −3 −1/2
D) 5 3 0 +1/2

Q9. How many sub-shells are associated with orbitals having 𝑛 = 4 ?

C2.4
C2 – Atomic Structure 43

HOMEWORK

LEVEL 1
Q1. Determine the ratio of the de Broglie wavelengths of two particles, whose velocities are in the ratio 1:2 and
masses are in the ratio 2: 3.
Q2. A cricket ball of mass 100 𝑔 does not seem to move like a wave when it is thrown by a bowler at a speed of
108 𝑘𝑚/ℎ. Calculate the wavelength of the ball and explain why its wave nature cannot be observed.
Q3. Calculate the minimum value of ∆𝑣. ∆𝑥 for the following objects. ∆𝑣 denotes the uncertainty in measuring
velocity and ∆𝑥 denotes the uncertainty in measuring position.
I. Dust particle 𝑚 = 1 𝑚𝑔 II. Electron 𝑚 = 9.1 × 10−31 𝑘𝑔
Q4. Match the quantum numbers with the information provided by them.

Quantum number Information provided

A) Principal quantum number P) Orientation of the orbital

B) Azimuthal quantum number Q) Distance from nucleus

C) Magnetic quantum number R) Spin of electron

D) Spin quantum number S) Shape of the orbital

Q5. Write the possible values of azimuthal quantum number 𝑙 when the principal quantum number 𝑛 is 2.

LEVEL 2
Q6. The ratio of the kinetic energies of two particles of mass 10 𝑔 and 40 𝑔 respectively is 1: 9. Determine the ratio
of their de Broglie wavelengths.
Q7. The circumference of the 5th Bohr orbit for the hydrogen atom is found to be 𝑛 times the de Broglie wavelength
associated with the electron in that orbit. Determine the value of 𝑛.
Q8. Calculate the wavelength of electron associated with energy of 1.0 electron volt.
Q9. A golf ball has a mass of 40 𝑔 and a speed of 45 𝑚/𝑠. If the speed can be measured with an accuracy of 2%,
calculate the uncertainty in its position.
Q10. How many orbitals can be described by the following quantum numbers?
I. 𝑛 = 2; 𝑙 = 1 II. 𝑛 = 4; 𝑚𝑙 = 2 III. 𝑛 = 3; 𝑙 = 0
Q11. In any sub-shell, what is the maximum number of electrons that have the same spin quantum number?
C2 – Atomic Structure 44

C2.5 Writing Electronic Configurations


CONCEPTS
1. Nodes and nodal plane.
2. Pauli's exclusion principle, Hund's rule of maximum multiplicity and Aufbau's principle to write valid electronic
configurations of species.
3. Screening Effect.

PRE-READING

Category Book Name Chapter Name Chapter Section

REQUIRED NCERT XI Structure of Atom 2 2.6

Several new concepts are being introduced in this pre-reading. Use the pre-reading to
INSTRUCTIONS
familiarize yourself with the key terms.

ADDITIONAL PRE-READING
1. To calculate maximum number of electrons that can be accommodated in a shell:
Considering each shell one by one, we notice that
For 𝑛 = 1, there is only one subshell present (s subshell), which can accommodate 2 electrons.
Hence for 𝑛 = 1, maximum number of electrons = 2(1)2 = 2
For 𝑛 = 2, there are two subshells present ( 𝑠 and 𝑝 subshell), which can accommodate 2 + 6 = 8 electrons
respectively.
Hence for 𝑛 = 2, maximum number of electrons = 2(2)2 = 8
For 𝑛 = 3, there are three subshells present ( 𝑠, 𝑝 and 𝑑 subshell), which can accommodate 2 + 6 + 10 = 18
electrons.
Hence for 𝑛 = 3, maximum number of electrons = 2(3)2 = 18
We can hence write a general formula:
Maximum number of electrons that can be accommodated in a shell with principal quantum number 𝑛 is equal to
2𝑛2 .

PRE-READING EXERCISE
Q1. For a hydrogen atom, the energy of the orbital depends on its ___________ quantum number.
Q2. For multi-electron species, lower the value of ___________ lower is the energy of the orbital.
Q3. Energies of orbitals in the same subshell decrease with increase in atomic number. True or False?
Q4. The number of subshells in the 4th shell is ___________.
Q5. For an electron occupying 2𝑝 orbital, the possible values of spin quantum numbers are ___________ and ___________.

C2.5
C2 – Atomic Structure 45

IN-CLASS EXERCISE

LEVEL 1
Q1. Arrange the following orbitals in the increasing order of energy using Aufbau’s principle
I. 1𝑠, 2𝑠, 3𝑠, 2𝑝 II. 4𝑠, 3𝑠, 3𝑝, 4𝑑 III. 5𝑝, 4𝑑, 5𝑑, 4𝑓, 6𝑠 IV. 5𝑓, 6𝑑, 7𝑠, 7𝑝
Q2. Write the electronic configuration, orbital electron diagram and number of unpaired electrons for the following
species :
I. 𝐵 (𝑍 = 5) II. 𝑃 (𝑍 = 15) III. 𝐹𝑒 (𝑍 = 26) IV. 𝑍𝑛 (𝑍 = 30)
Q3. What is the orbital angular momentum of an electron in the following orbitals :
I. 2s II. 4𝑑𝑥𝑦
Q4. Which of the following are isoelectronic species i.e., those having the same number of electrons?
𝑁𝑎+ , 𝐾 + , 𝑀𝑔2+ , 𝐶𝑎2+ , 𝑆 2– , 𝐴𝑟
Q5. Specify the number of radial and angular nodes in each of the following orbitals. Which of the following orbitals
will have the least number of radial nodes?
I. 3𝑝 II. 4𝑑𝑧 2 III. 2𝑝𝑥
1
Q6. What is the maximum number of electrons that can be present in a shell having 𝑛 = 4 and 𝑚𝑠 = − ?
2

LEVEL 2
Q7. Arrange the following orbitals of 𝑍𝑛 in the order of increasing effective nuclear charge
3𝑑𝑥𝑦 , 3𝑝𝑦 , 2𝑝𝑥 , 2𝑝𝑧 , 2𝑠

Q8. Consider the unpaired electrons present in the atoms of 𝐴𝑙 and 𝑆𝑖.
What is the azimuthal quantum number of the orbitals that these unpaired electrons are present in?
Q9. Shown below are some invalid electronic configurations. Which rule(s) is/are violated by the mentioned
electronic configurations?

I. II.

III. IV.

Q10. Full-dot represents the presence of an electron and empty-dot represents unfilled electrons.
Based on the orbital shell cards given below, determine the elements.
Fill in the electrons in the orbital shell cards according to the electronic configuration of the elements given
below.

I. a) Electronic Configuration: _______________ b) Electronic Configuration: _______________


Element: _______________ Element: _______________
C2 – Atomic Structure 46

II. a) Element: Nitrogen b) Element: Sulphur


Electronic Configuration: _______________ Electronic Configuration: _______________

Q11. Pick out the orbitals with the maximum number of nodal surfaces? (More than one options maybe correct)
A) 3𝑑𝑥𝑦 B) 4𝑑𝑧 2 C) 4𝑑𝑥𝑦 D) 2𝑝𝑥

HOMEWORK
LEVEL 1
Q1. Which neutral atoms are indicated by the following configurations?
I. [𝐻𝑒]2𝑠1 II. [𝑁𝑒]3𝑠 2 3𝑝3 III. [𝐴𝑟]4𝑠 2 3𝑑1
Q2. Write the electronic configurations of the atom having
I. 𝑍 = 23 II. 𝑍 = 38 III. 𝑍 = 42
Q3. What is the orbital angular momentum for a 𝑑-electron?
Q4. Which of the following pairs contains an isoelectronic species? (More than one option can be correct)
I. 𝑁𝑎+ , 𝑀𝑔2+ II. 𝐴𝑙 3+ , 𝑂– III. 𝑁𝑎+ , 𝑂2– IV. 𝑁 3– , 𝐶𝑙 –

LEVEL 2
Q5. Arrange the electrons described by the quantum number quadruple (𝑛, 𝑙, 𝑚, 𝑠) in decreasing order of energies
I. (1,0,0,1/2) II. (2,1, −1,1/2) III. (3,0, −1, −1/2) IV. (2,0,0,1/2)
Q6. Determine the number of electrons present in orbitals having a value of the azimuthal quantum number 𝑙 = 1 in
each of the following species
I. 𝐶𝑢 II. 𝑀𝑛
Q7. Write the value of the quantum numbers (𝑛, 𝑙, 𝑚) for the unpaired electron present in
I. 𝑀𝑛2+ II. 𝐴𝑔

Q8. Determine the number of unpaired electrons present in the following species:
I. 𝐴𝑔 (𝑍 = 47) II. 𝑀𝑛2+ (𝑍 = 25)

C2.5
C2 – Atomic Structure 47

Additional Reading
Derivation of Bohr’s Radius, Velocity of an Electron and Energy of an Electron Using Bohr’s Theory
1. In a hydrogen atom, the centripetal force (associated with revolving electron) is being supplied by the columbic
force of attraction between the nucleus (protons) and the electrons.

Hence,
𝐹𝑐𝑒𝑛𝑡𝑟𝑖𝑝𝑒𝑡𝑎𝑙 = 𝐹𝑒𝑙𝑒𝑐𝑡𝑟𝑜𝑠𝑡𝑎𝑡𝑖𝑐
𝑚𝑣𝑛2 −𝑒(𝑍𝑒)
= |𝑘 |
𝑟𝑛 𝑟𝑛2
𝑍𝑒 2
𝑚𝑣𝑛2 = 𝑘 eq. (i)
𝑟𝑛
Where Z represents the atomic number (the number of protons)
𝑒 is the charge on an electron(−𝑒) and a proton(+𝑒)
𝑘 is a constant
𝑟𝑛 is the radius of Bohr orbit
𝑣𝑛 is the velocity of revolution of the electron in the orbit.
Before equating these two forces, Bohr hypothesized that the electron's angular momentum was quantized.

Angular momentum, 𝐿 = 𝑛 ( ) (𝐿 = 𝐼𝜔)
2𝜋

𝐼𝜔 = 𝑛 ( )
2𝜋
𝑣𝑛 ℎ
𝑚𝑟𝑛2 ( ) = 𝑛 ( )
𝑟𝑛 2𝜋

𝑚𝑣𝑛 𝑟𝑛 = 𝑛 ( ) eq. (ii)
2𝜋
𝑛ℎ
Where L = angular momentum of the electron =
2𝜋
Upon solving eq. (i) and eq. (ii),
𝑍𝑒 2
𝑚𝑣𝑛2 = 𝑘
𝑟𝑛
𝑛ℎ 2 𝑍𝑒 2
𝑚( ) =𝑘
2𝜋𝑚𝑟𝑛 𝑟𝑛
𝑛2 ℎ 2 𝑍𝑒 2
𝑚( )=𝑘
4𝜋2 𝑚2 𝑟𝑛2 𝑟𝑛
𝑛2 ℎ 2 2
= 𝑘𝑍𝑒
4𝜋2 𝑚𝑟𝑛
𝑛2 ℎ 2
𝑟𝑛 =
4𝜋2 𝑘𝑚𝑍𝑒 2
2 ℎ2
𝑟𝑛 = 𝑛 ( )
4𝜋2 𝑘𝑚𝑍𝑒 2
C2 – Atomic Structure 48

For a ground state hydrogen electron,


𝑛 = 1 and 𝑍 = 1
ℎ2
𝑟1 =
4𝜋2 𝑘𝑚𝑒 2
𝑟1 = 0.53 × 10−10 𝑚
This is the value of energy of Bohr’s ground state orbit.
2. Bohr's second hypothesis in his model was that an electron only loses or releases energy (and therefore a photon)
when it goes from lower to higher energy state or drops from a higher energy state to a lower energy state. In
order to determine the energy lost/gained by the electron, an expression for an electron's total energy has to be
developed.
The electric potential energy (EPE) for an electron is given by
𝑬𝑷𝑬 = 𝒒𝑽𝒂𝒃𝒔
𝑘(𝑍𝑒)
𝐸𝑃𝐸 = −𝑒 ( )
𝑟𝑛
𝑘(𝑍𝑒 2 )
𝐸𝑃𝐸 = −
𝑟𝑛
Where 𝑉 is the voltage, and 𝑞 or 𝑒 is the charge on an electron
By extending the centripetal force relationship, an expression can also be derived for the electron's kinetic energy
𝑭𝒄𝒆𝒏𝒕𝒓𝒊𝒑𝒆𝒕𝒂𝒍 = 𝑭𝒆𝒍𝒆𝒄𝒕𝒓𝒐𝒔𝒕𝒂𝒕𝒊𝒄
𝑚𝑣𝑛2 −𝑒(𝑍𝑒)
= |𝑘 |
𝑟𝑛 𝑟𝑛2
𝑍𝑒 2
𝑚𝑣𝑛2 = 𝑘
𝑟𝑛
1 𝑍𝑒 2
𝑚𝑣𝑛2 = 𝑘
2 2𝑟𝑛
𝑍𝑒 2
𝐾𝐸 = 𝑘
2𝑟𝑛
𝟏
Because kinetic energy = 𝑲𝑬 = 𝒎𝒗𝟐
𝟐
Thus, the total energy, 𝐸𝑛 , of an electron equals
𝑬𝒏 = 𝑬𝑷𝑬 + 𝑲𝑬
𝑘(𝑍𝑒 2 ) 𝑍𝑒 2
=− +𝑘
𝑟𝑛 2𝑟𝑛
𝑍𝑒 2
= −𝑘
2𝑟𝑛
In this equation, notice that the total energy is negative. This indicates that the electron is trapped in an energy
well about the nucleus; that is, it would need additional energy to ionize or free the electron.
Substituting in the value for 𝑟1 into this total energy expression yields a ground state energy of 2.18 × 10−18 𝐽 or
−13.6 𝑒𝑉 for a hydrogen atom. Using the fact that 𝑟𝑛 = 𝑛2 𝑟1
ℎ2
𝑟𝑛 = 𝑛2 ( )
4𝜋2 𝑘𝑚𝑍𝑒 2
2
𝑟𝑛 = 𝑛 𝑟1

Read.
C2 – Atomic Structure 49

We can now compute the first four energy levels for hydrogen.
𝑬𝟏 = −𝟏𝟑. 𝟔 𝒆𝑽
𝑬𝟏
𝑬𝟐 = 𝟐 = −𝟑. 𝟒 𝒆𝑽
𝟐
𝑬𝟏
𝑬𝟑 = 𝟐 = −𝟏. 𝟓𝟏 𝒆𝑽
𝟑
𝑬𝟏
𝑬𝟒 = 𝟐 = −𝟎. 𝟖𝟓 𝒆𝑽
𝟒
3. Bohr's second hypothesis combined with Planck's formula for quantized energy (𝐸 = ℎ𝜈) will now allow us to
derive Balmer's equation. Remember that the energy released by the electron during de-excitation equals the
energy of the emitted photon.
Let's begin by assuming that an electron is falling from 𝐸𝑗 , a high energy state, to 𝐸𝑖 , a lower energy state.
∆𝐸 = 𝐸𝑗 − 𝐸𝑖
𝑍𝑒 2 𝑍𝑒 2
∆𝐸 = −𝑘 − (−𝑘 )
2𝑟𝑗 2𝑟𝑖
𝑍𝑒 2 1 1
∆𝐸 = −𝑘 ( − )
2 𝑟𝑗 𝑟𝑖
ℎ2
Where 𝑟𝑛 = 𝑛2 ( )
4𝜋2 𝑘𝑚𝑍𝑒 2

𝑍𝑒 2 1 1
∆𝐸 = −𝑘 ( ℎ2
− ℎ2
)
2 𝑗2 ( ) 𝑖2 ( )
4𝜋2 𝑘𝑚𝑍𝑒 2 4𝜋2 𝑘𝑚𝑍𝑒 2
𝑍𝑒 2 4𝜋 2 𝑘𝑚𝑍𝑒 2 1 1
∆𝐸 = −𝑘 ( ) ( 2 − 2)
2 ℎ2 𝑗 𝑖
2 2 4
2𝜋 𝑚𝑍 𝑒 1 1
∆𝐸 = −𝑘 2 ( 2 − 2)
ℎ2 𝑗 𝑖
2 2 4
2𝜋 𝑚𝑍 𝑒 1 1
∆𝐸 = 𝑘 2 ( 2 − 2)
ℎ2 𝑖 𝑗
𝑐
∆𝐸𝑙𝑜𝑠𝑡 𝑏𝑦 𝑡ℎ𝑒 𝑒𝑙𝑒𝑐𝑡𝑟𝑜𝑛 = 𝐸𝑝ℎ𝑜𝑡𝑜𝑛 = ℎ𝑓 = ℎ
𝜆
𝑐 2𝜋 2 𝑚𝑍 2 𝑒 4 1 1
ℎ = 𝑘2 ( 2 − 2)
𝜆 ℎ2 𝑖 𝑗
C2 – Atomic Structure 50

1 2𝜋 2 𝑚𝑍 2 𝑒 4 1 1
= 𝑘2 ( 2 − 2)
𝜆 𝑐ℎ3 𝑖 𝑗
For 𝑍 = 1,
1 2𝜋 2 𝑚𝑒 4 1 1
= 𝑘2 ( 2 − 2)
𝜆 𝑐ℎ3 𝑖 𝑗
1 1 1
= 1.097 × 107 ( 2 − 2 ) 𝑚−1
𝜆 𝑖 𝑗
1 1 1
= 𝑅 ( 2 − 2)
𝜆 𝑖 𝑗
[Putting the value of the constants 𝒌, 𝒆, 𝒎𝒆 , 𝝅, 𝒄, 𝒉]
Putting i = 2, and 𝑗 = {3, 4, 5, 6, … … } gives us Balmer's empirical formula.
4. To obtain the expression for velocity of the revolving electron, we equate centripetal force with coulombic force
between two point charges.
𝑚𝑣 2 𝑧𝑒 2
i. e. , =
𝑟 4𝜋𝜀𝑟 2
𝑧𝑒 2
⇒ that 𝑟 = (i)
4𝜋𝜀𝑚𝑣 2
𝑛ℎ
also, 𝑚𝑣𝑟 =
2𝜋
𝑛ℎ
⇒𝑟 = (ii)
2𝜋𝑚𝑣
equating (i) and (ii),
𝑍𝑒 2 𝑛ℎ
=
4𝜋𝜀𝑚𝑣 2 2𝜋𝑚𝑣
𝑧𝑒 2
so that 𝑣 =
2𝜀𝑛ℎ
for hydrogen, Z = 1 so that
2
(1.6×10−19 )
𝑣 = [2(8.85×10−12)(1)(6.626×10−34)] = 𝑣 = 2.18 × 106 𝑚/𝑠

Read.
C2 – Atomic Structure 51

Test Practice Problems


Purpose: To practice a mixed bag of questions in a speed based format similar to what you will face in entrance
examinations. In most entrance examinations, you will get not more than 3 minutes to attempt a question.
Hence, you need to be able to attempt a question in less than 3 minutes, and at the end of 3 minutes skip the
question and move to the next one.
Approach:
 Attempt the Test Practice Problems only when you have the stipulated time available at a stretch.
 Start a timer and attempt the section as a test.
 DO NOT look at the answer key / solutions after each question.
 DO NOT guess a question if you do not know it. Competitive examinations have negative marking.
 Solve as much as possible within the stipulated time, and then fill the OMR provided at the end of the TPP.
 Fill the table at the end of the TPP and evaluate the number of attempts, and accuracy of attempts, which will
help you evaluate your preparedness level for the chapter.

TEST PRACTICE PROBLEMS – 1


No. of questions: 30 Total time: 90 Minutes Time per question: 3 Minutes

Q1. Radius of 3rd orbit of 𝐿𝑖 2+ ion is ‘𝑥’ 𝑐𝑚 then de-broglie wavelength of electrons in the 1st orbit is
2𝜋𝑥 2𝜋𝑥
A) 𝑐𝑚 B) 6𝜋𝑥 𝑐𝑚 C) 3𝜋𝑥 𝑐𝑚 D) 𝑐𝑚
3 6

Q2. The ionization energy of a hydrogen atom in terms of Rydberg constant (𝑅𝐻 ) is given by the expression
A) 𝑅𝐻 ℎ𝑐 B) 𝑅𝐻 𝑐 C) 2𝑅𝐻 ℎ𝑐 D) 𝑅𝐻 𝑁𝐴 ℎ𝑐
Q3. STATEMENT-1: If an electron is located within the range of 0.1 Å then the uncertainty in velocity is
approximately 6 × 106 𝑚/𝑠.
STATEMENT-2: Trajectory (path of motion) of above electron can be defined clearly.
[ℎ = 6.6 × 10−34 , 𝑚𝑒 = 9.1 × 10−31 𝑘𝑔]
A) Statement-1 is True, Statement-2 is True; Statement-2 is a correct explanation for Statement-1.
B) Statement-1 is True, Statement-2 is True; Statement-2 is NOT a correct explanation for Statement-1.
C) Statement-1 is True, Statement-2 is False
D) Statement-1 is False, Statement-2 is True

Q4. What is the number of electrons in chromium ( 24𝐶𝑟) that have an orbital angular momentum equal to ?
√2𝜋
A) 10 B) 12 C) 9 D) 7
Q5. An isobar of 20𝐶𝑎40 is
40 38 42 38
A) 18𝐴𝑟 B) 20𝐶𝑎 C) 20𝐶𝑎 D) 18𝐴𝑟

Q6. Which of the following make up an isotonic triad?


78 77 74 233 232 239
A) 32𝐺𝑒 , 33𝐴𝑠, 31𝐺𝑎 B) 92𝑈, 90𝑇ℎ, 94𝑃𝑢
13 12 14 14 16 15
C) 6𝐶 , 7𝐶 , 7𝑁 D) 6𝐶 , 8𝑂 , 7𝑁

Q7. The number of electrons and neutrons of a neutral atom is 18 and 20 respectively. Its mass number is.
A) 2 B) 17 C) 37 D) 38
C2 – Atomic Structure 52

Q8. The work function (𝛷) of some metals is listed below. The number of metals which will show photoelectric effect
when light of 300 𝑛𝑚 wavelength falls on the metal is:

Metal 𝐿𝑖 𝑁𝑎 𝐾 𝑀𝑔 𝐶𝑢 𝐴𝑔 𝐹𝑒 𝑃𝑡 𝑊
Ф(𝑒𝑉) 2.4 2.3 2.2 3.7 4.8 4.3 4.7 6.3 4.75

A) 2 B) 4 C) 6 D) 8
Q9. A gas absorbs a photon of 355 𝑛𝑚 and emits two wavelengths. If one of the emission is at 680 𝑛𝑚, the other is at
A) 1035 𝑛𝑚 B) 325 𝑛𝑚 C) 743 𝑛𝑚 D) 518 𝑛𝑚
Q10. The frequency of light emitted for the transition 𝑛 = 4 to 𝑛 = 2 of 𝐻𝑒 + is equal to the transition in 𝐻 atom
corresponding to which of the following?
A) 𝑛 = 3 to 𝑛 = 1 B) 𝑛 = 2 to 𝑛 = 1
C) 𝑛 = 3 to 𝑛 = 2 D) 𝑛 = 4 to 𝑛 = 3
Q11. The energy of an electron in first Bohr orbit of 𝐻-atom is −13.6 𝑒𝑉. The possible energy value of electron in the
excited state of 𝐿𝑖 2+ is
A) −122.4 𝑒𝑉 B) 30.6 𝑒𝑉
C) −30.6 𝑒𝑉 D) 13.6 𝑒𝑉
Q12. The electronic transitions from 𝑛 = 2 to 𝑛 = 1 will produce shortest wavelength in (where 𝑛 = principal
quantum state)
A) 𝐿𝑖 2+ B) 𝐻𝑒 + C) 𝐻 D) 𝐻 +
8
Q13. The wave number of the spectral line in the emission spectrum of hydrogen will be equal to times the Rydberg’s
9
constant if the electron jumps from
A) 𝑛 = 3 to 𝑛 = 1 B) 𝑛 = 10 to 𝑛 = 1
C) 𝑛 = 9 to 𝑛 = 1 D) 𝑛 = 2 to 𝑛 = 1
Q14. The ratio of the difference in energy between the first and the second Bohr orbit to that between the second and
the third Bohr orbit is
1 1 4 27
A) B) C) D)
2 3 9 5

Q15. The number of photons emitted per second by a 60 W source of monochromatic light of wavelength 663 nm is
(ℎ = 6.63 × 10−34 𝐽𝑠)
A) 4 × 10−20 B) 1.54 × 1020 C) 3 × 10−20 D) 2 × 1020
Q16. An electron from one Bohr stationery orbit can go to next higher orbit.
A) By emission of electromagnetic radiation
B) By absorption of any electromagnetic radiation
C) By absorption of electromagnetic radiation of particular frequency
D) Without emission or absorption of electromagnetic radiation
Q17. The ionization energy of hydrogen atom is 13.6 eV. What will be the ionization energy of 𝐻𝑒 + ?
A) 13.6 eV B) 54.4 eV C) 122.4 eV D) Zero
Q18. According to Bohr’s theory, the angular momentum of an electron in 5𝑡ℎ orbit is
ℎ ℎ ℎ ℎ
A) 25 B) 1.0 C) 10 D) 2.5
𝜋 𝜋 𝜋 𝜋

T.P.P.
C2 – Atomic Structure 53

Q19. An electron is moving in Bohr’s fourth orbit. Its de-Broglie wavelength is 𝜆. What is the circumference of the
fourth orbit?
2 4
A) B) 2𝜆 C) 4𝜆 D)
𝜆 𝜆

Q20. Which of the following statements does not form a part of Bohr’s model of hydrogen atom?
A) Energy of the electrons in the orbits is quantized
B) The electrons in the orbit nearest the nucleus have the lowest energy
C) Electrons revolve in different orbits around the nucleus.
D) The position and velocity of the electrons in the orbit cannot be determined simultaneously.
Q21. If a helium atom and hydrogen molecule are moving with the same velocity, the ratio of their de-Broglie
wavelengths will be
A) 4:1 B) 1:2 C) 2:1 D) 1:4
Q22. Minimum uncertainty in the position of an electron (mass= 9.1 × 10−31 𝑘𝑔) moving with a velocity 300 𝑚𝑠 −1 ,
having a 0.001% error will be (ℎ = 6.63 × 10−34 𝐽𝑠)
A) 19.3 × 10−2 𝑚 B) 5.76 × 10−2 𝑚 C) 1.93 × 10−2 𝑚 D) 3.84 × 10−2 𝑚
Q23. The minimum uncertainties in the velocities of two particles A and B are 0.05 and 0.02 𝑚𝑠 −1 respectively. The
∆𝑥𝐴
mass of B is five times to that of mass A. what is the ratio of minimum uncertainties ( ) in their positions?
∆𝑥𝐵
A) 2 B) 0.25 C) 4 D) 1
Q24. What accelerating potential is needed to produce an electron beam with an effective wavelength of 0.090 Å?
A) 1.86 × 104 𝑒𝑉 B) 1.86 × 102 𝑒𝑉 C) 2.86 × 104 𝑒𝑉 D) 2.86 × 102 𝑒𝑉
Q25. Which of the following sets of quantum numbers is incorrect?
1 1
A) 𝑛 = 5, 𝑙 = 4, 𝑚 = 0, 𝑠 = + B) 𝑛 = 3, 𝑙 = 3, 𝑚 = +3, 𝑠 = +
2 2
1
C) 𝑛 = 6, 𝑙 = 0, 𝑚 = 0, 𝑠 = − D) 𝑛 = 4, 𝑙 = 2, 𝑚 = +2, 𝑠 = 0
2

Q26. The representation of the ground state electronic configuration of 𝐻𝑒 by orbital–electron diagram as ↿↿ is wrong
because it violates
A) Heisenberg’s uncertainty principle
B) Bohr’s quantization theory of angular momenta
C) Pauli exclusion principle
D) Hund’s rule
Q27. Consider 5 electrons whose principal and azimuthal quantum numbers are as follows:
I. 𝑛 = 3; 𝑙 = 2 II. 𝑛 = 5; 𝑙 = 0
III. 𝑛 = 4; 𝑙 = 1 IV. 𝑛 = 4; 𝑙 = 2
V. 𝑛 = 4; 𝑙 = 0
Can be placed in order of increasing energy, as
A) I < V < III < IV < II B) I < V < III < II < IV
C) V < I < III < II < IV D) V < I < IV < III < II
Q28. The stability of ferric ion (𝐹𝑒 3+ , atomic number 𝑜𝑓 𝐹𝑒 = 26) is due to
A) Half-filled 𝑓 − subshell B) Half-filled 𝑑 − subshell
C) completely filled 𝑓 − subshell D) completely filled 𝑑 − subshell
C2 – Atomic Structure 54

Q29. Select the incorrect graph for velocity of 𝑒 − , 𝑣 in an orbit versus atomic number 𝑍 and principal quantum
number 𝑛:

A) v B) v

n 1/n

C) v D) v

z n
Q30. 𝐴𝐵3+ ion and a proton are accelerated by the same potential, then their de-Broglie wavelengths will be in the
ratio:
(Assume mass of proton = mass of neutron and mass number of 𝐵 = 9)
A) 1:2 B) 1:4 C) 1:1 D) 1: 3√3

TEST PRACTICE PROBLEMS – 2


No. of questions: 30 Total time: 90 Minutes Time per question: 3 Minutes

Q31. Which among the following is the incorrect option?


A) X-rays : 𝜈 = 1018 𝐻𝑧 B) Microwaves : 𝜈 = 1010 𝐻𝑧
C) Long radio waves: 𝜈 = 100 − 1014 𝐻𝑧 D) UV rays: 𝜈 = 104 𝐻𝑧
Q32. Electrons are emitted with zero velocity from a metal surface when it is exposed to a radiation of wavelength
6800 Å.
A) Threshold frequency 𝜈0 = 3 × 1012 𝐻𝑧 B) Work function 𝜙 = 29.22 × 10−20 𝐽
C) Kinetic energy = 4.9 𝑒𝑉 D) None of the above
Q33. Iodine molecule breaks into atoms after absorbing light of 4500 Å. If one quantum of radiation is absorbed by
each molecule, calculate the energy of iodine atoms. Energy used for breaking up iodine molecule is 240 𝐾𝐽/𝑚𝑜𝑙.
A) 3.9 × 10−19 𝐽 B) 2.1 × 10−20 𝐽 C) 4.07 × 10−16 𝐽 D) 1.32 × 10−19 𝐽
Q34. What happens when violet light is incident on potassium metal? (Work function of potassium = 2.3 eV)
A) No change is observed. B) Electrons are just ejected with 𝐾. 𝐸 = 0.
C) Electrons are ejected with sufficient kinetic D) Ejection of some potassium atom takes
energy. place.
Q35. An electron jumps from 𝑛 = 3 to 𝑛 = 2 in a hydrogen atom. Which among the given is true? (Use ℎ = 6.6 ×
10−34 𝐽𝑠)
A) Energy of radiation emitted during the transition is 9 × 10−19 𝐽.
B) Frequency of the radiation emitted is 5.7 × 1014 𝐻𝑧
C) Wavelength of the transition is 540 𝑛𝑚.
D) None of the above.

T.P.P.
C2 – Atomic Structure 55

Q36. What is the maximum number of emission lines when the excited electron of a 𝐻 atom in 𝑛 = 5 drops to the
ground state?
A) 4 B) 6 C) 10 D) 12
Q37. Which excited state in the given hydrogen-like species will have the same radius as that of the Bohr radius of
hydrogen atom?
A) 𝑛 = 2, 𝐿𝑖 2+ B) 𝑛 = 2, 𝐵𝑒 3+ C) 𝑛 = 2, 𝐻𝑒 + D) 𝑛 = 3, 𝐿𝑖 2+
Q38. Ratio of frequencies of revolution of electrons in the 2𝑛𝑑 excited state of 𝐻𝑒 + to the 2𝑛𝑑 state of hydrogen is:
A) 32/27 B) 27/32 C) 1/54 D) 27/2
−1
Q39. The velocities of two particles 𝐴 and 𝐵 are 0.05 and 0.02 𝑚𝑠 respectively. The mass of 𝐵 is five times the mass
of 𝐴. The ratio of their de Broglie’s wavelengths is
A) 2:1 B) 1:4 C) 1:1 D) 4:1
Q40. A possible set of quantum numbers for the outermost electron of copper (𝑍𝐶𝑢 = 29) in its ground state is
1 1 1 1
A) 4, 1, 1, + B) 3, 2, 2, + C) 4, 0, 0, + D) 4, 2, 2, +
2 2 2 2

Q41. Which of the following sets of quantum numbers represents the highest energy of an atom?
1 1
A) 𝑛 = 3, 𝑙 = 1, 𝑚 = 1, 𝑠 = + B) 𝑛 = 3, 𝑙 = 2, 𝑚 = 1, 𝑠 = +
2 2
1 1
C) 𝑛 = 4, 𝑙 = 0, 𝑚 = 0, 𝑠 = + D) 𝑛 = 3, 𝑙 = 0, 𝑚 = 0, 𝑠 = +
2 2

Q42. Which of the following is a correct set of quantum numbers for an electron?
1 1
A) 𝑛 = 1, 𝑙 = 1, 𝑚 = 0, 𝑠 = − B) 𝑛 = 0, 𝑙 = 1, 𝑚 = 1, 𝑠 = +
2 2
1 1
C) 𝑛 = 3, 𝑙 = 1, 𝑚 = −2, 𝑠 = − D) 𝑛 = 2, 𝑙 = 0, 𝑚 = 0, 𝑠 =
2 2

Q43. Which among the given statements is correctly matched?


A) Azimuthal quantum number: describes spatial orientation of orbitals.
B) Number of orbitals for a given shell is 2𝑛2 .
C) Number of subshells is always: less than 𝑛.
D) Principal quantum number: determines the energy of atomic orbital.
Q44. Which of the following statements are correct?
I. The electronic configuration of chromium is [Ar]3𝑑 5 4𝑠1 .
II. The magnetic quantum number may have a negative value.
III. In silver atom, 23 electrons have a spin of one type and 24 of the opposite type. (Atomic number of silver is 47).
IV. 3𝑠 has one radial node.
A) I, II, III B) II, III, IV C) III, IV D) I, II, IV
Q45. The electrons identified by quantum numbers 𝑛 and 𝑙, (i) 𝑛 = 4, 𝑙 = 1, (ii) 𝑛 = 4, 𝑙 = 0, (iii) 𝑛 = 3, 𝑙 = 2, (iv) 𝑛 =
3, 𝑙 = 1 can be placed in the order of increasing energy from the lowest to highest as
A) (iv) < (ii) < (iii) < (i) B) (ii) < (iv) < (i) < (iii)
C) (i) < (iii) < (ii) < (iv) D) (iii) <(i) < (iv) < (ii)
Q46. The increasing order for the values of 𝑒/𝑚 (change/mass) is:
A) 𝑒, 𝑝, 𝑛, 𝛼 B) 𝑛, 𝑝, 𝑒, 𝛼 C) 𝑛, 𝑝, 𝛼, 𝑒 D) 𝑛, 𝛼, 𝑝, 𝑒
C2 – Atomic Structure 56

Q47. The ratio of specific charge of a proton and an 𝛼-particle is


A) 2 ∶ 1 B) 1 ∶ 2 C) 1 ∶ 4 D) 1 ∶ 1
Q48. The mass to charge ratio (𝑚/𝑒) for a univalent cation is 1.5 × 10−8 𝑘𝑔/𝐶. What is the mass of this atom?
A) 2.4 × 10−19 𝑔 B) 2.4 × 10−27 𝑔 C) 2.4 × 10−24 𝑔 D) None of these
Q49. 𝛼-particles are represented by
A) Lithium atoms B) Helium nuclei C) Hydrogen nuclei D) None of these
Q50. Assume that 2 × 10−17 𝐽 of light energy is needed by the interior of the human eye to see an object. How many
photons of yellow light with 𝜆 = 595.5 𝑛𝑚 are needed to generate this minimum energy?
A) 6 B) 30 C) 45 D) 60
Q51. Find the value of wave number (𝑣̅ ) in terms of Rydberg’s constant (R), when transition of electron takes place
between two levels of 𝐻𝑒 + ion whose sum is 4 and difference is 2.
8𝑅 32𝑅 3𝑅
A) B) C) D) none of these
9 9 4

Q52. Which graph shows how the energy 𝐸 of a photon of light is related to its wavelength(𝜆)?

A) B) C) D)

Q53. According to Einstein’s photoelectric equation, the graph between kinetic energy of photoelectrons ejected and
the frequency of the incident radiation is:

A) B) C) D)

Q54. Slope of 𝑉0 vs 𝜈 curve is (where 𝑉0 = Stopping potential, 𝜈 = subjected frequency)


A) 𝑒 ℎ
B)
𝑒
C) 𝜙 D) ℎ
Q55. The photoelectric emission from a surface starts only when the light incident upon the surface has certain
minimum :
A) intensity B) wavelength C) frequency D) velocity
Q56. If 𝜆0 and 𝜆 be the threshold wavelength and the wavelength of incident light respectively, the velocity of photo-
electrons ejected from the metal surface is:
2ℎ 2ℎ𝑐 2ℎ𝑐 𝜆 −𝜆 2ℎ 1 1
A) √ (𝜆0 − 𝜆) B) √ (𝜆0 − 𝜆) C) √ ( 0 ) D) √ ( − )
𝑚 𝑚 𝑚 𝜆𝜆 0 𝑚 𝜆 𝜆 0

Q57. A light source of wavelength 𝜆 illuminates a metal and ejects photo-electrons with (𝐾. 𝐸)max = 1 𝑒𝑉
𝜆
Another light source of wavelength , ejects photo-electrons from same metal with (𝐾. 𝐸)max = 4 𝑒𝑉
3
Find the value of work function?
A) 1 𝑒𝑉 B) 2 𝑒𝑉
C) 0.5 𝑒𝑉 D) None of these

T.P.P.
C2 – Atomic Structure 57

Q58. Electromagnetic radiation having 𝜆 = 310 Å is subjected to a metal sheet having work function = 12.8 𝑒𝑉. What
will be the velocity of photo-electrons having maximum kinetic energy?
A) 0, no emission will occur B) 4.352 × 106 𝑚/𝑠
C) 3.09 × 106 𝑚/𝑠 D) 8.72 × 106 𝑚/𝑠
Q59. The ratio of slopes of 𝐾𝐸max vs 𝜈 and 𝑉0 vs 𝜈 curves in the photoelectric effect gives (𝜈 = frequency, 𝐾max =
maximum kinetic energy, 𝑉0 = stopping potential):
A) charge of an electron
B) Planck’s constant
C) work function
D) the ratio of Planck’s constant to electronic charge
Q60. In Bohr’s stationary orbits
A) Electrons do not move B) Electrons move emitting radiations
C) Energy of the electron remains constant D) Angular momentum of the electron is ℎ/2𝜋

TEST PRACTICE PROBLEMS – 3


No. of questions: 30 Total time: 90 Minutes Time per question: 3 Minutes

Q61. Ionization energy for hydrogen atom in ergs, Joules and 𝑒𝑉 respectively is
A) 21.8 × 10−12 , 2.18 × 10−18 , 13.6 B) 13.6 × 218 × 10−20 , 21.8 × 10−13
C) 21.8 × 10−20 , 13.6, 21.8 × 10−13 D) 21.8 × 10−13 , 21.8 × 10−20 , 13.6
Q62. If in Bohr’s model, for unielectronic atom, time period of revolution is represented as 𝑇𝑛,𝑍 where 𝑛 represents
shell no. and 𝑍 represents atomic number then the value of 𝑇1,2 : 𝑇2,1 will be:
A) 8 ∶ 1 B) 1 ∶ 8 C) 1 ∶ 1 D) 1 ∶ 32
−13.6
Q63. The energy of an electron moving in 𝑛th Bohr’s orbit of an element is given by 𝐸𝑛 = 𝑍 2 𝑒𝑉/atom (𝑍 = atomic
𝑛2
number). The graph of 𝐸 vs. 𝑍 2 (keeping “𝑛” constant) will be:

A) B) C) D)

Q64. Which of the following electronic transitions in a hydrogen atom will absorb the largest amount of energy?
A) From 𝑛 = 1 to 𝑛 = 2 B) From 𝑛 = 2 to 𝑛 = 4
C) From 𝑛 = 5 to 𝑛 = 1 D) From 𝑛 = 3 to 𝑛 = 5
Q65. Which electronic transition in a hydrogen atom, starting from the orbit 𝑛 = 7, will produce infrared light of
wavelength 2170 𝑛𝑚? (Given 𝑅𝐻 = 1.09677 × 107 𝑚−1 )
A) 𝑛 = 7 to 𝑛 = 6 B) 𝑛 = 7 to 𝑛 = 5 C) 𝑛 = 7 to 𝑛 = 4 D) 𝑛 = 7 to 𝑛 = 3
Q66. When an electron jumps from 𝐿 to 𝐾 shell:
A) Energy is absorbed
B) Energy is released
C) Energy is neither absorbed nor released
D) Energy is sometimes absorbed and sometimes released
C2 – Atomic Structure 58

Q67. How do the energy gaps between successive energy levels in an atom vary from low to high 𝑛 values?
A) All energy gaps are the same
B) The energy gap decreases as 𝑛 increases
C) The energy gap increases as 𝑛 increases
D) The energy gap changes unpredictably as 𝑛 increases
Q68. Which of the following has the largest de Broglie wavelength (all have equal velocity)?
A) 𝐶𝑂2 molecule B) 𝑁𝐻3 molecule C) Electron D) Proton
Q69. The de-Broglie wavelength associated with a particle of mass 10−6 𝑘𝑔 with a velocity of 10 𝑚 𝑠 −1 is:
A) 6.63 × 10−22 𝑚 B) 6.63 × 10−29 𝑚 C) 6.63 × 10−31 𝑚 D) 6.63 × 10−34 𝑚

Q70. √𝑉 of two particles 𝐴 and 𝐵 are plotted against de-Broglie wavelengths, where 𝑉 is the stopping potential of the
particles. Which of the following relation is correct about the mass of particles?

A) 𝑚𝐴 = 𝑚𝐵 B) 𝑚𝐴 > 𝑚𝐵 C) 𝑚𝐴 > 𝑚𝐵 D) 𝑚𝐴 ≤ 𝑚𝐵
Q71. Which one of the following graphs correctly represents the variation of particles momentum with de-Broglie
wavelength?

A) B) C) D)

Q72. The de-Broglie wavelength of an electron accelerated by an electric field of 𝑉 volts is given by:
1.23 1.23 1.23 1.23
A) 𝜆 = B) 𝜆 = 𝑚 C) 𝜆 = 𝑛𝑚 D) 𝜆 =
√𝑚 √ℎ √𝑉 𝑉

Q73. The momentum (in 𝑘𝑔-𝑚/𝑠) of photon having 6 𝑀𝑒𝑉 energy is:
A) 3.2 × 10−21 B) 2.0 C) 1.6 × 10−21 D) none of these
Q74. The mass of a particle is 10−10 𝑔 and its radius is 2 × 10−4 𝑐𝑚 . If its velocity is 10−6 𝑐𝑚/𝑠 with 0.0001%
uncertainty in measurement, the minimum uncertainty in its position is:
A) 5.2 × 10−8 𝑚 B) 5.2 × 10−7 𝑚 C) 5.2 × 10−6 𝑚 D) 5.2 × 10−9 𝑚
Q75. If an electron is travelling at 200 𝑚/𝑠 within 1 𝑚/𝑠 uncertainty, what is the theoretical uncertainty in its position
in 𝜇𝑚 (micrometer)?
A) 14.5 B) 29 C) 58 D) 114
Q76. “The exact path of electron in 2𝑝-orbital cannot be determined.” The above statement is based upon
A) Hund’s Rule B) Bohr’s Rule
C) Uncertainty principle D) Aufbau principle
Q77. Which series of subshells is arranged in the order of increasing energy for multi-electron atoms?
A) 6𝑠, 4𝑓, 5𝑑, 6𝑝 B) 4𝑓, 6𝑠, 5𝑑, 6𝑝 C) 5𝑑, 4𝑓, 6𝑠, 6𝑝 D) 4𝑓, 5𝑑, 6𝑠, 6𝑝

T.P.P.
C2 – Atomic Structure 59

Q78. Ψ 2 (𝑟, 𝜃, ϕ) represents: (for Schrodinger wave mechanical model)


A) Amplitude of electron wave
B) Probability density of electron
C) Total probability of finding electron around nucleus
D) Orbit
Q79. For which of the following sets of quantum numbers, an electron will have the highest energy?
𝑛 𝑙 𝑚 𝑠 𝑛 𝑙 𝑚 𝑠
A) 3 2 1 − 1⁄2 B) 4 3 −1 +1/2
C) 4 1 −1 +1/2 D) 5 0 0 −1/2

Q80. Maximum number of electrons in a subshell is given by


A) (2𝑙 + 1) B) 2(2𝑙 + 1) C) (2𝑙 + 1)2 D) 2(2𝑙 + 1)2
Q81. Which two orbitals are located along the axis, and not between the axis?
A) 𝑑𝑥𝑦 , 𝑑𝑧 2 B) 𝑑𝑥𝑦 , 𝑝𝑧 C) 𝑑𝑦𝑧 , 𝑝𝑥 D) 𝑝𝑧 , 𝑑𝑥2−𝑦2

Q82. In a set of degenerate orbitals the electrons distribute themselves to retain similar spins as far as possible. This
statement is attributed to
A) Pauli’s exclusion principle B) Aufbau principle
C) Hund’s Rule D) Slater rule
Q83. Which of the following rules could explain the presence of three unpaired electrons in 𝑁-atom?
A) Hund’s rule B) Aufbau’s rule
C) Heisenberg’s uncertainty principle D) Pauli’s exclusion principle
Q84. The orbital angular momentum of 3𝑝 electron is :
C) zero ℎ
A) √3 ℎ B) √6 ℎ D) √2
2𝜋

Q85. The orbital diagram in which both the Pauli’s exclusion principle and Hund’s rule are violated, is :
A) B) C) D)

Q86. The ratio of magnetic moments of 𝐹𝑒 (𝐼𝐼𝐼) and 𝐶𝑜 (𝐼𝐼) is :


A) √5 ∶ √7 B) √35 ∶ √15 C) 7 ∶ 3 D) √24 ∶ √15
Q87. A compound of vanadium has a magnetic moment (𝜇) of 1.73 𝐵𝑀. If the vanadium ion in the compound is present
as 𝑉 𝑥+ , then, the value of 𝑥 is?
A) 1 B) 2 C) 3 D) 4
Q88. The correct order of screening effects of 𝑠, 𝑝, 𝑑, 𝑓 sub-shells is:
A) 𝑠 > 𝑝 > 𝑑 > 𝑓 B) 𝑠 < 𝑝 < 𝑑 < 𝑓 C) 𝑑 > 𝑝 > 𝑠 > 𝑓 D) 𝑠 > 𝑓 > 𝑑 > 𝑝
Q89. The Schrodinger wave equation for hydrogen atom is
3
1 1 2 𝑟
Ψ2𝑠 = ( ) (2 − ) 𝑒 −𝑟/𝑎0
4√2𝜋 𝑎0 𝑎0
Where 𝑎0 is Bohr’s radius. If the radial node in 2𝑠 is at a distance 𝑟0 , then 𝑟0 would be equal to:
𝑎0 𝑎0
A) B) 2𝑎0 C) √2𝑎0 D)
2 √2
C2 – Atomic Structure 60

Q90. The Schrodinger wave equation for hydrogen atom is


1 𝑍 3/2
Ψ(radial) = ( ) [(𝜎 − 1)(𝜎 2 − 8𝜎 + 12)]𝑒 −𝜎/2
16√4 𝑎0
2𝑍𝑟
where 𝑎0 and 𝑍 are the constants in which answer can be expressed and 𝜎 = . The minimum and maximum
𝑎0
positions of radial nodes from nucleus are _______ and _______ respectively.
𝑎0 3𝑎0 𝑎0 𝑎0 𝑎0 3𝑎0 𝑎0 4𝑎0
A) , B) , C) , D) ,
𝑍 𝑍 2𝑍 𝑍 2𝑍 𝑍 2𝑍 𝑍

DATA Guide

A # of questions Total problems in TPP

B # Attempts Total attempts in OMR

C # Correct Total questions correct

D # Incorrect Out of the ones marked in OMR

E # Unattempted 𝐴– 𝐵

ANALYSIS

𝐵
F Percentage attempts × 100
𝐴
𝐶
G Percentage Accuracy × 100
𝐵

Question type # Correct (C) # Incorrect (I) # Unattempted (U)

Easy

Medium

Hared

Tip: To begin with, your accuracy must be high, typically > 60%. Percentage attempts should be > 50%
As time progresses, your percentage attempts should increase without a reduction in accuracy.
Additionally, you should be able to get > 80% Level 1 questions correct, as they involve basic recall of the concepts
and formulae of the chapter.

T.P.P.
C2 – Atomic Structure 61

Answer Key
C2.1 SUB ATOMIC PARTICLES
PRE READING EXERCISE Q4. I. Spherical
II. Uniformly
Q1.
III. Uniformly
Sub – atomic Charge
particle (positive/negative/neutral) Q5. Atom1
Mass number: no. of protons + no. of neutrons =
Electron Negative 10
Atomic Number: 3 (corresponds to lithium)
Proton Positive
Atom designation: 103𝐿𝑖
Neutron Neutral Atom 2
Number of neutrons: 11 − 5 = 6
Q2. Anode Number of electrons = 5 (Atomic number, 𝑍 =
5, or boron)
Q3. Repel, attract
Atom designation: 115𝐵
Q4. False Atom 3
Q5. I. atomizer Number of electrons = Number of protons=7
II. electric field (corresponds to nitrogen)
Number of neutrons =14 − 7 = 7
Q6. Zinc Sulphide
Atom designation: 147𝑁
Atom4
IN CLASS EXERCISE
No. of protons = 9
LEVEL 1 Atomic number = 9 (corresponds to fluorine)
Q1. I. Charged Particle Atom designation: 199𝐹
II. Only B Q6. Across: 1. Charge, 3. Lead, 4. Anode, 5.
III. a) Positive terminal Rutherford, 7. Neutron, 8. Mass
b) Electrons Down: 2. Electron, 5. Xrays, 6. Tritium.
c) 1.758820 × 1011 𝐶/𝐾𝑔
IV. The reversal of magnetic field will produce
glow at C.
Q2. The 𝑋-ray beam was used to ionize the air inside
the experimental chamber which would in turn
put charge on the oil molecules.
Q3. I. Thomson’s and Rutherford’s model of atom.
II. D) Electrical neutrality of an atom
III. C) 𝛼-particles
IV. 1. Most of the space in the atom is empty.
2. The positive charge and most of the
mass of the atom occupies a small
volume which was called nucleus.

LEVEL 2
Q7. I. 24 × 1020 neutrons
II. 40.2 × 10−7 𝑘𝑔
C2 – Atomic Structure 62

37 −
Q8. The symbol of the ion is 𝐶𝑙
17
Q9.
𝑨
Element ( 𝑿) No. of electrons No. of neutrons (𝑨 − 𝒁) No. of protons
𝒛
35
𝐶𝑙 17 18 17
17
40
𝐶𝑎 20 20 20
20
235
𝑈 92 143 92
92

40 40 3 4 Q4. B
Q10. ( 𝐶𝑎, 𝐾) and ( 𝐻𝑒, 𝐻𝑒) are pairs of
20 19 2 2
39 40 12 13 Q5. Neutron
isobars; ( 𝐾, 𝐾) and ( 𝐶, 𝐶) are pairs of
19 19 6 6 Q6. D
isotopes.
Q7. C
HOMEWORK Q8. Number of Neutrons, 𝑁 = 7
∴ Atomic number, 𝑍 = 𝐴 − 𝑁 = 6.
LEVEL 1
LEVEL 2
Q1. A
Q9. 5.48 × 10−4 𝑔, −9.64 × 104 𝐶
Q2. 8
56 3+
Q3. 1.097 × 1027 electrons Q10. 𝐹𝑒
26

C2.2 PHOTOELECTRIC EFFECT


PRE-READING EXERCISE Q2. I. 6.626 × 10−31 𝐽
II. 6.626 × 10−19 𝐽
Q1. Charged, accelerated
Q3. 𝑣𝑜 = 0.25 × 1015 sec −1
Q2. I. Particle, wave
𝜙 = 1.65 × 10−19 𝐽
II. 400 𝑛𝑚, 700 𝑛𝑚
III. short, high Q4. 𝑉𝑜 = 10 𝑉
IV. wave
V. 1.72 × 106 𝑚−1 LEVEL 2

Q3. ℎ𝜈 Q5. 360 𝑘𝐽/𝑚𝑜𝑙

Q4. I. Photons Q6. I. 4.14 𝑒𝑉


II. ℎ𝜈 II. 2 𝑒𝑉
III. Particle III. 0.84 × 106 𝑚/𝑠

Q5. I. 12.48 × 1019 eV Q7. 3.7 𝑒𝑉


II. 2.56 × 10−19 𝐽 Q8. 1.988 × 10−19 𝐽
Q9. 𝑛 = 6.25 × 1019 𝐽
IN-CLASS EXERCISE
LEVEL 3
LEVEL 1
Q10. No electron will be emitted.
Q1. 𝜈 = 5 × 109 𝐻𝑧; 𝜆 = 6 𝑐𝑚; 𝜈̅ = 0.167 𝑐𝑚−1
HOMEWORK

Ans.
C2 – Atomic Structure 63

LEVEL 1
Q8. ℎ𝜈0 energy will be required to just free the
−15 electrons.
Q1. I. 3.976 × 10 𝐽
II. 3.976 × 10−19 𝐽 Q9. I. Higher the intensity, higher will be the
Q2. Photoelectric effect is not observed in either number of photons striking the metallic
case. surface per unit time, higher will be the
number of ejected electrons and hence
Q3. 𝑛 = 27.2 × 1018
higher will be the magnitude of
Q4. I. 5 × 1014 𝐻𝑧 photocurrent.
II. 9 × 109 𝑚 II. Kinetic energy of the electrons ejected
III. 3.313 × 10−19 𝐽 depends upon frequency of light and not the
IV. 6 × 1018 intensity.
Q5. 500 𝑛𝑚. This light belongs to the visible part of Q10. 6.6 × 105 𝑚/𝑠
EM radiation.
LEVEL 3
LEVEL 2
Q11. 1.22 × 10−15 𝐽
Q6. 2 × 1016 photons
Q7. 𝐸(𝐷) = 𝐸(𝐶) > 𝐸(𝐴) > 𝐸(𝐵)

C2.3 ATOMIC SPECTRA AND BOHR’S ATOMIC MODEL


PRE-READING EXERCISE HOMEWORK
Q1. Refraction LEVEL 1
Q2. Violet, red Q1. 𝜈̅ = 20564.44 𝑐𝑚−1
Q3. Emission Q2. 𝜆 = 0.10 × 10−4 𝑐𝑚
Q4. Orbits/ stationary states/shells Q3. 6.90 × 1014 𝐻𝑧
Q5. Absorb, release Q4. I. 1.058 Å
Q6. 3 II. −8.72 × 10−20 𝐽
III. 6.54 × 106 𝑚/𝑠
IN-CLASS EXERCISE
Q5. 𝑛𝑓 = 5
LEVEL 1
LEVEL 2
Q1. 𝜆 = 5 × 107 𝑚
Q6. 22.8 𝑛𝑚
Q2. 0.544 𝑒𝑉
Q7. 𝜆 = 3.65 × 10−5 𝑐𝑚
Q3. 0.75
Q8. C) Maximum for 𝑛3 → 𝑛2 transition
Q4. 7.27 × 105 𝑚/𝑠𝑒𝑐
2𝑅𝐻 𝑍 2 𝑐
Q9.
LEVEL 2 𝑛3

Q10. 12.19 × 10−8 𝑚


Q5. 0.082 × 1016 𝐻𝑧
Q11. 3.66 × 105 𝐽
Q6. 6 emission lines
Q7. 1.524 × 106 𝑚−1 LEVEL 3

Q8. The transition is from 𝑛 = 2 to 𝑛 = 1 in 𝐻- atom. Q12. B


32
Q9. 𝑟1 = 𝑟
45 𝑜
C2 – Atomic Structure 64

C2.4 QUANTUM MECHANICAL MODEL OF ATOM


PRE READING EXERCISE
III. 9
Q1. I. Schrodinger Equation IV. 18 𝑒 −
II. Hamiltonian Operator
Q4. 3𝑝𝑥 , 3𝑝𝑦
III. Wave Function
IV. E represents the total energy of the system. LEVEL 2
Hence all the options A, B, C, D are correct.
Q5. 0.38 × 10−9 𝑚
Q2. Atomic orbitals
Q6. 0.89 × 10−6 𝑚
Q3. |𝜓|2
Q7. 10−18 %
Q4. Microscopic
Q8. C)
Q5. D
Q9. 4 subshells are associated with 𝑛=
Q6. Stark Effect is the splitting of spectral line in the
4, namely, 𝑠, 𝑝, 𝑑 and 𝑓.
presence of an electric field. Zeeman Effect is the
splitting of spectral lines in the presence of HOMEWORK
magnetic field. LEVEL 1
IN CLASS EXERCISE Q1. 3: 1
LEVEL 1 Q2. 2.20 × 10−34 𝑚
Q1. 6.626 × 10−37 𝑚. Here we can observe that the wavelength of the
ball is of the order of 10−34 𝑚 which is negligible
Q2. 5.27 × 10−32 𝑚
as compared to the dimensions of sub-atomic
Q3. I. 0, 1 and 2 particles. Therefore, the wave properties of the
II. Hence the possible values of 𝑚𝑙 are: ball cannot be observed.
𝑙 𝑚𝑙 Q3. I. 0.52 × 10−28 𝑚2 /𝑠
0 0 II. 5.8 × 10−5 𝑚2 /𝑠
1 −1, 0, +1
2 −2. −1, 0, +1, +2
Q4.
Quantum number Information provided
A) Principal quantum number Q) Distance from nucleus
B) Azimuthal quantum number S) Shape of the orbital
C) Magnetic quantum number P) Orientation of the orbital
D) Spin quantum number R) Spin of electron

Q5. Possible values of 𝑙 are 0 and 1 Q9. 1.465 × 10−33 𝑚

LEVEL 2 Q10. I. 3; 2𝑝𝑥 . 2𝑝𝑦 , 2𝑝𝑧


II. 12
Q6. 6∶1 III. 1; 3s
Q7. 𝑛=5 Q11. In any sub-shell the maximum number of
electrons with the same spin quantum number
Q8. 1.23 × 10−9 𝑚
will be equal to (2𝑙 + 1).

Ans.
C2 – Atomic Structure 65

C2.5 WRITING ELECTRONIC CONIGURATIONS


1
PRE READING EXERCISE Q6. Electrons having 𝑚𝑠 = − and 𝑛 = 4 will be 16
2
Q1. Principal electrons
Q2. 𝑛+𝑙 LEVEL 2
Q3. True
Q7. 3𝑑𝑥𝑦 <3𝑝𝑦 < 2𝑝𝑥 = 2𝑝𝑧 < 2𝑠
Q4. 4 (namely 𝑠, 𝑝, 𝑑 and 𝑓)
1 1 Q8. 𝑙=1
Q5. + ,−
2 2
Q9. I. Aufbau Principle
II. Hund’s rule
IN-CLASS EXERCISE
III. Pauli’s Exclusion Principle
LEVEL 1 IV. Aufbau Principle, Hund’s Rule
Q1. I. 1𝑠 < 2𝑠 < 2𝑝 < 3𝑠 Q10. I. a) Magnesium 1𝑠 2 2𝑠 2 2𝑝6 3𝑠 2
II. 3𝑠 < 3𝑝 < 4𝑠 < 4𝑑 b) Fluorine 1𝑠 2 2𝑠 2 2𝑝5
III. 4𝑑 < 5𝑝 < 6𝑠 < 4𝑓 < 5𝑑
IV. 7𝑠 < 5𝑓 < 6𝑑 < 7𝑝
Q2. I. 𝐵 = 1𝑠 2 2𝑠 2 2𝑝1 II. a)
1𝑠 2𝑠 2𝑝
𝐵: ↿⇂ ↿⇂ ↿ ↑↑ ↑↑
II. 𝑃 = 1𝑠 2 2𝑠 2 2𝑝6 3𝑠 2 3𝑝3
↿⇂ ↿⇂ ↿⇂ ↿⇂ ↿⇂ ↿⇂ ↿ ↿ ↿
1𝑠 2𝑠 2𝑝 3𝑠 3𝑝 b)
2 2 6 2 6 2 6
III. 𝐹𝑒 = 1𝑠 2𝑠 2𝑝 3𝑠 3𝑝 4𝑠 3𝑑
↿⇂ ↿⇂ ↿⇂ ↿⇂ ↿⇂ ↿⇂ ↿⇂ ↿⇂ ↿⇂ ↿⇂
1𝑠 2𝑠 2𝑝 3𝑠 3𝑝 4𝑠 Q11. B), C)
↿⇂ ↿ ↿ ↿ ↿ The maximum number of nodal surface is 𝑛 − 1
3𝑑
A) For 3 𝑑𝑥𝑦 , the maximum number of nodal
IV. 𝑍𝑛 = 1𝑠 2 2𝑠 2 2𝑝6 3𝑠 2 3𝑝6 4𝑠 2 3𝑑10
surface will be 2
↿⇂ ↿⇂ ↿⇂ ↿⇂ ↿⇂ ↿⇂ ↿⇂ ↿⇂ ↿⇂ ↿⇂
B) For 4 𝑑𝑧 2 it will be 3
1𝑠 2𝑠 2𝑝 3𝑠 3𝑝 4𝑠
C) For 4 𝑑𝑥𝑦 it will be 3
↿⇂ ↿⇂ ↿⇂ ↿⇂ ↿⇂
3𝑑 D) For 2𝑝𝑥 it will be 1

Q3. I. 0 HOMEWORK
√6ℎ
II. LEVEL 1
2𝜋

Q4. 𝑁𝑎+ and 𝑀𝑔2+ are isoelectronic Q1. I. Lithium


𝐾 + , 𝐶𝑎2+ , 𝑆 2− and 𝐴𝑟 are isoelectronic II. Phosphorus
III. Scandium
Q5. I. Number of radial nodes = 𝑛 − 𝑙 − 1 = 1
Number of angular nodes = 𝑙 = 1 Q2. I. 𝑍(23) = 1𝑠 2 2𝑠 2 2𝑝6 3𝑠 2 3𝑝6 4𝑠 2 3𝑑 3
II. Number of radial nodes = 𝑛 − 𝑙 − 1 = 1 II. 𝑍(38)
Number of angular nodes = 𝑙 = 2 = 1𝑠 2 2𝑠 2 2𝑝6 3𝑠 2 3𝑝6 4𝑠 2 3𝑑10 4𝑝6 5𝑠 2
III. Number of radial nodes = 𝑛 − 𝑙 − 1 = 0 III. 𝑍(42)
Number of angular nodes = 𝑙 = 1. = 1𝑠 2 2𝑠 2 2𝑝6 3𝑠 2 3𝑝6 3𝑑10 4𝑠 2 4𝑝6 4𝑑 5 5𝑠1

Q3. √6
2𝜋
C2 – Atomic Structure 66

Q4. I. 𝑁𝑎+ and 𝑀𝑔2+ represent an isoelectronic Q6. Azimuthal quantum number = 1 represents p
pair. subshell.
II. 𝐴𝑙 3+ and 𝑂− do not represent an I. The number of electrons present in p
isoelectronic pair. subshell of 𝐶𝑢 is hence 6 + 6 = 12
III. 𝑁𝑎+ and 𝑂2− represent an isoelectronic II. The number of electrons present in p
pair. subshell of 𝑀𝑛 is hence 6 + 6 = 12
IV. 𝑁𝑎+ and 𝐶𝑙 − do not represent an (𝑛, 𝑙) ≡ (3, 2)
Q7. I.
isoelectronic pair.
𝑚𝑙=2 = −2, −1, 0, +1, +2
LEVEL 2 II. (𝑛, 𝑙) ≡ (5, 0)
𝑚𝑙=0 = 0
Q5. III > II > IV > I
Q8. I. Number of unpaired electrons in 𝐴𝑔 = 1
II. Number of unpaired electrons in 𝑀𝑛 = 5

ANSWERS TO TEST PRACTICE PROBLEMS


Mark (C) / (I) / (U) Mark (C) / (I) / (U)
Q. No. Ans. Level Q. No. Ans. Level
as appropriate as appropriate
Q1. A Medium Q19. C Medium

Q2. A Medium Q20. D Easy

Q3. C Medium Q21. B Easy

Q4. B Easy Q22. C Easy

Q5. A Easy Q23. A Medium

Q6. D Easy Q24. A Hard

Q7. D Easy Q25. B Easy

Q8. B Medium Q26. C Easy

Q9. C Medium Q27. C Easy

Q10. B Hard Q28. B Easy

Q11. C Easy Q29. D Medium

Q12. A Medium Q30. D Medium

Q13. A Medium Q31. D Easy


Q14. D Medium Q32. B Medium
Q15. D Medium Q33. B Hard
Q16. C Easy
Q34. C Hard
Q17. B Hard
Q35. D Medium
Q18. D Easy
Q36. C Easy

Ans.
C2 – Atomic Structure 67

Q37. B Medium Q64. A Medium

Q38. A Hard Q65. C Medium

Q39. A Medium Q66. B Easy

Q40. C Easy Q67. B Easy

Q41. B Easy Q68. C Medium

Q42. D Medium Q69. B Easy

Q43. D Easy Q70. B Medium

Q44. A Medium Q71. C Easy

Q45. A Easy Q72. C Medium

Q46. D Easy Q73. A Medium

Q47. A Medium Q74. A Easy

Q48. C Easy Q75. C Easy

Q49. B Easy Q76. C Easy

Q50. D Medium Q77. A Easy

Q51. B Medium Q78. B Easy

Q52. D Medium Q79. B Medium

Q53. C Medium Q80. B Easy

Q54. B Medium Q81. D Medium

Q55. C Easy Q82. C Easy

Q56. C Medium Q83. A Easy

Q57. C Medium Q84. D Easy

Q58. C Medium Q85. D Medium

Q59. A Medium Q86. B Easy

Q60. C Easy Q87. D Medium

Q61. A Easy Q88. A Easy

Q62. D Medium Q89. B Medium

Q63. B Easy Q90. C Hard


C3 – Periodic Properties of Elements 68

Fifth Edition

C3. Periodic Properties of Elements


TABLE OF CONTENTS

C3. Periodic Properties of Elements 68


C3.1 Early Classifications of Elements and the Modern Periodic Table ……………………………………………..……69
C3.2 Periodic Trends of Properties ………………………………………………………………………………………………………72
Test Practice Problems …………………………………………………………………………………………………………………..……75
Answer Key …………………………………………………………………………………………………………………………………………81
C3 – Periodic Properties of Elements 69

C3.1 Early Classifications of Elements and the Modern Periodic Table


CONCEPTS
1. The necessity of grouping together of elements
2. Mendeleev and Moseley’s contribution in the formation of the modern periodic table
3. Periods and groups in a periodic table
4. The general electronic configurations and properties of groups
5. Metallic and covalent radii
6. Comparing sizes of anions and cations

PRE-READING

Category Book Name (Edition) Chapter Section

COMPULSORY NCERT Class XI Chemistry Part 1 3 3.1 − 3.6.4 , 3.7.1 (a)

PRE-READING EXERCISE
Q1. Atoms have sharp boundaries. (True/False)
Q2. The atomic radius can be determined by measuring the distance between atoms in ___________ state.
Q3. The term atomic radius refers to ___________ radius and ___________ radius.
Q4. Atomic radii can be measured by ___________ methods.
Q5. Atomic size ___________ (increases/decreases) across the period.
Q6. The size of the cation is ____________ than the size of parent atom, while the size of the anion is ___________ then the
size of the parent atom.

IN-CLASS EXERCISE
LEVEL 1
Q1. How would you name elements with the following atomic numbers according to the 𝐼𝑈𝑃𝐴𝐶 system?
I. 𝑍 = 108 II. 𝑍 = 115 III. 𝑍 = 110
Q2. The atomic numbers of three elements are given below. Write down the group number and the period to which
they belong.
I. 𝑍 = 22 II. 𝑍 = 31 III. 𝑍 = 17

Q3. What will be the atomic number of the second last element in the 4th period of the modern periodic table?
Q4. Arrange the following atoms in the order of increasing atomic radius
I. 𝑆𝑟, 𝐶𝑎, 𝐵𝑎, 𝑀𝑔, 𝐵𝑒 II. 𝐹, 𝑂, 𝑃, 𝑁𝑎, 𝑀𝑔 III. 𝐶𝑜, 𝑁𝑖, 𝐶𝑢, 𝑀𝑛, 𝑆𝑐
Q5. Classify the following elements into 𝑠-block, 𝑝-block, 𝑑-block and 𝑓-block elements:
I. 𝑅ℎ(𝑍 = 45) II. 𝑆𝑒(𝑍 = 34) III. 𝑁𝑑(𝑍 = 60) IV. 𝑆𝑟(𝑍 = 38)
C3 – Periodic Properties of Elements 70

LEVEL 2
Q6. Classify the elements with the following atomic numbers as alkali metals, transition metals, chalcogens, halogens,
noble gases, lanthanoids or actinoids.
𝑍 = 17, 𝑍 = 40, 𝑍 = 91, 𝑍 = 11, 𝑍 = 29
Q7. Arrange the following in an increasing order of size
𝑀𝑔, 𝑀𝑔2+ , 𝐴𝑙, 𝐴𝑙 3+
Q8. Arrange the following species in the order of increasing ionic radius
I. 𝑁𝑎+ , 𝑀𝑔2+ , 𝐹 − , 𝑂2− , 𝐴𝑙 3+ II. 𝐶𝑎2+ , 𝐾 + , 𝑆 2− , 𝐶𝑙 −

LEVEL 3
Comprehension (Q9 – Q10)
Let’s consider a hypothetical planet “Pseudo Earth” which is similar to our earth in several aspects. The similarities are

On Pseudo Earth:
I. The number of elements are the same and they are known by the same name.
II. Pauli’s exclusion principle, Hund’s rule and Aufbau principle are known to the people of pseudo earth in the same
manner as we know on our earth.
III. They classify elements as representative, transition and inner transition elements in the same manner as we classify
on our earth.
However, there is one basic difference in understanding the electron’s spin on these two earths. On our earth
electron can have only two spin directions clockwise and anti-clockwise, while on pseudo earth there is an
additional possible value of spin quantum number called neutral spin in which electron is believed to be fluctuating
harmonically between clockwise and anti-clockwise spins. Answer the following three questions based on the
above information.

Q9. The first noble gas on the pseudo planet would be


A) 𝐻𝑒 B) 𝐻 C) 𝐿𝑖 D) 𝑁𝑒
Q10. On pseudo earth, atomic number of the first transition metal would be
A) 21 B) 26 C) 29 D) 31

HOMEWORK
LEVEL 1
Q1. An element with atomic number 120 is being discovered, what would be its 𝐼𝑈𝑃𝐴𝐶 name?
Q2. Arrange the following atoms in the order of increasing atomic radius
I. 𝐵𝑟, 𝐼, 𝐹, 𝐶𝑙 II. 𝑇𝑒, 𝑆𝑒, 𝑆, 𝑂
III. 𝐾, 𝐿𝑖, 𝐶𝑠, 𝑁𝑎, 𝑅𝑏 IV. 𝐹, 𝑂, 𝐵, 𝑁, 𝐶
Q3. Which of the following is the correct order of size of the given species?
A) 𝐼 > 𝐼 − > 𝐼+ B) 𝐼 + > 𝐼− > 𝐼
C) 𝐼 > 𝐼 + > 𝐼− D) 𝐼 − > 𝐼 > 𝐼+
Q4. State if the term atomic radius will refer to metallic radius or covalent radius for each of the following elements.
𝐿𝑖, 𝐾, 𝑂, 𝑁𝑎, 𝑀𝑔, 𝐹, 𝐶𝑙, 𝐴𝑟, 𝐴𝑙, 𝑁𝑒

C3.1
C3 – Periodic Properties of Elements 71

LEVEL 2
Q5. Two elements in period 3 are adjacent to one another in the periodic table. The ground state configuration of one
has electrons only in the valence 𝑠 subshell, while the other has one electron in its valence 𝑝 subshell. Identify
the elements.
Q6. Write down the electronic configuration of the element which belongs to group 16 and the 4 th period of the
modern periodic table. What block does this element belong to?
Q7. Match the element with the correct atomic radius:
Element Radius
𝐵𝑒 74
𝐶 88
𝑂 111
𝐵 77
𝑁 66

Q8. Which of the following describes the correct relationship of ionic radii?
A) 𝑇𝑖 4+ < 𝑀𝑛7+ B) 35𝐶 𝑙 > 37𝐶 𝑙 −
C) 𝐾 + > 𝐶𝑙 − D) 𝑃3+ > 𝑃5+
Q9. In which period do the elements with the following atomic numbers lie?
A) 𝑍 = 30 B) 𝑍 = 45 C) 𝑍 = 50
C3 – Periodic Properties of Elements 72

C3.2 Periodic Trends of Properties


CONCEPTS
1. Ionization enthalpy, electron gain enthalpy (electron affinity), electronegativity
2. Identification of metals, non – metals, and trends in their properties
3. Trends of electronegativity, ionization enthalpy, electron gain enthalpy
4. Valency, common valencies of an element given its position in the periodic table
5. Prediction of formulae of binary compounds
6. Diagonal relationship

PRE-READING (60 MINS)


Category Book Name (Edition) Chapter Section

COMPULSORY NCERT XI 3 3.7.1 (b) to 3.7.3

PRE READING EXERCISE


Q1. Electronegativity is a relative property. True or False?
Q2. Lithium unlike other alkali metals forms compounds with covalent character. (True/False)
Q3. Beryllium like other alkaline earth metals forms compound with ionic character. (True/False)

IN-CLASS EXERCISE
LEVEL 1
Q1. Ionization enthalpy 1, 2 and 3 (in 𝑘𝐽/𝑚𝑜𝑙) of two unknown elements is given below. Identify the group to which
these elements belong:
Ionization Ionization Ionization Group of the
Element
Enthalpy 1 Enthalpy 2 Enthalpy 3 periodic table
A 500 1900 2100
B 1400 1600 2550

Q2. Among the elements with following atomic numbers 9, 11 and 18, identify the element which is:
I. highly electronegative II. inert gas III. highly electropositive
Q3. Arrange the first ionization enthalpy of 𝑁𝑎, 𝑀𝑔, 𝐴𝑙, 𝑆𝑖 in the correct order.
Q4. Predict the formulae of the stable binary compounds that would be formed by the combination of the following
pairs of elements
I. Lithium and oxygen II. Magnesium and oxygen
III. Aluminium and iodine IV. Phosphorous and fluorine

C3.2
C3 – Periodic Properties of Elements 73

LEVEL 2
Q5. Fourth ionization potential of 𝐶, 𝑁 and 𝑂 is in the order:
A) 𝐶 > 𝑂 > 𝑁 B) 𝑂 > 𝐶 > 𝑁 C) 𝐶 > 𝑁 > 𝑂 D) 𝑁 > 𝑂 > 𝐶
Q6. Arrange the following hydroxides in increasing order of their basic character
I. 𝑁𝑎𝑂𝐻 II. 𝑅𝑏𝑂𝐻 III. 𝐾𝑂𝐻 IV. 𝐶𝑠𝑂𝐻
Q7. Which of the three elements – 𝑁𝑎, 𝑀𝑔 or 𝑆𝑖 would have the greatest difference between first and second
ionization enthalpies?
Q8. From the ground state electronic configuration of the elements given below, pick up the one with the highest
value of second ionization energy:
A) 1𝑠 2 2𝑠 2 2𝑝6 3𝑠 2 B) 1𝑠 2 2𝑠 2 2𝑝6 3𝑠1 C) 1𝑠 2 2𝑠 2 2𝑝6 D) 1𝑠 2 2𝑠 2 2𝑝5
Q9. The first four 𝐼. 𝐸. values of an element are 284, 412, 656 and 3210 𝑘𝐽 𝑚𝑜𝑙 −1 . The number of valence electrons in
the element are:
A) One B) Two C) Three D) Four
Q10. Two elements 𝐴 and 𝐵 have atomic numbers 16 and 19 respectively. Write down the formula of the compound
formed between 𝐴 and 𝐵.
Q11. Arrange the following in order of increasing metallic character:
A) 1𝑠 2 2𝑠 2 2𝑝6 B) 1𝑠 2 2𝑠 2 2𝑝5 C) 1𝑠 2 2𝑠 2 2𝑝4

HOMEWORK
LEVEL 1
Q1. Which of the following quantities has no unit?
A) Electron gain enthalpy B) Electronegativity
C) Atomic radius D) Ionization Enthalpy
Q2. Which of the following elements will gain one electron more readily in comparison to other elements?
A) 𝑆(𝑔) B) 𝑁𝑎(𝑔)
C) 𝑂(𝑔) D) 𝐶𝑙(𝑔)
Q3. Among halogens, the correct order of amount of energy released when an electron is added to the following
gaseous atoms (electron gain enthalpy) is :
A) 𝐹 > 𝐶𝑙 > 𝐵𝑟 > 𝐼 B) 𝐹 < 𝐶𝑙 < 𝐵𝑟 < 𝐼
C) 𝐹 < 𝐶𝑙 > 𝐵𝑟 > 𝐼 D) 𝐹 > 𝐶𝑙 < 𝐵𝑟 < 𝐼
Q4. Assertion: Nitrogen has higher first ionization energy than oxygen.
Reason: Atomic radius of nitrogen is less than that of oxygen.
A) Both assertion and reason are correct and reason is the correct explanation of the assertion.
B) Both assertion and reason are correct and reason is not the correct explanation of the assertion.
C) Assertion is correct but reason is wrong.
D) Assertion is wrong but reason is correct.
Q5. Classify the following oxides as acidic or basic:
I. 𝑆𝑂2 II. 𝐶𝑎𝑂 III. 𝑀𝑔𝑂 IV. 𝐶𝑂2 V. 𝑁𝑎2 𝑂
C3 – Periodic Properties of Elements 74

Q6. Compare qualitatively the first and second ionization enthalpies of copper and zinc.

LEVEL 2
Q7. Which of the following sequence correctly represents the decreasing acidic nature of oxides?
A) 𝐿𝑖2 𝑂 > 𝐵𝑒𝑂 > 𝐶𝑂2 > 𝑁2 𝑂3 > 𝐵2 𝑂3 B) 𝐶𝑂2 > 𝑁2 𝑂3 > 𝐵2 𝑂3 > 𝐿𝑖2 𝑂 > 𝐵𝑒𝑂
C) 𝑁2 𝑂3 > 𝐶𝑂2 > 𝐵2 𝑂3 > 𝐵𝑒𝑂 > 𝐿𝑖2 𝑂 D) 𝐿𝑂2 > 𝐵𝑒𝑂 > 𝐿𝑖2 𝑂 > 𝐵2 𝑂3 > 𝑁2 𝑂3
Q8. Arrange the following elements in the order of ionization energy 𝑂, 𝐹, 𝑁, 𝐶
Q9. Electronic configuration of four elements 𝐼, 𝐼𝐼, 𝐼𝐼𝐼 and 𝐼𝑉 are given below :
I. 1𝑠 2 2𝑠 2 2𝑝6 II. 1𝑠 2 2𝑠 2 2𝑝4
III. 1𝑠 2 2𝑠 2 2𝑝6 3𝑠1 IV. 1𝑠 2 2𝑠 2 2𝑝5
Which of the following is the correct order of increasing tendency to gain one electron:
A) 𝐼 < 𝐼𝐼𝐼 < 𝐼𝐼 < 𝐼𝑉 B) 𝐼 < 𝐼𝐼 < 𝐼𝐼𝐼 < 𝐼𝑉
C) 𝐼𝑉 < 𝐼𝐼 < 𝐼𝐼𝐼 < 𝐼 D) 𝐼𝑉 < 𝐼 < 𝐼𝐼 < 𝐼𝐼𝐼
Q10. In the series carbon, nitrogen, oxygen and fluorine, electronegativity:
A) Decreases from carbon to fluorine
B) Remains constant
C) Decreases from carbon to oxygen and then increases
D) Increases from carbon to fluorine
Q11.
Column I Column II
A. 𝐻𝑒 i) High electron gain enthalpy
B. 𝐶𝑙 ii) Most electropositive element
C. 𝐶𝑠 iii) Inert gas
D. 𝐹 iv) Most electronegative element

The correct match of Column I with options in Column II is:


A) A − (iii), B − (i), C − (ii), D − (iv) B) A − (iv), B − (iii), C − (ii), D − (i)
C) A − (i), B − (ii), C − (iii), D − (iv) D) A − (iv), B − (i), C − (ii), D − (iii)
Q12. The valence shell of element 𝐴 contains 3 electrons while the valence shell of element 𝐵 contains 6 electrons. If
𝐴 combines with 𝐵, the probable formula of the compound formed will be:
A) 𝐴𝐵2 B) 𝐴2 𝐵 C) 𝐴2 𝐵3 D) 𝐴3 𝐵2

C3.2
C3 – Periodic Properties of Elements 75

Test Practice Problems


Purpose: To practice a mixed bag of questions in a speed based format similar to what you will face in entrance
examinations. In most entrance examinations, you will get not more than 3 minutes to attempt a question. Hence,
you need to be able to attempt a question in less than 3 minutes, and at the end of 3 minutes skip the question and
move to the next one.
Approach:
 Attempt the Test Practice Problems only when you have the stipulated time available at a stretch.
 Start a timer and attempt the section as a test.
 DO NOT look at the anwer key / solutions after each question.
 DO NOT guess a question if you do not know it. Competitive examinations have negative marking.
 Solve as much as possible within the stipulated time, and then fill the OMR provided at the end of the TPP.
 Fill the table at the end of the TPP and evaluate the number of attempts, and accuracy of attempts, which will
help you evaluate your preparedness level for the chapter.

TEST PRACTICE PROBLEMS – 1


No. of questions: 30 Total time: 90 Minutes Time per question: 3 Minutes

Q1. 𝑀3+ has electronic configuration as [𝐴𝑟] 3𝑑10 4𝑠 2 . Hence, 𝑀 lies in:
A) 𝑠-block B) 𝑝-block
C) 𝑑-block D) 𝑑- and 𝑓-block
Q2. Transition elements have vacant
A) 𝑠-orbitals B) 𝑝-orbitals C) 𝑑-orbitals D) 𝑓-orbitals
Q3. Transition metals are not characterized by:
A) Fixed valency B) Coloured compound
C) High melting and boiling points D) Tendency to from complexes
Q4. 𝐿𝑎 (Ianthanum) having atomic number 57 is a member of:
A) 𝑠-block B) 𝑝-block C) 𝑑-block D) 𝑓-block
Q5. Which of these will show maximum non-metallic character?
A) 𝐵𝑒 B) 𝐵 C) 𝑀𝑔 D) 𝐴𝑙
Q6. Which of the following explanations is the best reason for not placing hydrogen in either the group of alkali metals
or halogens?
A) The ionization energy of hydrogen is too-high for group of alkali metals, but too low for halogen group.
B) Hydrogen is much lighter element than the alkali metals or the halogens.
C) Hydrogen can form compounds with all other elements.
D) None of the above.
Q7. Electron affinity is the:
A) Energy released when an electron is added to an isolated atom in the gaseous state.
B) Energy absorbed when an electron is added to an isolated gaseous state.
C) Energy required to take out an electron from an isolated gaseous atom.
D) Power of an atom to attract an electron to itself.
C3 – Periodic Properties of Elements 76

Q8. The incorrect statement among the following is:


A) The first ionization potential of 𝐴𝑙 is less than the first ionization potential of 𝑀𝑔.
B) The second ionization potential of 𝑀𝑔 is greater than the second ionization potential of 𝑁𝑎.
C) The first ionization potential of 𝑁𝑎 is less than the first ionization potential of 𝑀𝑔.
D) The third ionization potential of 𝑀𝑔 is greater than that of 𝐴𝑙.
Q9. The electron affinity of halogens are 𝐹 = 322, 𝐶𝑙 = 349, 𝐵𝑟 = 324, 𝐼 = 295 𝑘𝐽 𝑚𝑜𝑙 −1 . The higher value for 𝐶𝑙 as
compared to that of 𝐹 is due to:
A) Weaker electron-electron repulsion in 𝐶𝑙. B) Higher atomic radius of 𝐹.
C) Smaller electronegativity of 𝐹. D) More vacant 𝑝-sub-shell in 𝐶𝑙.
Q10. STATEMENT-1: The first ionization energy of 𝐵𝑒 is greater than that of 𝐵.
STATEMENT-2: 2𝑝 orbital is lower in energy than 2𝑠.
A) Statement-1 is True, Statement-2 is True; Statement-2 is a correct explanation for Statement-1.
B) Statement-2 is True, Statement-2 is True; Statement-2 is NOT a correct explanation for Statement-1.
C) Statement-1 is True, Statement-2 is False.
D) Statement-1 is False, Statement-2 is True.
Q11. According to the Modern Periodic Law, the properties of elements repeat at regular intervals when the elements
are arranged in order of:
A) Decreasing atomic number B) Increasing atomic weight
C) Increasing atomic number D) Decreasing atomic weights
Q12. Give the symbol of the element of lowest atomic number that has three 2𝑝 electrons
A) 𝑀𝑔 B) 𝑃 C) 𝑁 D) 𝑆𝑖
Q13. The second ionization potential of 𝐿𝑖, 𝐵𝑒 and 𝐵 is in the order:
A) 𝐿𝑖 > 𝐵𝑒 > 𝐵 B) 𝐿𝑖 > 𝐵 > 𝐵𝑒
C) 𝐿𝑖 > 𝐵𝑒 > 𝐵 D) 𝐵 > 𝐵𝑒 > 𝐿𝑖
Q14. The first ionization energy of boron is less than that of beryllium because:
A) Beryllium has a higher nuclear change than boron
B) Beryllium has a lower nuclear change than boron
C) The outermost electron in boron occupies 2𝑝-orbital
D) The 2𝑠 and 2𝑝 orbitals of boron are degenerate
Q15. The ionic radii of 𝐿𝑖 + , 𝐵𝑒 2+ and 𝐵3+ follow the order:
A) 𝐵𝑒 2+ > 𝐵3+ > 𝐿𝑖 + B) 𝐿𝑖 + > 𝐵3+ > 𝐵𝑒 2+
C) 𝐵3+ > 𝐵𝑒 2+ > 𝐿𝑖 + D) 𝐿𝑖 + > 𝐵𝑒 2+ > 𝐵3+
Q16. Which one of the following is the correct order of increase in size?
A) 𝑀𝑔 < 𝑁𝑎+ < 𝐹 − < 𝐴𝑙 B) 𝑁𝑎+ < 𝐴𝑙 < 𝑀𝑔 < 𝐹 −
C) 𝑁𝑎+ < 𝐹 − < 𝐴𝑙 < 𝑀𝑔 D) 𝑁𝑎+ < 𝐹 − < 𝑀𝑔 < 𝐴𝑙
Q17. The largest species amongst the following is:
A) 𝑁𝑎+ B) 𝑁𝑒
C) 𝐹 − D) All are equal in size
Q18. The first 𝐼. 𝐸. of 𝑁𝑎, 𝑀𝑔, 𝐴𝑙 and 𝑆𝑖 are in the order:
A) 𝑁𝑎 < 𝑀𝑔 < 𝐴𝑙 < 𝑆𝑖 B) 𝑁𝑎 < 𝐴𝑙 < 𝑀𝑔 < 𝑆𝑖
C) 𝑁𝑎 < 𝐴𝑙 < 𝑆𝑖 < 𝑀𝑔 D) 𝑁𝑎 > 𝑀𝑔 > 𝐴𝑙 > 𝑆𝑖

T.P.P.
C3 – Periodic Properties of Elements 77

Q19. The ionization energy will be higher when an electron is removed from:
A) 𝑠-subshell B) 𝑝-subshell C) 𝑑-subshell D) 𝑓-subshell
Q20. Which of the following isoelectronic ions has the lowest first ionization energy?
A) 𝐾 + B) 𝐶𝑙 − C) 𝐶𝑎2+ D) 𝑆 2−
Q21. The most electropositive element possesses the electronic configuration:
A) [𝐻𝑒]2𝑠1 B) [𝑁𝑒]3𝑠 2 C) [𝑋𝑒]6𝑠1 D) [𝑋𝑒]6𝑠 2
Q22. Amongst the following elements, the highest first ionization energy is possessed by:
A) [𝑁𝑒]3𝑠 2 3𝑝1 B) [𝑁𝑒]3𝑠 2 3𝑝3
C) [𝑁𝑒]3𝑠 2 3𝑝2 D) [𝐴𝑟]3𝑑10 4𝑠 2 4𝑝3
Q23. Which of the following is arranged in decreasing order of size?
A) 𝑀𝑔2+ > 𝐴𝑙 3+ > 𝑂2− B) 𝑂2− > 𝑀𝑔2+ > 𝐴𝑙 3+
C) 𝐴𝑙 3+ > 𝑀𝑔2+ > 𝑂2− D) 𝐴𝑙 3+ > 𝑂2− > 𝑀𝑔2+
Q24. Which of the following electronic configuration is associated with the biggest jump between the second and third
ionization energies?
A) 1𝑠 2 2𝑠 2 2𝑝2 B) 1𝑠 2 2𝑠 2 2𝑝6 3𝑠1 C) 1𝑠 2 2𝑠 2 2𝑝6 3𝑠 2 D) 1𝑠 2 2𝑠 2 2𝑝1
Q25. The number of elements present in the fifth period is:
A) 18 B) 32 C) 8 D) 24
Q26. A large difference between the fourth and fifth ionization energies indicates the presence of:
A) 5 valence electrons in an atom B) 6 valance electrons in an atom
C) 4 valence electrons in an atom D) 8 valence electrons in an atom
Q27. The element with the lowest atomic number that has a ground state electronic configuration of (𝑛 − 1)𝑑 6 𝑛𝑠 2 is
located in the:
A) Fifth period B) Sixth period
C) Fourth period D) Third period
Q28. Which of the following sets of atomic numbers corresponds to elements of group 16?
A) 8, 16, 32, 54 B) 16, 34, 54, 86 C) 8, 16, 34, 52 D) 10, 16, 32, 50
Q29. Among the following configurations, the element which has the highest electron affinity is:
A) [𝑁𝑒]3𝑠1 3𝑝2 B) [𝑁𝑒]3𝑠 2 3𝑝5
C) [𝑁𝑒]3𝑠 2 3𝑝4 D) [𝑁𝑒]3𝑠 2 3𝑝6 3𝑑 5 4𝑠1
Q30. The increasing order of electron affinity of the electronic configurations of elements 𝐼, 𝐼𝐼, 𝐼𝐼𝐼 and 𝐼𝑉 given below
is:
I. 1𝑠 2 2𝑠 2 2𝑝6 3𝑠 2 3𝑝5 II. 1𝑠 2 2𝑠 2 2𝑝3
III. 1𝑠 2 2𝑠 2 2𝑝5 IV. 1𝑠 2 2𝑠 2 2𝑝6 3𝑠1
A) IV < II < III < I B) I < II < III < IV
C) I < III < II < IV D) IV < III < II < I
C3 – Periodic Properties of Elements 78

TEST PRACTICE PROBLEMS – 2


No. of questions: 25 Total time: 75 Minutes Time per question: 3 Minutes

Q31. Ionization enthalpy of an atom is numerically equal to:


A) Electron gain enthalpy of the cation B) Electronegativity of the ion
C) Ionization enthalpy of the cation D) None of these
Q32. The size of the species, 𝑃𝑏, 𝑃𝑏 2+ , 𝑃𝑏 4+ decreases as:
A) 𝑃𝑏 4+ > 𝑃𝑏 2+ > 𝑃𝑏 B) 𝑃𝑏 > 𝑃𝑏 2+ > 𝑃𝑏 4+
C) 𝑃𝑏 > 𝑃𝑏 4+ > 𝑃𝑏 2+ D) 𝑃𝑏 4+ > 𝑃𝑏 > 𝑃𝑏 2+
Q33. The atomic numbers of the metallic and non-metallic elements which are liquid at room temperature respectively
are:
A) 55, 87 B) 33,87 C) 35, 80 D) 80, 35
Q34. The element having very high ionization enthalpy and extremely low electron affinity is:
A) 𝐻 B) 𝐹 C) 𝐻𝑒 D) 𝐵𝑒
Q35. An element has successive ionization enthalpies as 940 (first 𝐼𝐸 ), 2080, 3090, 4140, 7030, 7870, 16000 and
19500 𝑘𝐽 𝑚𝑜𝑙 −1 . To which group of the periodic table does this element belong?
A) 14 B) 15 C) 16 D) 17
Q36. In the periodic table, metallic character of the elements shows which of the following trends?
A) Decreases down the group and increases across the period
B) Increases down the group and decreases across the period
C) Increases across the period and also down the group
D) Decreases across the period and also down the group
Q37. Nucleus of the atom of a neutral element contains 11 protons. It’s valency would be:
A) 1 B) 2 C) 3 D) 5
Q38. Among 𝐴𝑙2 𝑂3 , 𝑆𝑖𝑂2 , 𝑃2 𝑂3 and 𝑆𝑂2 , the correct order of acidic nature is:
A) 𝑃2 𝑂3 < 𝐴𝑙2 𝑂3 < 𝑆𝑖𝑂2 < 𝑆𝑂2 B) 𝑆𝑖𝑂2 < 𝑆𝑂2 < 𝐴𝑙2 𝑂3 < 𝑃2 𝑂3
C) 𝐴𝑙2 𝑂3 < 𝑆𝑖𝑂2 < 𝑃2 𝑂3 < 𝑆𝑂2 D) 𝑆𝑂2 < 𝑃2 𝑂3 < 𝑆𝑖𝑂2 < 𝐴𝑙2 𝑂3
Q39. An element 𝑋 which occurs in the second period has an outer electronic structure 𝑠 2 𝑝1 . The formula and acid-
base character of its oxide, respectively are,
A) 𝑋𝑂3 , basic B) 𝑋2 𝑂3 , basic
C) 𝑋2 𝑂3 , amphoteric D) 𝑋𝑂2 , acidic
Q40. The correct order of the increasing ionic character is:
A) 𝐵𝑒𝐶𝑙2 < 𝑀𝑔𝐶𝑙2 < 𝐶𝑎𝐶𝑙2 < 𝐵𝑎𝐶𝑙2 B) 𝐵𝑒𝐶𝑙2 < 𝐵𝑎𝐶𝑙2 < 𝐶𝑎𝐶𝑙2 < 𝑀𝑔𝐶𝑙2
C) 𝐵𝑎𝐶𝑙2 < 𝐶𝑎𝐶𝑙2 < 𝑀𝑔𝐶𝑙2 < 𝐵𝑒𝐶𝑙2 D) 𝑀𝑔𝐶𝑙2 < 𝐶𝑎𝐶𝑙2 < 𝐵𝑎𝐶𝑙2 < 𝐵𝑒𝐶𝑙2
Q41. The first ionization potential of 𝑁𝑎 is 5.1 𝑒𝑉. The value of electron gain enthalpy of 𝑁𝑎+ will be:
A) −2.55 𝑒𝑉 B) −5.1 𝑒𝑉 C) −10.2 𝑒𝑉 D) +2.55 𝑒𝑉
Q42. Which one of the following set of ions represents a collection of isoelectronic species?
A) 𝐵𝑎2+ , 𝑆𝑟 2+ , 𝐾 + , 𝐶𝑎2+ B) 𝑁 3− , 𝑂2− , 𝐹 − , 𝑆 2−
C) 𝐿𝑖 + , 𝑁𝑎+ , 𝑀𝑔2+ , 𝐶 𝑎2+ D) 𝐾 + , 𝐶𝑙 − , 𝐶𝑎2+ , 𝑆𝑐 3+

T.P.P.
C3 – Periodic Properties of Elements 79

Q43. Identify the least stable ion amongst the following:


A) 𝐵𝑒 − B) 𝐿𝑖 − C) 𝐵− D) 𝐶 −
Q44. Which of the following represents correct order of decreasing size of the ions?
A) 𝐶𝑎2+ > 𝐾 + > 𝐶𝑙 − > 𝑆 2− B) 𝐶𝑙 − > 𝑆 2− > 𝐾 + > 𝐶𝑎2+
C) 𝑆 2− > 𝐶𝑙 − > 𝐶𝑎2+ > 𝐾 + D) 𝑆 2− > 𝐶𝑙 − > 𝐾 + > 𝐶𝑎2+
Q45. In which of the following arrangements, the order is not according to the property indicated against it?
A) 𝐴𝑙 3+ < 𝑀𝑔2+ < 𝑁𝑎+ < 𝐹 −
Increasing ionic size
B) 𝐵 < 𝐶 < 𝑁 < 𝑂
Increasing first ionization enthalpy
C) 𝐼 < 𝐵𝑟 < 𝐹 < 𝐶𝑙
Increasing electron gain enthalpy (considering just numerical value)
D) 𝐿𝑖 < 𝑁𝑎 < 𝐾 < 𝑅𝑏
Increasing metallic radius
Q46. Which of the following sequence correctly represents the decreasing acidic nature of oxides?
A) 𝐿𝑖2 𝑂 > 𝐵𝑒𝑂 > 𝐶𝑂2 > 𝑁2 𝑂3 > 𝐵2 𝑂3 B) 𝐶𝑂2 > 𝑁2 𝑂3 > 𝐵2 𝑂3 > 𝐿𝑖2 𝑂 > 𝐵𝑒𝑂
C) 𝑁2 𝑂3 > 𝐶𝑂2 > 𝐵2 𝑂3 > 𝐵𝑒𝑂 > 𝐿𝑖2 𝑂 D) 𝐶𝑂2 > 𝐵𝑒𝑂 > 𝐿𝑖2 𝑂 > 𝐵2 𝑂3 > 𝑁2 𝑂3
Q47. Consider the following species
𝑁 3− , O2− , 𝐹 − , 𝑁𝑎+ , 𝑀𝑔2+ and 𝐴𝑙 3+
What is common in them?
A) Number of electrons B) Number of valence electrons
C) Number of protons D) All of the above
Q48. Which one of the following ions has the highest value of ionic radius?
A) 𝐿𝑖 + B) 𝐵3+ C) 𝑂2− D) 𝐹 −
Q49. The atomic numbers of vanadium(𝑉), chromium(𝐶𝑟), manganese (𝑀𝑛) and iron (𝐹𝑒) are, respectively 23, 24, 25
and 26. Which one of these may be expected to have the highest second ionization enthalpy?
A) 𝑉 B) 𝐶𝑟 C) 𝑀𝑛 D) 𝐹𝑒
Q50. Which of the following is a metalloid?
A) 𝑆𝑏 B) 𝑀𝑔 C) 𝑍𝑛 D) 𝐵𝑖
Q51. Which one of the following order represents the correct sequence of the increasing basic nature of the given
oxides?
A) 𝐴𝑙2 𝑂3 < 𝑀𝑔𝑂 < 𝑁𝑎2 𝑂 < 𝐾2 𝑂 B) 𝑀𝑔𝑂 < 𝐾2 𝑂 < 𝐴𝑙2 𝑂3 < 𝑁𝑎2 𝑂
C) 𝑁𝑎2 𝑂 < 𝐾2 𝑂 < 𝑀𝑔𝑂 < 𝐴𝑙2 𝑂3 D) 𝐾2 𝑂 < 𝑁𝑎2 𝑂 < 𝐴𝑙2 𝑂3 < 𝑀𝑔𝑂
Q52. According to IUPAC nomenclature, a newly discovered element has been named as Uun. The atomic number of
the element is
A) 111 B) 112 C) 109 D) 110
Q53. Which of the following oxides is amphoteric in character?
A) 𝐶𝑎𝑂 B) 𝐶𝑂2 C) 𝑆𝑖𝑂2 D) 𝑆𝑛𝑂2
Q54. Which of the following is least soluble in water?
A) 𝐵𝑒𝐶𝑂3 B) 𝑀𝑔𝐶𝑂3 C) 𝑁𝑎2 𝐶𝑂3 D) 𝐶𝑎𝐶𝑂3 .
C3 – Periodic Properties of Elements 80

Q55. Ionisation enthalpy of lithium is 520 𝑘𝐽 𝑚𝑜𝑙 −1 . How much energy in joules must be needed to convert all atoms
of lithium to ions present in 7 𝑚𝑔 of lithium vapours?
A) 74.3 𝑘𝐽 B) 260 𝑘𝐽 C) 520 𝐽 D) 780 𝑘𝐽

DATA Guide
A # of questions Total problems in TPP
B # Attempts Total attempts in OMR
C # Correct Total questions correct
D # Incorrect Out of the ones marked in OMR
E # Unattempted 𝐴– 𝐵
ANALYSIS
F Percentage attempts 𝐵
× 100
𝐴
G Percentage Accuracy 𝐶
× 100
𝐵

Question type # Correct (C) # Incorrect (I) # Unattempted (U)


Level 1
Level 2
Level 3

Tip: To begin with, your accuracy must be high, typically > 60%. Percentage attempts should be > 50%

As time progresses, your percentage attempts should increase without a reduction in accuracy.

Additionally, you should be able to get > 80% Level 1 questions correct, as they involve basic recall of the concepts
and formulae of the chapter.

T.P.P.
C3 – Periodic Properties of Elements 81

Answer Key
C3.1 EARLY CLASSIFICATIONS OF ELEMENTS AND THE MODERN PERIODIC TABLE
PRE-READING EXERCISE II. 𝑆 2− > 𝐶𝑙 − > 𝐾 + > 𝐶𝑎2+
Q1. False
LEVEL 3
Q2. Combined
Q9. C
Q3. Metallic, Covalent
Q10. D
Q4. X-ray or spectroscopic
Q5. Decreases HOMEWORK
LEVEL 1
Q6. Smaller, Larger
Q1. Unbinilium
IN-CLASS EXERCISE
Q2. I. 𝐹 < 𝐶𝑙 < 𝐵𝑟 < 𝐼
LEVEL 1 II. 𝑂 < 𝑆 < 𝑆𝑒 < 𝑇𝑒
Q1. I. Unniloctium, 𝑈𝑛𝑜 III. 𝐿𝑖 < 𝑁𝑎 < 𝐾 < 𝐶𝑠 < 𝑅𝑏
II. Ununpentium, 𝑈𝑢𝑝 IV. 𝐵>𝐶>𝑁>𝑂>𝐹
III. Ununnilium, 𝑈𝑢𝑛 Q3. D
Q2. I. Group 4 Q4. This is the correct matching:
II. Group 13
III. Group 17 Metallic radius Covalent radius

Q3. 35 𝐾 𝑂

Q4. I. 𝐵𝑒 < 𝑀𝑔 < 𝐶𝑎 < 𝑆𝑟 < 𝐵𝑎 𝐴𝑙 𝐹


II. Na > Mg > P > O > F 𝐿𝑖 𝐶𝑙
III. 𝑆𝑐 > 𝑀𝑛 > 𝐶𝑜 > 𝑁𝑖 > 𝐶𝑢
𝑁𝑎 𝐴𝑟
Q5. I. 𝑑-block 𝑀𝑔 𝑁𝑒
II. 𝑝-block
III. 𝑓-block LEVEL 2
IV. 𝑠-block
Q5. 𝑀𝑔 and 𝐴𝑙
LEVEL 2 Q6. 1𝑠 2 2𝑠 2 2𝑝6 3𝑠 2 3𝑝6 4𝑠 2 3𝑑10 4𝑠 2 4𝑝4 ; 𝑝-block
Q6. I. Halogens
Q7. 𝐵𝑒 = 111𝑝𝑚 (𝐻𝑖𝑔ℎ𝑒𝑠𝑡), 𝐵 = 88 𝑝𝑚, 𝐶 =
II. Transition element
77 𝑝𝑚, 𝑁 = 74 𝑝𝑚, 𝑂 = 66 𝑝𝑚
III. Actinoids
IV. Alkali Metal Q8. D
V. Transition metal Q9. A) 4th Period
Q7. 𝐴𝑙 3+
< 𝑀𝑔 +2
< 𝐴𝑙 < 𝑀𝑔 B) 5th Period
C) 5th Period
Q8. I. 𝑂2− > 𝐹 − > 𝑁𝑎+ > 𝑀𝑔2+ > 𝐴𝑙 3+
C3 – Periodic Properties of Elements 82

C3.2 PERIODIC TRENDS OF PROPERTIES


PRE-READING EXERCISE Q10. 𝐴𝐵2

Q1. True Q11. C

Q2. True HOMEWORK


Q3. False LEVEL 1

IN CLASS EXERCISE Q1. B

LEVEL 1 Q2. D

Q1. A. 1st Group Q3. C


B. 2nd Group Q4. C
Q2. I. Electronegative ⇒ 𝑍 = 9 Q5. I. Acidic
II. Noble Gas ⇒ 𝑍 = 18 II. Basic
III. Electropositive ⇒ 𝑍 = 11 III. Basic
IV. Acidic
Q3. 𝑁𝑎 < 𝑀𝑔 > 𝐴𝑙 < 𝑆𝑖
V. Basic
Q4. I. 𝐿𝑖2 𝑂
Q6. First 𝐼𝐸 ∶ 𝐶𝑢 < 𝑍𝑛 and Second 𝐼𝐸 ∶ 𝐶𝑢+ > 𝑍𝑛+
II. 𝑀𝑔𝑂
III. 𝐴𝑙𝐼3 LEVEL 2
IV. 𝑃𝐹3
Q7. C
LEVEL 2 Q8. 𝐶<𝑁>𝑂<𝐹
Q5. D Q9. A
Q6. 𝑁𝑎𝑂𝐻 < 𝐾𝑂𝐻 < 𝑅𝑏𝑂𝐻 < 𝐶𝑠𝑂𝐻 Q10. D
Q7. 𝑁𝑎 Q11. A
Q8. B Q12. C
Q9. C

Ans.
C3 – Periodic Properties of Elements 83

ANSWERS TO TEST PRACTICE PROBLEMS


Mark (C) / (I) / (U)
Mark (C) / (I) / (U) Q. No. Ans. Level
Q. No. Ans. Level as appropriate
as appropriate
Q28. C Medium
Q1. B Easy
Q29. B Medium
Q2. C Easy
Q30. A Medium
Q3. A Easy
Q31. A Easy
Q4. C Medium
Q32. B Medium
Q5. B Medium
Q33. D Easy
Q6. A Easy
Q34. C Medium
Q7. A Easy
Q35. C Medium
Q8. B Medium
Q36. B Easy
Q9. A Easy
Q37. A Medium
Q10. C Medium
Q38. C Medium
Q11. C Easy
Q39. C Medium
Q12. C Medium
Q40. A Medium
Q13. B Medium
Q41. B Easy
Q14. C Easy
Q42. D Medium
Q15. D Medium
Q43. A Medium
Q16. C Medium
Q44. D Medium
Q17. C Easy
Q45. B Medium
Q18. A Medium
Q46. C Medium
Q19. A Easy
Q47. A Easy
Q20. D Medium
Q48. C Medium
Q21. C Easy
Q49. B Medium
Q22. B Medium
Q50. A Easy
Q23. B Medium
Q51. A Medium
Q24. C Medium
Q52. D Easy
Q25. A Easy
Q53. D Easy
Q26. C Medium
Q54. A Medium
Q27. C Medium
Q55. C Medium

You might also like